You are on page 1of 370

-


. .

2008

53(023)
22.3
-13

. . , . . , . . , . . ,
. .
. . ,
: . / . . , . . ,
. . .; . . . . 3- ., . .
: , 1999. 370 ., .
ISBN 5861340242.
2000 - . , , .
, - .
, .
,
.
, , , , .
. 973.
:
;
.-. . .
.-. . .

c , 1999


6
6
8

1.
1.1.
1.2.
1.3.
1.4.
1.5.

. . . . . . . . . . . . . . . . . . . . . . . . . . . .
. . . . . . . . . . . . . . . . . . . . . . . . . . . .
. . . . . . . . . . .
. . . . . . . . . . . . . . . . . . . . . . . . . . . . . . . . . . . . . . .
. . . . . . . . . . . . . . . . . . . . . . . . . . . . . . . . . . . . . . . . . .

9
13
16
19
22

280
282
283
284
285

25
33
39
45
51
56
61
67

285
287
288
290
291
292
294
295

74
76
81
86
89
93
97
99
102

296
296
297
298
300
303
304
306
307

2.
2.1.
2.2.
2.3.
2.4.
2.5.
2.6.
2.7.
2.8.

. . . . . . . . . . . . . . . . . . . . . . . . . . . . . . . . . . . . . . . . . . . . . . .
. . . . . . . . . . . . . . . . . . . . . . . . . . . . . . . . . . . . . . . . . . .
. . . . . . . . .
. . . . . . . . . . . . . . . .
. . . . . . . . . . . . . . . . . . . . . . . . . . . . . . . . . . . . . . . . . . . . . . . . . .
. . . . . . . . . . . . . . . . . . . . . . . . . . . . . . .
. . . . . . . . . . . . . . . . . . . . . . . . . . . . . . . . . . . . . . .
. . . . . . . . . . . . . . . . . . . . . . . . . . . . . . . . . . . . . . . . . . . . . . . . . . . . . . . .

3.
3.1.
3.2.
3.3.
3.4.
3.5.
3.6.
3.7.
3.8.
3.9.

. . . . . . . . . . . . . . . . . . . . . . . . . . . . . .
. . . . . . . . . . . . . . . . . . . . . . .
. . . . . . . . . . . . . . . . . . . . . . . . . . . . . . . . . . . . . .
. . . . . . . . . . . . . . . . . . . . . . . . . . . . . . . . . . . . . . . . . .
. . . . . . . . . . . . . . . . . . . . . . .
. . . . . . . . . . . . . . . . . . . . .
. . . . . . . . . . . . . . . . . . . . . . . . . . . . . . . . . . . . . . . . .
. . . . . . . . . . . . . . . . . . . . . . . . . . . . . . . . . .
. . . . . . . . . . . . . . . . . . . . . . . . . . . . . . . .

4.
4.1. . . . . . . . . . . . . . . . . . . . . . . . . . . . . . . . . . . . . . . . . . . . .

106

309

4.2. . . . . . . . . . . . . . . . . . . . . . . . . . . . . . . . . . . . . .

109

310

4.3. . . . . . . . . . . . . . . . . . . . . . . . . . . . . . . . .

113

310

4.4. . . . . . . . . . . . . . . . . . . . . . . . . . . . . . . . . . . . . . .

116

312

4.5. . . . . . . . . . . . . . . . . . . . . . . . . . . .

117

313

4.6. . . . . . . . . . . . . . . . . . . . . . . . . . . . . . . . . . . . . . . . . .

120

314

5.1. . . . . . . . . . . . . . . . . . . . . . . . . . . . . . . . . . . . .

124

315

5.2. . . . . . . . . . . . . . . . . . . . . .

125

315

5.3. . . . . . . . . . . . . . . . . . . . . .

128

315

5.4. .
. . . . . . . . . . . . . . . . . . . . . . . . . . . . . . . . . . . . . . . . . . . . . . . . . .

129

316

5.

5.5. . . . . . . . . . . . . . . . . . . . . . . . . .

131

317

5.6. . . . . . . . . . . . . . . . . . .

136

317

5.7. . . . . . . . . . . . . . . . . . . . . . . . . . . . . . . . . . . . . . . . . . . . . . . . . .

139

318

5.8. . . . . . . . . . . . . . . . . . .

140

319

5.9. . . . . . . . . . . . . . . . . . . . . . . . . . . . . . . . .

143

320

5.10. . . . . . . . . . . . . . . . . . . . . . . . . . . . . . . . . . . . . . . . . . . . . .

145

320

5.11. . . . . . . . . . . . . . . . . . . . . . . . . . . . . . . . . . . . . . . . . . . . .

148

321

6.1. . . . . . . . . . . .

151

322

6.2. . . .

154

322

6.3. .
. . . . . . . . . . . . . . . . . . . . . . . . . . . . . . . . . . . . . . . . . . . .

156

324

6.4. . . . . . . . . . . . . . . . . . . . . . . . . . . . . . . . . . . . . . . . . . . . . . . . . . .

161

325

6.

6.5. . . . . . . . .

163

325

6.6. . . . . . . . . . . . . . . . .

167

326

7.
7.1. . . . . . . . . . . . . . . . . . .

171

7.2. . . . . . . . . . . . . . . . . . . . . . . . . . . . . . .

175

328

7.3. . . . . . . . . . . . . . . . . . .

178

329

7.4. . . . . . . . . . . . . . . . . . . . . . . . . . . .

179

330

8.1. . . . . . . . . . . . . . . . . . . . . . . . . . . . .

184

332

8.2. . . . . . . . . . . . . . . . .

186

333

8.3. . . . . . . . . . . . . . . . . . . . . . . . . . . . . . . . . . . . . . . . . . . .

191

334

8.4.

198

335

327

8.

9.
9.1. .

202

336

9.2. .
. . . . . . . . . . . . . . . . . . . . . . . . . . . . . . . . . . . . . . . . . . .

204

337

9.3. ,
. . . . . . . . . . . . . . . . . . . . . . . . . . . . . . . . . . . . . . . . . . . . . .

207

338

9.4. . . . . . . . . . . . . . . . . . . . . . . . . . . . . . . . . . . . . . . . . . . . . . .

210

340

10.1. . . . . . . . . . . . . . . . . . . . . . .

213

341

10.2. . . . . . . . . . . . . . . . . . . . . . . . . . . . . . . . . . . .

218

342

10.

11.
11.1. . . . . . . . . . . . . . . . . . . . . . . . . . . . . . . . . . . . . . . . . . . . . . . . . . . .

220

342

11.2. . . . . . . . . . . . . . . . . . . . . . . . . . . . . . . . . . .

225

343

11.3. . .
. . . . . . . . . . . . . . . . . . . . . . . . . . . . . . . . . . . . . . . . . . . . . .

228

344

11.4. . . . . . . . . . . . . . . . . . . . . . . . .

231

345

11.5. .
. . . . . . . . . . . . . . . . . . . . . . . . . . . . . . . . . . . . . . . . . . . . . . . . . . . . . . . . . . . .

234

346

11.6. . . . . . . . .

238

348

12.1. , . . .

241

349

12.2. . . . . . . . . . . . . . . . . . . . . .

247

352

250

352

13.2. . . . . . . . . . . . . . . . . . . . . . . . . . . . .

252

353

13.3. . . . . . . . . . . . . . . . . . . . . . . . . . . . . . . . . . . . . . . . . . . .

254

354

13.4. . . . . . . . . . . . . . . . . . . . . . . . . . . . . . . . . . . . . . . . . . . . . . . . . . . . .

257

356

13.5. . . . . . . . . . . . . . . . . . . . . . . . . . . . . . . . . . . . . . .

260

356

262

357

12.

13. . .

13.1. . . . . . . . . . .

14.
14.1. . . . . . . . . . . . . . .

14.2. ,
. . . . . . . . . . . . . . . . . . . . . . . . . . . . .

267

363

14.3. . . . . . . . . . .

269

364

14.4.
. . . . . . . . . . . . . . . . . . . . . . . . . . . . . . . . . . . . . . . . . . . .

273

366

14.5. . . . . . . . . . . . . . . . . . . . . . . . . . . .

277

368
5


, , 300 000 , ,
- ,

.
, .
. .
.
. .

, - ,
, ,
.
: , . , , ( ) ,
. , .
.
.
6


, - . ,
. , , , ,
. , .
. ,
( 4.3), , ( 11.5).
. ,

. ,
, .
,
.
, . ,
,
. , , , ,
, .
. , , , ,
. .
.
. .

c
0
0 = (0 c2 )1
h
me
mp
mn

2,998 108 /
4 107 / = 1,257 106 /
8,85 1012 /
6,63 1034 /
9,11 1031
1,67 1027
1,67 1027

mp /me
e

1836,15
1,60 1019

e/me
e
NA
1 ...

1,76 1011 /
9,28 1024 /
6,02 1023 1
1,66 1027

F = NA e

9,65 104 /

8,31 /( )

T0

273,15

p0

1,01 105

Vm = RT0 /p0

22,41 103 3 /

k = R/NA

1,38 1023 /

5,67 108 /(2 4 )

6,67 1011 2 /2

gn

9,8 /2

1.1.
1.1.1. ) . 30 , 10 .
. 0,1 .
.
1.1.2.
, . : 1 = 44 ,
R1 = 100 . 5 : 2 = 46 ,
R2 = 100 . y, ,
; . .
1.1.3. .
1 /, ,
, 2 /, 2 /.
1 .
,
. 2,5 , 4 , 4 .
1.1.4. A B, -, 2 . 0 - -. ,
A - 109 , B.
3 108 /.
1.1.5 . , A, B, C,
tA > tB > tC
, O, AC. AO, AB = BC = L. ?
)

, .
9

1.1.6. l v.
u < v. , , .
, ?
1.1.7. , , 0 .
. c.
?
1.1.8. w. ,
. .
, . ,
v < w, u < w?

1.1.9. . l.
( )
v, u < v.
, ,
? .
. ,
.
?

10

1.1.10 . v. , A.
, u < v?
u = v/2.
1.1.11 . . , l ,
, ,
t. c. ?
1.1.12. 2
v .
?
1.1.13.
.

1.1.14.
, . v, v/2.
t = 0 x = 0, t1 x = a.
1.1.15.
.
.

1.1.16. .
vx vy ,
x(0) = 2 , y(0) = 1 .
11

1.1.17. x y .
y = x , x /3, 3x ? (. ). . ? x
y ?
1.1.18 . v ,
. , l,
. ,
? c.

1.1.19.
, ? ,
= /2? , .
.
1.1.20 . b. ,
, ?

12

1.1.21. , ,
. ,
c = (cx , cy , cz ),
, yOz? ?
1.1.22. R
, , h.
, r, h?

1.1.23 . R,
,
. .

R . ?
? ? L > 4R, 4R > L > 2R, L
.
1.2.
1.2.1. ,
. R r.
v.
.
1.2.2. t = 0 .
.
13

1.2.3. t0 v0 .
, 2t0
2v0 . , t > t0 .
1.2.4. , : )
2 6 5 /; ) 15 /.
1.2.5 , v1 v2 .
, ? l.
1.2.6. . v0 , t0 . ,
.

1.2.7. 20 ,
310 . 15 /. ,
.
1.2.8 . , , L, a.
?
1.2.9. , , .
,
.
N , bv 2 v.
,
? ( .)
1.2.10. R, ,
q.
?
14

1.2.11 . . v0 .
s? .
. , , h.
, q?
.
1.2.12. . 10
1 /.
?
1.2.13. ,
.
( . // . 1984. 12. . 111). .
?
1.2.14.
4 15 , 1 c 3 /.

1.2.15. 6 100 /2 , 7 ,
3 200 /2 . , .
? ? ?
, ?
1.2.16. , , . t 4 ,
1 .
A .
1.2.17. ,
t, , . .
x.
1.2.18.
, x(0) = 0.
1.2.19. 15 ;
150 /. ,
2 /2 .
15

1.2.20 . A t0 , . A?
1.2.21 . 12.00. 12.00, ,
t1 .
t2 . .
?
1.3. ) .
1.3.1. t
v.
?
1.3.2. .
. ,
D?
. A .
t?
)

16

g,
.

1.3.3 .
A ,

?
1.3.4.
A vA . B, h A?
1.3.5. v .
?
1.3.6. .
v. . : ) ; )
x y ; ) , . . y x;
) , .
1.3.7. v . , ?
45 .
45 .
1.3.8. ,
. ,
v, ?

1.3.9.
, , a?
L, 45 .
1.3.10. u.
, , . . v
. ,
?
2

17

1.3.11. , , v.
?
1.3.12 . , , 45
10 /. 5 2 .
, .
1.3.13 . , , x y . v. : )
, ; )
; ) ,
x, y.
.
1/ cos2 = tg2 + 1.
1.3.14. t
v . ?
t?
1.3.15. R .
, ?
1.3.16 . l, ,
v.
, .
1.3.17. , ,
.
t. .
.

1.3.18 . ,
, .
T1 ,
T2 6= T1 . .
1.3.19 . ,
, H L, h ?
18

1.3.20. , - - . 6400 . 60 .
1.3.21. , ,
g, ?
R = 6400 , g = 10 /2 .
1.3.22 . v. l.
, , .
?
1.3.23. v ,
R R/3. .

1.3.24. , 106 /, 104 /2 30 .


102 ? ?
?
1.3.25. r , v = kt.
.
1.3.26.
r. , , ,
?
1.3.27 . , , R.

, ?
1.3.28. 600 / 30 ,
45 , 60 .
.
1.3.29.
, R.
. ,
v?
1.3.30 . V .
?
1.4.
19

1.4.1. .
. ?
1.4.2. , . ,
1.

1.4.3. ( A) , ,
A. , B?
1.4.4. A
. , ,
4/3 .
1.4.5. . - v, . , u ,
?
. 10 / 30 .
45 ?
1.4.6 . .
, . v, .
30 .
?
1.4.7 .
, ,
u? v, L. ?
1.4.8. v .
, : ) u ; )
w < v .
1.4.9. v , . , :
) ; ) w ; ) , , w .
20

1.4.10. R, u,
r, , ,
v, u.
. , .
.
1.4.11. h . u.
. .
1.4.12. v
, u. n-
. L. .
1.4.13. R .
v1 v2 .
?
1.4.14 . , v,
. v,
u < v.
1.4.15 . , v.
. , , 3v. ,
, .
1.4.16. , v,
. . ,
. , ,
. .
2

21

1.4.17. . u. v .
, , ?
, ?
1.4.18 . , ,
, ,
. ?
, 200 ?
1.5.
1.5.1. A vA . B?
1.5.2. , r,
R. ?
1.5.3. , 30 , . , v,
. ?

1.5.4. .
,
?
1.5.5 . , , .
.
1.5.6. ,
. A B (1, 2)
(3, 1), (3, 1) (2, 3).
.
1.5.7. . A v 45
AB. B u.
B AB.
. A B v. C, AB v, u > v.
C , .

22

1.5.8. ,
. , : r > R, r = R, r < R. ,
v. , r 6= R
.
1.5.9 . , , v
. . .
? ? ,
.

1.5.10. 2r r.
.
1.5.11. . . ?
. ?
365, 25 .
1.5.12. R, , .
. .
1.5.13. , , , . v,
,
. ?

1.5.14 . a.
v.
.
?
23

1.5.15. B , vA A . ,
x(0) = 0.
1.5.16. . v. ,
. ,
. L.

1.5.17. , , H .
, v.

1.5.18 . , , . . .
.
1.5.19. R
. u. ,
h ?
1.5.20. t
v.
( ) R, ( ) r.
?

24

2.1.
2.1.1. , ,
, . ?
2.1.2. , , t = 5
l = 20 . m = 100 .
.
2.1.3. -
v l,
. , , , y
? L . me .
2.1.4. . T1 T2 , T3
T4 . ?

2.1.5. l x , F ?
2.1.6. m1 m2 , T . F1 = t F2 = 2t,
, t . ,
.
25

2.1.7. m0 , .
()
t0 ,
t > t0 . ?
2.1.8. , . ,
F1 F2 F .

2.1.9. ,
. t F = nmg, m .
?
, t = 0, 04 , n = 125?
2.1.10.
. , ,
.
?
2.1.11. , . , .
2.1.12. . . ,
400 H. 12 , 72 . ?
2.1.13. , , . .
.

2.1.14. m1 m2 k.
m2 F ,
m1 . , ,
26

, .
F ?
2.1.15. m k1 k2 , . a. , .

2.1.16. m
k1 k2 .
F . , ?
2.1.17. , m0 .
, ? ,
m > m0 ?
2.1.18. ,
, . ,
,
. . 10 , 0,51.
2.1.19. , , , ,
, . max , - ,
. , .
2.1.20. m.
. , ,
. .
2.1.21. . , ? .
2.1.22. , v, v? ,
, tg > .
2.1.23. m, ,
F . . ,
.
27

2.1.24. , .

. . . .
2.1.25. ,
, . ,
m1 m2 . ,
F .
2.1.26 . , , . . ,
?
2.1.27 . m1 , , m2 . .
?
?

2.1.28.
, .
,
?
2.1.29. , , 5
72 /. .
, ?
28

2.1.30 . m1 m2 ,
.
.
. , ?
, F0 ,
l?
. , F0
m1 ?
2.1.31. , . F , . m1 m2 .
.
2.1.32. ,
, m. . .
.

2.1.33.
?
2.1.34. , , : f = v 2 . 20 /.
, 70 , 0,4.
2.1.35 . , , m; , , F ; . , , : f = v 2 . ,
u.
2.1.36 . m
l
v = v0 l, v0 , . ,
, ?
2.1.37. , , :
f = A0 r2 v 2 , 0 1,3 /3 , A 1. , ,
? r = 1
.
29

2.1.38. , , : f = rv.
r = 0, 1 , , 1 /.
, ?
?
2.1.39 . , ,
. , ,
. .
- ? ,
.
2.1.40. u.
, v . ,
,
.
2.1.41. , ,
? .

2.1.42. ,
?
2.1.43 . R, , , , ,
, . F , .
F > F .
2.1.44 . , ,
u .
. , , tg < .

2.1.45 . ,
30

, .
v. .
2.1.46. , . T .
. .
2.1.47 . , . F
m.
, M , .
2.1.48. m1
m2 . .

2.1.49 . ,
, . m0 ,
.
.
, , ,
. ?
2.1.50.
. m, , , > . .

2.1.51 . . . . .
.

2.1.52. , , a1 a2 .
, m1 ?
2.1.53. ( m
, ) , .
,
. .
31

2.1.54. , , v. me .
, , ? ,
,
v? v?
2.1.55. m , l,
v ,
( ). .
?
2.1.56. M l ,
. .
, . m
x .
2.1.57. R, , .
, . ?
2.1.58. , l, . , ,
.
.

2.1.59. m, k , R .
?
2.1.60 . m k R0 . .
, .
2.1.61 . m R. T .
, ,
, .
2.1.62. ,
R v.
?
2.1.63. = t.
, r , , ?
32

2.1.64.
, R, ? ?


?
2.1.65 . . , ,
.
?
.
2.1.66 . R. , . . , , .

2.1.67 . ,
?
1, R.
2.2. .
2.2.1. m v, 2m 2v ,
. .
3

33

2v
, . .
2.2.2. ,
, F . t .
,
3t ?
2.2.3. , ,
p .
F
?
2.2.4 . -
,
D1 , L .
F . , D2 ,
. , . ,
t. ?
,
?
2.2.5 . M , . v
m . , , , ?
?
2.2.6. . . , ,
. , , . , , ,
, . ?
2.2.7.
-, (. .
). v1 u1 - . , , v2
u2 . -,
. ,
?
2.2.8. , , . ,
: , ,
. ,
?

34

2.2.9 . m1 v m2 . ,
, ,
F0 t0 ,
t0 . , ,
.
2.2.10. v.
1, 01 v,
0, 04 v. , m0 ?
2.2.11. v . ?
.
2.2.12. L
.
. ?
2.2.13. m ,
. , 5m.
?
2.2.14. m1 , v,
m2 u . m2 ?
2.2.15. - , . p1 p2 ,
. .

2.2.16. m1 , m2 , m3 ,
v1 , v2 v3 .
?

35

2.2.17. m1 m2 . , l. ?
2.2.18. m 2m, , A B.
.
2m, , .
2.2.19. R m2 . m1 .
. .
2.2.20. : ,
? ?
?

2.2.21. , . S, l.
. . , v.
m.

2.2.22 . , 0 ; V0 . V u .
? ,
.
2.2.23.
( 1 : 2) R
.
,
.
36

2.2.24 . m1 m2 l
.
, ,
, v,
. .
2.2.25 .
m1 m2 , L. ,
. ,
T1 , T2 ? ?
2.2.26 . m1 , m2 , m3 l ,
. .
2.2.27. , , a
, V .
. m.
2.2.28 . S, . ,
.
L. a. ?

2.2.29 . m
A. A 2m B.
. ,
u ? .
2.2.30. , , v.
m1 m2 . .
?
2.2.31. N m . , ,
?
, ?
2.2.32 . M , , N m . ,
, .
h. .
, ? N  1.
3

37

2.2.33 . R v m, . , . ,
? ,
, N ?
.

2.2.34. M
v1 v2 . m, ,

. .
N ?
,
.
2.2.35. S, u, .
,
?
. .
2.2.36. m .
,
u? , a?
2.2.37. - ,
v.
1 2 . u.
2.2.38. .
F = kv 2 .
u. , S, .
2.2.39. r 0 . , F , , . ,
, . .
, , 0 ?
.
2.2.40. . ,
2P0 . P0 . . . t,
T .
38

2.2.41. , . .
.
, m, l.
2.2.42. , , ,
v? m,
l.
2.2.43. ,
, . , . .
h.

2.2.44 . , , v . F , .
, , . ?
2.2.45. v k . (
) , n

v?
2.2.46 . , , v . , -
n .
2.2.47 . , ,
2 /. ,
104 . ?
2.3. . .

2.3.1. ,
v, ,

F .
,
l?
2.3.2. ,
m l ,
v1 v2 . ?
39

2.3.3.
125
, .
, , h = 1 ,
m = 2 ?
2.3.4. , .
2.3.5. , m ,
F0 l1 ,
l2 , ,
l3 . ?
2.3.6. , , L, .
?

2.3.7. 1 2 . ?
.
2.3.8. k . v m. ?
,
, (
x0 ).
2.3.9. k .
, ,
F .
2.3.10. ,
? ?
2.3.11. l, ,
m. , ?
, < ctg .
2.3.12. , .
, v,
< tg ?
2.3.13. m h . A.
, . ,
?
2.3.14. m
l. , ,
40

. , ?
.

2.3.15. ,
l m ?
2.3.16. , l, r
. ?
2.3.17. , R,
v m.
, F
.
? .

2.3.18 . h. , , , T . , ?
2l, m. .

2.3.19. ,
.
, .

?
2.3.20. ,
h, . v,
41

. .
.
2.3.21. m v F .
F ?
l. ?
2.3.22. . . ? ? ?
2.3.23. l m
90 .
, , , ?
T .

2.3.24. m ,
.
? .
2.3.25.

, , , ?
, R,
< tg .
.
2.3.26. R,
,
.
?
2.3.27. , R.
, ?
2.3.28 . m. R, (
) h.
, H? .
42

2.3.29 . m . , k .
?
2.3.30. ,
, .
.
m.

2.3.31. l . .
.
2.3.32 . ,
? v,
m, .
2.3.33 . , , .
. , K0 .

2.3.34 . .
,
. md = 2mp , mt = 3mp ,
mp .
,
,
,
,



5
0,47
0,58
0,63
30
1,66
2,10
2,40
10
0,78
0,98
1,10
40
2,00
2,55
2,89
20
1,26
1,60
1,82
50
2,32
2,97
3,37
43

2.3.35. m , , .
, : l = p.
2.3.36 . m , (.
). (l = p), x , m,
p.
2.3.37. ,
, . . , K?
2.3.38. q Q,
r , U = kqQ/r. . , ?
2.3.39. -
U 0 = kqQ/r
kqQ/R, R , . U 0 U = kqQ/r
?
2.3.40. U = V (l2 /r2 2l/r), r
, V l , . , E?
2.3.41. , k, m. ?
2.3.42. m, k,
. . .
.

2.3.43. , h . ,
, h/3. ,
, , ?
?
44

2.3.44. 2l . m,
. , , x0 . ?
2.3.45. m h
k l.
. , .
2.3.46. m1 ,
m2 , ,
?

2.3.47 . m, k,
, .
a. ?
?
2.3.48 . m1 m2 , . , , ,
.

2.3.49. m0 .
, .
. , , ,
m0 , ?
2.4. . .
2.4.1. m , ,
v. , mv 2 /2,
. ,
v, 4 (mv 2 /2).
?
?
45

2.4.2. m1 2l m2 .
v. . , F ?
2.4.3 . k .
, x1 x2 .
?
2.4.4 . u. , v.
. ,
.
?

2.4.5. , F , 2r .
,
. , r? ?
2.4.6 .
, ?
2.4.7. m k. v,
.
?
2.4.8. , l ,
, ,
v. ,
l?
2.4.9 . m .
, 2l
. 2m,
.
.

46

2.4.10 . , , m.
.
2.4.11 . M m,
. H , h < H .
, .
. .

2.4.12 . R .
- , -
, , .
.
2.4.13 . l m , .
.
, l/3?
2.4.14 . 1 1 . ,
.
2.4.15 . . . , K, 21%.
,
?
?

2.4.16. m ,
k, .
, l.
47

. . x ,
? ?
2.4.17. .
M V 2 /2, M , V .
?
. ,
, .
2.4.18. , , .
. , v, ,
, l.

2.4.19. , m, , . , h, T < mg?



? .
2.4.20 . m L
M > m. . ,
?
2.4.21. , k l0 .
F , . .

2.4.22 . , ,
m , 48

k. ,
, .
2.4.23.
( ). .
2.4.24. m1 m2
k. F . .
?
2.4.25.
( ). ? ,
, .
2.4.26. 2l,
F . ,
, .
2.4.27. m F . t , , l. t
?

2.4.28 . m1 m2
k. m1 F . - .
? ?
m2 F
l, ?
2.4.29. . ,
m, u. , .
?
2.4.30. A. .
,
.
2.4.31. m1 m2 W1 W2 V1 V2 . ,
4

49

?

?
2.4.32. , . , .
2.4.33. m ,
v ,
h. ) ?
2.4.34. m1 m2 (m1 > m2 ) ,
. m1
h . .
? .
2.4.35. R ,
, . ,
- ? l,
m.

2.4.36.
36 /, 500 . 10 , 5 .
2.4.37. , , , , , 4
, . ,
75 , 10 ?
2.4.38. , , .
, ?
2.4.39 . m .
. . . N .
2.4.40 . m = 0 (1 m/m0 ), 0
, m0 , .
? ,
?
2.4.41 . ,
, . n1 ; ,
)

50

, n2 .
?
.
2.4.42 . N = (AB), N > 0.
70 /. 45 .
30 ?
, ?
2.4.43 . S v
, .
.
R. ,
,
.

2.4.44. v. u.
. ?
2.4.45. m, ,
. , u?
2.5.
2.5.1. , . ?
?
2.5.2 . .
.
2.5.3. u , k
.
u.
k.
51

2.5.4. , .
, ?
2.5.5. m1
m2 . ,
m2 ?
.
2.5.6. v1 v2 .
u1 . .
2.5.7.
v. ,
v1 v2 ?
.
2.5.8. m1 m2
v1 v2 . 1987, 1988
? , .
2.5.9 . m1 , m2 , m3
, m1  m2 m3  m2 .
, ,
v. .

2.5.10. m1 m2 . .
, ,
?
2.5.11. ,
, . . ,
R?

2.5.12. , , ,

. .
52

2.5.13. , . . v .
, ?
.
2.5.14. F
t.
. .
. .
2.5.15. R , v.
.
, v
, .
2.5.16. ,
. .

2.5.17 . m1 m2 . ?
2.5.18 . m1 v m2 , .
m2 .
2.5.19. m1 .
, p0 , p, . .
2.5.20.
- . u0
, v ,
,
90) . ?
?
2.5.21 .
h m1 . . m2 ,
, ?
)

, , .
53

2.5.22. m2 v , .
, .
2.5.23 . m1 R
. m2 .
, . ? .
2.5.24. m1 m2 k. ,
m1 x. ,
?
2.5.25. m1 m2 .
, . U .
, , . ,
.

2.5.26. 2m m. 45 .
?
2.5.27. 250 6 Li. 7 Li.
.
2.5.28. m , , E.
me ?
2.5.29. ,
, 13,6 .
, ?
mp = 1836me , me .
2.5.30. m1
m2 . , E.
2.5.31. m1 m2 p1 p2 , . .
54

2.5.32. K . ,
, 1 2 . ?

2.5.33. (2 H+ 2 H 3 H+ 1 H) ,
1,8 . , .
, ,
3,5 . .
2.5.34 . m p
. ,
E?
2.5.35. m1 m2
,
. . .
, ,
, .
2.5.36. m1 , v m2
. ?
2.5.37. m, v, m . ,
Q2 . .
2.5.38 . m .
F . n- .
n, ,
l. .

2.5.39. . . ?
55

2.6. .
2.6.1.
?
2.6.2. , , , . ?
, , ? ( ,
.)
2.6.3. , , .
81% ?
2.6.4. , , 385 000 , 27,3 .
, .
6370 , 9,8 /2 .
2.6.5. . . ,
.
, .
?
2.6.6.
(
). (), m,
. M , m. ,

. ( 10 40 1600
.)
. M m.
. ,
,
. , .
,
( ): m
M 2r = 10 , M = 7,0 ,
a = 2,8 107 /2 .
2.6.7. . , g = 9,8 /2 , R = 6370 .
56

2.6.8. . 1,5 108 ,


3,14 107 .
2.6.9. ,
, , .
2.6.10.
9400 7 39 .
?
2.6.11. 81 , 1700 .
,
?
2.6.12. , , m .
2.6.13. , ,
?
2.6.14 . m1 m2
R.
?
2.6.15. , , .
, r, T .
2.6.16. m L.
?
2.6.17. ,
.
2.6.18. m0 R
m. ,
, ?
2.6.19. R
v,
. , m?
2.6.20. 1000
30 /. ,
?
2.6.21. . , ,
?
. , 60 . (
, ,
.)
2.6.22. m U = M m/r, M , r
. U
h .
mgh U ? g, R.
2.6.23 . , . ,
, ?
57

2.6.24. v0 . , v.

? ( .)
2.6.25. ,
0,5%.
?
2.6.26. . .
2.6.27. v .
,
?
2.6.28. .
.
, (5g), ? .
1700 , 6 ,
.
2.6.29. : v = Hr; . ? (

.)
2.6.30 . m
R,
? .
2.6.31. , .
?
2.6.32 . ,
, . , ,
30 /.
2.6.33 . , -
. ?
2.6.34. K.
?
2.6.35 .
. ,
, !
.
2.6.36 . -, , . ( , ,
58

.) t -,
, r,
v, - ?
2.6.37. 1
500 , 40 000
.
?

2.6.38 . R M v
.
, ?

2.6.39 . v
, r. ?
?
2.6.40 . m M
r ( ) r ( ). ,
?
2.6.41 . , u
,
.
v ( 2 u > v > u).
; .
.
2.6.42 . M d.
,
v ,
r .
2.6.43 . , R
u, ,
V . , ,
- V . , ?
59

2.6.44 . V
? . ,
.

2.6.45 . , , . . ,
, .
. 2.6.43,
. a M .
2.6.46 . M ) a b.
, -, ,
. .
2.6.47. 35,4 ,
0,6.
1986 ;
?
2.6.48 . , R ,
. .
R, g.
2.6.49 . , .
2.6.50 . . , 30 . ,
.
2.6.51. , m, R1 R2
, . M .

60

a b a
k = a/b . S = a2 /k = ab.

2.6.52 . m r M .
,
R.
?

? R0  r. .
2.6.53 . . , , .
,
, , . = 4 103 /3 .
2.7.
2.7.1. , .

?
2.7.2. R m, .
?
2.7.3. m R
. - .
, t;
N .
2.7.4. R .
, ? ?
2.7.5 . , , : K = J 2 /2. J .
, m1 m2 ,
r1 r2 .
2.7.6. R ,
.
. ?
2.7.7. 2.7.6 ,
. J = mR2 /2, m .
2.7.8. , , M . , M ,
M/J, , J .
61

2.7.9 . R J, m1 m2 , ,
, .
2.7.10. ,
,
l. .
,
w, J.
m.
2.7.11. m1 .
m2 R,
. , F .
.

2.7.12. ,
. , ?
2.7.13 .
.
. , m.
.
2.7.14 . ,
,
. ,
l? , .

62

2.7.15. m1 .
, m2 .
, ?
2.7.16. ,
. r,
. , , m.
. t h.
.
2.7.17. m1 m2 .
. J,
R1 R2 .
2.7.18. r .
J, m.
, . . .
.

2.7.19 . ,
, . ,
?
2.7.20 . . .
?

2.7.21 . R m 0 . , ?
.
?
?
2.7.22 . R m
v0 .
,
? ?
63

2.7.23 . v .
,
?
.
2.7.24. 2.4.18. R ,
? v.
2.7.25. ,
. .
?
2.7.26. . .
?
, ?

2.7.27 . m1
m2 .
, . , ?
2.7.28 . l m
, a. .
?
2.7.29. m1 m2 ,
l, ,
, . ,
.

2.7.30. J1 J2 1 2 .
. -
. ?
?
64

2.7.31 . R v 30 ,
. ?
2.7.32 . . v1 v2 , . 1 2 .
,
.

2.7.33. m1 R, , m2 , h
v. m2  m1 ,
.

2.7.34. m1 R m2 . , .
, r
v ? .
2.7.35. , R J,
m. , ?
? .
2.7.36. , , ,
R .
. r = 0,1R
. ,
? .
2.7.37.
.
?
2.7.38 . 40 103 .

. , , , 40 .
5

65

2.7.39 . . ,
. ,
.
. .
?
2.7.40 . l, , .
.
2.7.41. m l . m v, , .
?
2.7.42. m1 l .
m2 , v.
. .
2.7.43 .
, ? .
2.7.44. m O J. R . ,
F ,
x,
. x , , ?

2.7.45. v1
v2 , . l.
?
2.7.46 .
m1 l, H
? m2 .

66

2.7.47 . , ,
. , , . , ,
u R.
.

2.7.48 . , , , . .
?
2.8.
2.8.1. , 10 . , .
, , .

2.8.2. 0,01
. , 1,2
. ?
2.8.3. ,
, 107 ,
1,5 103 /3 .
2.8.4. ,
. . . .
67

2.8.5. l
R.
, m1 , .
.
2.8.6. , , .
m1 = 0,1 m2 = 0,3 . .

2.8.7. ( A),
, F ? k.
2.8.8.
, l1 .
, l2 . .
2.8.9. m ,
.
.
2.8.10. m (A)
(B). , A ,
. , B,
. , .
2.8.11. m
, , .
A
B?

68

2.8.12. , ,
,
, .
? .
. .
2.8.13. R. ,
, . , d0 .
. .
2.8.14 .
, . ,
. 1
2 .

2.8.15. ,
,
.
?
2.8.16 . .
, F ,
f . ,
n ? .
2.8.17 . , ,
F0 .
, , ?

2.8.18. ,
10 . 4 . . .

2.8.19. 0,01 ,

69

. . ,
?
2.8.20. .
, m1 .
,
m2 . ?

2.8.21. h , m0 . ,
2L, .
, ,
?
2.8.22. , r,
. m .
M , 2L.
, ?
2.8.23.
.
?
2.8.24. m L h. ,
. . .

2.8.25 . , , m, . ,
n- .
2.8.26.
m, ? . .
2.8.27. ,
, ? AB
.
70

2.8.28. l.
, , , ?
2.8.29. ,
.
, ?
2.8.30. , ,
.
?

2.8.31 . > 30
, .
?
2.8.32. ,
, .
,
?
2.8.33 . m. ,
,
?

2.8.34. . ,
m1 m2 ?
2.8.35 . h , l. ,
71

, .
F , , .
,
?

2.8.36. R
l. .
,
.
2.8.37 . R .
h , v. ,
.
2.8.38. l ,
l/3 . A . D.
?

2.8.39. l m l/3
. ,
, .
.
, . ,
.

72

2.8.40. , r .
R ,
. , ? M .
2.8.41. ,
. m.
2.8.42 . . , t2 ,
t1 ,
l. , ; .
N ? > tg .
.
2.8.43. .
, , ,
?

2.8.44 . m . ,
, ,
?

73

3.1.
3.1.1.
2l .
,

x  l, F ?
, , x ?
? ,
x. , x0 ? m.
3.1.2. m k. , , x ?
x.
3.1.3. . m, , ,
v0 , x0 . .
. m,
p , x v = v0 1 (x/x0 )2 .
, ,
x. , ?
3.1.4. ,
?
3.1.5. l, m. , ,
x , x  l. x ?
3.1.6. l, ,
x0 .
3.1.7. r, R.

.
74

3.1.8 . q,
l, ,
qE. ,
x0 , v0 .
3.1.9. m r ,
R.
x .
3.1.10 . q 2L, Q.
x .
,
x . , m,
v.

3.1.11. k, , , . , x ?
3.1.12. . m
,
, .
R . , ,
x ?
. , . ?
75

, ,
?
3.1.13. m 3.1.12
m . .
3.1.14 . h, l ,
L > l m .
. ,
, , , 2 .
, , 0 .
3.1.15. ,
, x0 , y0 . l.
. .

3.1.16. , , x0
l, m.
3.1.17.
R N N + ,
 N . .
3.2.
3.2.1. . m, , , A
. ?
, x?
T ?
T /2?
. ,
m F = kx . , k
m. A,
x0 ?
76

3.2.2. , , , , l. ?
3.2.3. .
,
, ?
3.2.4. ,
.
? k1 k2 , m .
3.2.5. , ) 1 .
3.2.6. l
. , . .

3.2.7. . m,
, T0 . ,
, T .
.

. ,
.
)

, , .
77

, T .
3.2.8.
0,1%. 8 103 /3 . , 5,6 103 /3 , 6400 .
3.2.9. ,
(8,9 )? (0,5 )?
3.2.10. m,
, 2l, ) .
.
3.2.11. , 3.1.10.
3.2.12.
, . , 6400 .
3.2.13. , .

, .
3.2.14. m , . L,
. .

3.2.15 . ,
400 , 40 . ,
.
?
5 .
3.2.16 . a , .
l, .
3.2.17 . . l R
? .
3.2.18. m, , O, k.
, 2 < k/m.
)

78

= 2v = 2/T , T ,
, .

3.2.19. ,
l M .
m x
, .
, , x.
3.2.20. ,
,
k? .
3.2.21.
m. ,
, ,
R, R < g/ 2 .

3.2.22. R , 2l.
: ) ; ) .

3.2.23. k m, ,
79

. ? .
3.2.24. R,
. ; .
3.2.25. m1 m2 k.
, ?
3.2.26. H2 HD
(D ).
3.2.27 . : ) ,
; ) , .
.
3.2.28. M
l m.
.
3.2.29. m , k, . ,
. : ) ;
) ?

3.2.30. , ,
, I0 .
T0 . . T .
? , , .
3.2.31 . . k,
m. ,
.
3.2.32 . m, , , , l
. M .
- .
.
3.2.33. - . , , l.
3.2.34 .
.
80

, ,
. H.

3.2.35. , , ,
H,
.
3.2.36 . R l, k
. r.
.
, l  R m.
3.2.37 . 7 7,5 . ? S = 500 2 .
.
3.3.
3.3.1. m, , . x = A cos t.
? , ,
? ?
3.3.2. 5 , 1 . ?
: ) ; )
.
3.3.3. , , 0,01
0,5 , 1 .
?
3.3.4. .
?
3.3.5. l,
l/2 .
3.3.6. 3.1.7.
6

81

3.3.7 . l
,
. . ? . ,
?
3.3.8 . ,
. x R. p
, .
m.

3.3.9. , ()
OO0 ,
OO00 v0 , , , F = kr, r
. OO00 , m?
3.3.10. R v0 . l ?

3.3.11. l.
, , . ,
.
3.3.12. , , A T0 .
,
T ? .
3.3.13. m H k h,
.
, mg < 2k(H h).
3.3.14. v l.
82

. ,
?
3.3.15 . R,
, . .
, a  R? A = 2R?
, R
?

3.3.16. k m. , ,
d.
.

? .
3.3.17. . , .
,
T , < T /2?
3.3.18. m, k, . . , : ) ; ) l.
3.3.19. m, v, M ,
k, . ,
.
3.3.20. v m, k.
. . ?
83

3.3.21 . m1 m2
. m1 v,
. , ?
3.3.22. , ,
F .

F ? T .
3.3.23 . t0 ,
, x0 , v0 . ,
x = x0 cos (t t0 ) + (v0 /) sin (t t0 ).
3.3.24 . m, ,
x = A0 cos t. t0 F .
. t0 ? ?
3.3.25 . , u,
m, k
.
-
. .
3.3.26 . 3.3.25 ,
.
?
u?
3.3.27 . M , k . l
. n , .
, ?
3.3.28. AB M BC m. A
. BC, , AB
.
3.3.29. m x = A cos (t + ).
, , .
? ?
3.3.30.
A. .
84

,
?
?
3.3.31. ,
, , . m
, . ,
. , , ,
, F .

3.3.32. .
, A.
?
3.3.33 . (. 3.3.32),
: , ,
?
.
3.3.34 . = 107 1 .
, . = 1. ,
.
, A = 106 .

85

3.3.35. , ,
, , ,
. .
3.3.36 . . ,
? , tg  , , , v0
?
3.4.
3.4.1. , m.
,
k1 6= k2 ? ?
3.4.2. 3.4.1 k1 = k2 =
k/2. , .
,
? ,
.
.
3.4.3. . . A, . v, .
? ?
. , v , x .
3.4.4.
x = A cos (t ),

y = A cos (t + ).

.
.
3.4.5 . 3.4.4 , ; .
2/ .
,
, . .
3.4.6.
, , x-
, , , y-. ?
3.4.7.
x = A cos [( /2)t],
86

y = A cos[( + /2)t],

 , , 2/.
?
3.4.8. x- y- ,
, . x y ?

3.4.9. , x, y, ,
. ,
, .
?
3.4.10. ,
x A, y B, 2A x 2B y. p = 3 ,
q = 4 . ?
3.4.11 . m1 m2 , , , M .
. , F .
.
?
3.4.12 . k :
x1 = A1 cos (t + 1 ),

x2 = A2 cos (t + 2 );

.
?
. 2 1
? ?
3.4.13 . (. 3.4.12)
:
x1 = A cos 1 t, x2 = A cos 2 t.
?
. ?
.
3.4.14. F = F0 cos t
x = A cos (t ). ?
87

3.4.15. . m, k, , . ,
. .
. ()
. 3.4.15 v, : , v/2 v/2, ,
v/2 v/2. , , .
? ? ?
. 3.4.15 ,
3v, v.

3.4.16 .
v . , . M ,
m, k.
3.4.17 . 1 2 .
, , l.
v.
,
.
3.4.18. , ,

x1 = B cos (0 t + ) + A cos t,
x2 = B cos (0 t + ) A cos t.
, . , m.

3.4.19.
, .
.
88

3.5.
3.5.1. m ,
p0 . ,
, , , T0 T0 /2 (T0
).
3.5.2. m , A
, . x
, y, m, p.
, p, x,
. 3.5.1.
3.5.3 . 3.5.1 v0
x0 . n ,
? .
3.5.4. . . ,
?
3.5.5. . , 0,1 .
3,6 /, , .
?
3.5.6.
, ,
. , .
, ,
.
3.5.7. , , , . . ,
. .
3.5.8. , , ,
. ?
3.5.9.
:
f = bv. ?
3.5.10 . K = mv 2 /2, U = kx2 /2. , N = bv 2
89

f = bv,
.
3.5.11.
,
, ,
.
3.5.12 . , , , . , , f1 = b1 v1 , f2 = b2 v2 ,
k1 /m1 = k2 /m2 = 02 b1 /m1 = b2 /m2 = 2
(0 , , k1 , k2
m1 , m2 ).
3.5.13. ,
x1 = x1 (t) v1 = v1 (t), x2 (0) = nx1 (0), v2 (0) = nv1 (0) x2 = nx1 (t),
v2 = nv1 (t).
3.5.14. ,
. ,
0 (. 3.5.12).
3.5.15. , ,
v, T v/n, n > 1.
2T , 3T ?
3.5.16. . ?
?
3.5.17. , , , , .
.
3.5.18. t = 0 , , v0 ,
t 6= 0
v0
x=
exp (t) sin t,

p
p
=
02 2 , < 0 =
k/m,
k, m ,
. , ,
3.5.12 3.5.15,
.
3.5.19. x t ,
, .  0 , 0 ?
3.5.20. .
1%. ?
90

. ,
. ?
3.5.21 . .
, 1 .
0
(  0 ). Q ,
e ?
. Q = 108 109 .
0 = 104 1 . ,
.
3.5.22 . , ,
p. ? . : 2/  1 2/  1.
3.5.23. ,
, .
3.5.24. m F = F0 sin t,
. , , ,
.
3.5.25 . , , . , ,
, .

3.5.26. . m, ,
(. 3.5.25 ).
, , k,
F0 sin t.
. m, k,
, (.
3.5.25 ). , ,
m F0 cos t.
3.5.27 .
p , 0 =
g/l.

, p
= g/l + 2k/m.
:
x1 = B cos (0 t + )A cos t,

x2 = B cos (0 t + ) A cos t.
91

, F0 cos t, , A F0 ,
m, 0 . B cos (0 t + )
. B ?
3.5.28 . 3.5.27 :
.
?
3.5.29 .
?
3.5.30. , , , , , ?
3.5.31. F (t)/k,
F (t) , k .
F (t)/m, m . .
3.5.32 . t = 0
F = F0 cos t. m, 0 . | 0 |  .

cos cos 2 sin

+
sin
.
2
2

3.5.33 . , 3.5.32 , . 0 ,
, , .
, , 2/| 0 |. sin (  1)
.
3.5.34 . = 0 ,
x(t) 0 (. 3.5.32 ).
,
?
3.5.35 . :  0 . | 0 |  = 0 ?
? ?
3.5.36. . , m
0 x = A cos (0 t )?
. F0 , = 0 . .
F0 ?
3.5.37. x = x0 sin t, ,
, F = F0 cos t. ? m.
3.5.38.
92

,
. , .
3.5.39.
Q = 109
0 = 104 1 . ( ) ,
?
3.5.40. . 0 .
?
.
3.5.41 . m t = 0
. F = F0 sin t. t
. ? ? ,
t = /, /2.
3.5.42 . t = 0 m x Fx = F0 sin t, y
Fy = F0 cos t. , . ? ,
? ?
3.6. .
3.6.1. , k,
F . .
N - .
3.6.2. 1 0,1 .
,
1010 ?
3.6.3. E
, .
L S ? . ,
, L?
3.6.4. E a.
3.6.5. 0,5 2 75 25 2 .
. () . 2 1011 . ?
3.6.6. . 30 C 30 C
, 10 C? 1, 25 105 K1 .
93

3.6.7.
, , , . 1 , 2 , 2 2 .
, 500 C?
3.6.8. - 30 .
?
2,7 103 /3 , 1011 .
3.6.9. m, l S a. E. . ?
3.6.10. . , E. ,
.
3.6.11 . ,
, a a? E, l  a.
3.6.12 . , ,
, . , . k, k0 .

.
3.6.13 .
0
= l/l. = 0 / . , 3.6.12.
3.6.14. = 0,3. ?
. ?
3.6.15. ,
. , E
.
3.6.16. 5105 1 .
, . 3
4 . , 10 .
, ?
?
3.6.17. .
. F .
?
3.6.18. F0 . ,
. .
?
94

3.6.19. , c, .
u?
3.6.20. . , .
,  1 c.
. , , F0 ?
F0 .

3.6.21. .
, ,
. , ,
c. ?
E.
. ( ),
, . E
.

3.6.22. . c . S, .

?
95

. , , .
?
, c .
3.6.23. 2 1011 , 7,8 103 /3 .
?
, . ?
3.6.24. , 3 105 ,
5
510 0,71 1 , 13,6103 , 1103 1,2 /3 .
.
3.6.25. ,
P P0 0 . .
3.6.26 .
0 . P ( > 1). ,
.
?
3.6.27 . , . . , h. h.

3.6.28 . m , l k = m02 , .
. A l. . (  0 ) l k, E
, . 0 .
96

3.7.
3.7.1. S b .
c. .
3.7.2. . c , .
?

. L,
l. ?
, ?
3.7.3. c.
, P (t).
t0 .
3.7.4. ,
c, u = u(x),
x .
x0
.
3.7.5. , 2 /,
. ,
, 1,4 /. 5 2 .
3.7.6.
F . , S,
E, .
?
0, 5 1, 5 .
3.7.7. 102 2
0,5 5 105 , 107 .
,
.
3.7.8. , 1 2 ,
Fk .
F . ?
3.7.9. F , ,
. v, . , ,
, .

p
? v F/?
7

97

3.7.10. 160200 /.
?
3.7.11. :
. . . ,
. , , , ,
,
. .
3.7.12. .
. ,
. ? ?
3.7.13.
v, c .
? ?
3.7.14. c1
.
,
c2 .

3.7.15. ,
. , , . ?
3.7.16.
, ,
.
. ,
c1 c2 (c1 < c2 ).
3.7.17. .
?
98

3.7.18. . .
,
, ,
. ?
. , ,
.
3.7.19.
. ,
. .
3.7.20 . , ,
, ?
.
3.7.21. 0 .
, v?

3.7.22. 0 . , v ? ?
c. .
3.8.
3.8.1. , E .
, ?

3.8.2. , 10 ,
. ?
5 106 , .
99

3.8.3.
,
. , , .
, 1/6,
1/2, 2/3 .
3.8.4. A?
, c.
? ?
3.8.5. . 3.8.3 , .
3.8.6. -
v = v0 sin t. ?
? ?
3.8.7. , . . .

3.8.8 . . ,
. , , , .
,
P = 5 104 = 4 106 .
c = 5 /, = 8 103 /3 .

3.8.9 . , . -
100

0 L. ,
< 0 ? 0  0 .
3.8.10. 1 . 100 /. ? ? ?
?
3.8.11. , l L > l v.
.

3.8.12 . . , ,
, , ,
, .
3.8.13 . l1 , v, l2 , l1 /c1 > l2 /c2 ,
c1 c2 .
.
.
3.8.14. ,
, ,
. ,
, ,
, ,
. 1 ,
E 1 2 , E 2 .
3.8.15.
.
.
3.8.16. , ,
. 1
c1 ,
, c 2 , c2 . 1 c1 /c = c/2 c2 = 4.
,
,
.
3.8.17. l , l. -

(-), 7

101

. , , 1 2 . c1 c2 .
- , .
3.8.18 . (. 3.8.17)
. , , ? .
, ?
3.8.19. .
. .
. ,
. ? 1 ,
5 /. .
3.8.20. ,
1 c1  0 c0 . ,
. ?
3.9. .
3.9.1. c = 330 /. = 50 .
3.9.2.
. l ,
B = 100 ?
3.9.3. , , l,
, l. , .
3.9.4. , 1 ( 1 /2 )
( 1012 /2 ).
3.9.5 . ?

3.9.6 . L L
 c/L, c . ,
, , v.
102

. ,  c/L?
?
3.9.7. R
() A , .
?
,
c? A  R.
3.9.8 . .
F = F0 cos t.
? S, ,
c.
. , l
, F = F0 cos t. ? l
? ?
, ?
3.9.9 . . F1 = F0 cos t,
F2 = F0 sin t. l ?
?
3.9.10. L . . ,
, .
3.9.11 . . L,
,
. ? , 0 , S.
. 0 S/F0
. ,
0 S/F0 . , ,
?
3.9.12.
1 . , ?
3.9.13.
?
3.9.14 . . A0
. L  A0 . ? ,
c.
3.9.15.
, L = 20
. .
c = 330 /.
103

3.9.16. , .
, , . ?
?

3.9.17. 7 ?
3.9.18. 300 . ,
? ?
3.9.19. ?
?
3.9.20. , ,
. ,
? ?
3.9.21. l = 60
= 0,1 /, = 100
, . . ?
3.9.22. ,
, M . . l.
, ?
3.9.23. ,
. . , ? ?
3.9.24 .
108 .
? 3 103 /3 , 330 /, 1,3 /3 .
104

3.9.25 . . .
: . . . , . . .
, , . , . . .
( . . .: , 1965. . 333).
, ,
, .
3.9.26 . , k m ,
.

105

4.1.
4.1.1. ? .
4.1.2. , . ,
, 2 105 . ,
?
4.1.3 . ,
, F . a. .
4.1.4. ,
. .
?

4.1.5. r , , 3P P .
?
4.1.6. ,
P . r R.
, .
106

4.1.7 . R
P . .
4.1.8 . , ?
4.1.9. .
. F , ?
.
4.1.10. , , ,
100 10 2 . 80 .
?
4.1.11. , 20 , .
1 . ,
? ?
? ,
. 105 .
4.1.12 . a, , h. ,
, P .

4.1.13 . , ,
a, .
h. . P ,
0 . , .
4.1.14. r
H. R, h.
a. 0 , .
, ?
107

4.1.15. ,
. .
10 , 5 , 20 .
.
4.1.16. , 10 , .
, , 1 0,5 . ?
105 .

4.1.17 . ,
,
. , - . , R,
.
4.1.18. , ,
.
4.1.19 . R, , , .
, l?
108

r. h.
, . .
4.1.20 . r R, . ?
.
4.1.21. . .
.
?
4.1.22.
, , ,
?
4.1.23 . R,
,
.
?

4.1.24.
,
.
4.2. .
4.2.1.
S m, P0 .
4.2.2. 1 2
(1 < < 2 ). H.
.
4.2.3. m 1 2 .
, h S,
1 .

109

4.2.4 . 0
. b, a.
a b ?
4.2.5. 0,5 , . ,
?
4.2.6. 9,8 . .
7,8 103 /3 .
4.2.7. , .
?
4.2.8. .
1 , 2 ,
. .
4.2.9 . d1 d2 . ,
m , ?
4.2.10. , , .
2 .
4.2.11. , r
? , l.
4.2.12. ,
10 3 , , .
.

4.2.13. ,
. , 45 . .
.
4.2.14. m . ,
. .
4.2.15 . , , H, , ?
R, L, .
4.2.16 . , , ,
, .
110

4.2.17. 10 90 5 .
, ?
4.2.18 . 4.2.17 ,
r R, .
4.2.19 . , , . .
. , a.
111

4.2.20 . , , . m.
.

4.2.21. . 1 1 0,2 . 0,8 103 /3 .


?
. R, , r h.
H, . ?
, 0 > .

4.2.22. R = 0,1
H = 10 ?
4.2.23. ,
2 35 ?
100 , 3 ,
1,02 /3 .
4.2.24 . R
. H,
.
112

. ?
.
l < H, .
. :
, ,
, ,
, . .
4.2.25 . R, ,
. r 2.
, .
4.2.26.
R, , .
l
r;
. .
4.2.27.
.
, m1 m2 .
R , ,
. .
4.3.
4.3.1. 5 . ( ) , ,
, 4
. ?
4.3.2. . h.
, , S? .
4.3.3. V h
S. ?
.
4.3.4. . ,
, A 1 2,
. , , ,
. 2
, 1?
. , A?
1 2 S1 S2 . .
2 v,
.
4.3.5.
.
? P0 .
8

113

4.3.6. S v .
, P ? .
4.3.7.
S s,
. , F .
.
4.3.8. .
l .
?
4.3.9. m
N , . S. v.
, , , ?
4.3.10 .
r
R, 0 ?
, h?
4.3.11 . ,
?

114

4.3.12 .
. H h?
4.3.13. h v . ?

4.3.14 . ,
2 , v .
, ,
.
4.3.15 . ,
R r,
.
4.3.16 . . . . 1941 .
. . . . 1944 .
. .
. , , ? , , . , ,
, : 8 /,
4 /, 8 / ( . .
- 1971 .). . , ,
, 30 .

115

4.3.17 . l L- .
. ,
. , .
4.3.18 . . S, s.
. .
4.3.19. , V .
P . , ?
4.3.20 . R. P . ,
r?
4.3.21 . ,
.
4.4.
4.4.1.
. v0 ,
.
, . h.
4.4.2.
. h,
. ,
( ) P .
4.4.3 . . , .
. h.
. 2 ,
1 /. 103 /2 .
4.4.4 . m
R , , ,
.
4.4.5.
R l. P ,
. : ) ; ) . ,
.
4.4.6. ,
, T . ?
4.4.7 . l d .
, .
4.4.8. r,
, R . , , M .
: ) :
) , .
116

4.4.9.
. 1 2 . 1 2 S1 S2 ,
P1 P2 . 1 v1 . ,
1 2.
4.5.
4.5.1. ?
.
4.5.2.
? , .
?
4.5.3. ,
. 0,073 /.
4.5.4. l.
1 2 .
?
4.5.5. ,
l k, ,
R, .

4.5.6. . ,
V ,  3 V ?
8

117

. , 1 , , , 290 /.
0,465 / 13,6 /3 .
4.5.7. , , ,
. .
.
4.5.8. h r 2h, .
.
. .
4.5.9. , ,
.
4.5.10 . l . . , , x? , . m.

4.5.11. ,
. . .
4.5.12. . , , . , ,
. .
.
. ,
?
4.5.13. . ,
. , , .
. , ,
.

118

4.5.14 . . .
, , .. .
.
. 1977 . -, 28 691 ,
; , .
. . 0,03 /,
0, 8 103 /3 , .
0,8 .
4.5.15 . , ,
( ),

.
4.5.16. . , R /R ( ).
, A.
. , R 2/R.

4.5.17.
,
.
h, R,
, .
4.5.18.
(. 4.5.12). x,
?
, .
4.5.19. R, h. .
P0 , .
4.5.20. , ?
0,1 2 1 2 . .
4.5.21 . , ,
, .
, , . .
4.5.22 . , .
119

1 , 2 . ,
r.
4.5.23. R0
r. , ?
?

4.5.24 . R.
, h, , , r? ,
, .
4.5.25. , ,
, (
). 1 , 80 , 0,465 /. .
?

4.5.26. , 3 ?
4.5.27 . , . 0,3 /,
2 .
4.6.
4.6.1. .
, . ?
. ,
: ?
120

4.6.2. R . , . , , .
?
4.6.3. 10 .
.
4.6.4 . . 4.4.5, ,
r, , (- ) h . , ,
. .
. ,
1 10 1 ,
, 105 , 5103 /2 ,
7 102 /.
4.6.5. , , , 5 ,
0,465 /, 13,6 /3 ?
4.6.6. ,
, 2 4 .
,
1 , .

4.6.7. r h
.
x ? x ?
, . .

121

4.6.8. h.
, h/2.
r, .
4.6.9 . , ,
OO0 . 2l, r. , . .
?
4.6.10. , l, h. ,
. , , .

4.6.11 . , , r = 0, 1 . .
, H = 15 ?

4.6.12. ?
4.6.13 .  1 ? ,
, H. .
4.6.14.
, , 1 , 2 ? , .
4.6.15. a, ,
90 , ? , .
122

4.6.16 . a, ,
? , , .
4.6.17 .
, S, l?
? .

123

5.1.
5.1.1. 10 , 5 C.
5.1.2. , , ?
5.1.3. , 1 / .
5.1.4.
, .
20 C. 1 , 10 .
5.1.5 . . , , ,
20 . 300 K.
,
,
M = , = 1,38 1015 .
, 100 K?
5.1.6. .
, , . .
, , . , ,
. .
5.1.7. . . 2 .
A. 2.
5.1.8 . n . 1 1
2 . .
, . 1 2 n-
.
124

5.1.9 .
. ,
( ) n ?
5.1.10. , ,
R L, ,
R. , .

5.1.11 . V . N (. . ,
).
, T1 ,
T2 > T1 ? . ?
5.2.
5.2.1. 1 3 0,1 2,7 1019 . ,
999 1001 /, 1,3 1012 .
. 1 ?
. ,
1000 0,1 1000 10 /, 1 3 ? , , , ,
.
5.2.2. vi
(i = x, y, z) . , 5.2.1 1 3 , ,
, 999 1001 /: , 10000,1 /,
1000 2 /.
5.2.3. dN ,
x (vx , vx + dvx ), N
T :
r
dN = N

 mv 2 
m
x
exp
dvx = N f (vx ) dvx ,
2kT
2kT

m , k .
r
f (vx ) =

 mv 2 
m
x
exp
2kT
2kT
125

.
(T = 293 K). , : )
1 3
, 499
501 /, ) 1 3 ,

498 502 /, 1 3 2 1019 .
5.2.4.

.
5.2.5. , x 3000 3010 /, , 1500 1505 /. 300 K.
5.2.6. , x 3000 3010 /, y 3000
3010 /, z 3000 3002 /, , x 1500 1505 /,
y 1500 1501 /, z 1500
1502 /. 300 K.
5.2.7 . .
?
5.2.8 . , , l, .
. 1
, 2 .
: 1, , , t0
2. , , .
t 2?
5.2.9. ,
R = 30
.
= 100 /. ,
= 0,314 .

5.2.10. . h.
126

, . . . , ?
5.2.11. ,
, f (v).
.
, ,
, l.
. k
, . .
5.2.12. , ,
v0 2v0 . . ?
,
F ? m.
5.2.13 . v v + v (v  v). n ,
m.
.
F .
.
.
, l F .
5.2.14 . .
p
f (vx ) = 2 / exp (vx2 ), > 0.
m. , l,
F ?
. 0 , T ,
m. .
h
.
5.2.15. 3 1 3
102 , 105 . , ,
1,5 /3 . 27 C.
5.2.16. 105 , 106 , .
? ? 0 C.
5.2.17 . , . . ,
. , ,
. , , ,
, .
. T , q,
127

n0 . ( ) .
. 100 C
4 104 /. 100 C.
5.3. .
5.3.1. 0 C 0,1 . .
5.3.2. . 0,18 .
5.3.3. , 1 1 3
.
5.3.4. ,
. ?
5.3.5. . n1 n2 . R1 R2 . .
5.3.6 . 1 3
2,7 1019 . , 0,1
. , . 0,06 .
5.3.7 . A2 , B2
AB,
A2 + B2 2AB

AB + AB A2 + B2 .

A B, A2 , B2 , AB rA2 , rB2 , rAB , .


5.3.8. . .
: n = h. m,
, T .
.
. 20 C.
0,21 . 0,18 .
5.3.9. A B1 B2 D1 D2 , n . A ,
n1 B1 n2 B2 .
5.3.10 . L
S ,
, .
0 C. .
,
. , , . D,
.
5.3.11. .
h, T = T0 + h. h/T0
. ,
128

m, n.
. ,
?
. ? 0,14 ,
0,18 . .
5.3.12. , 5 5 4 ,
1, 5 2, 0 , 0,2 , ,
1 C, +20 C,
20 C. ?
5.3.13 . A1 A2 1 2 .
, A1 ,
A2 . , .
1 2 .
5.3.14. t. ,
n ?
5.4. .
)
5.4.1. , 1 2
1 , , .
5.4.2. , k- , , ?
5.4.3. v r. n, m, . ,
.
5.4.4. ?
5.4.5. r
v, . , .
. P , T .
5.4.6. P .
s,
. ,
.
5.4.7. ,
. , ,
v. S,
P , T .
.
5.4.8. , ,
. , . P1
)

, (
),
, , .
129

1 . , , 2 , .
.
 2 ?
5.4.9. r1 r2
. . .
5.4.10 .
, . ,
. , , ,
. ,
. .

5.4.11 . 1 2 , 100 C, 0 C.
20 C, 0,1 .
5.4.12. ,
, 310 K,
300 K. 300 K.
5.4.13 . S ; T1 T2 ,
T1 . F .
.
5.4.14 . T0 .
, P , T .
, ? .

5.4.15 . , ,
130

P ,
T 2T . ,
. .
5.4.16 . , , ( ). ,
T T + T ,
n ; .
5.4.17.
,
.
. , , ?

5.4.18. , , ( 293 K), , 0,133 . 600 2 ,


0,2 /, 81 K.
.
5.4.19 . -
, n ,
W1 .
, , W2 ,
. . .
.
5.4.20. , . N .
: ) /N  ; ) < ,
, .
5.5.
5.5.1. ,
. ?
5.5.2. , .
P1 P2
131

, V1 V2 .
?
5.5.3.
n , m. ,
k ?
5.5.4 . r L  r,
,
m, h  r. P0 . ,
.
5.5.5. 50 27 C
10 . , 40 ?
3 C.
5.5.6.
L, , ,
L/2? P .
5.5.7. 2 27 C.
50 C ( ).
.
5.5.8. 0,1
7 C. , ? , 10 . 2 2 .
5.5.9. , ?
5.5.10 . 2L ( ) , . T0 , .
, ?
L.

5.5.11. V
P0 P , V0 ?
.
5.5.12. V
V0 P0 P ?
5.5.13.
?
5.5.14. . , , ,
. .
132

5.5.15 . 50 60 C.
, . 10 C, 1,29 /3 .
5.5.16. V0 , l S.
. .
, .
T0 . ,
.
5.5.17. 200 100 3 , . 300 K,
1013 , 273 K,
373 K. ?
5.5.18.
. .
3. ,
?

5.5.19. H l. .
. , . P0 , .
5.5.20. A
, B, . x A, B
H? B
. P0 , H0 ,
h0 . .

133

5.5.21. , .
. , ? h, P0 .
. ,
?
5.5.22. (, ), .
. , ?
. . 2a.

5.5.23 . ,
. P0 . ,
? H, .
5.5.24 . 3 ,
.
0,15 . , ? ?
5.5.25. , 1
1 17 C
0,314 .
5.5.26 . ,
100 0 C
, ,
, .
1 3 , 82 C,
180 .
5.5.27. ,
? .
29 /.
5.5.28. .
, . , m?
5.5.29.
, ,
50 /2 , 105 , 27 C?
5.5.30. V T , T0 . P0
? .
5.5.31. .
500 C,
134

100 . 1 ,
?
5.5.32. , : N2 O4  2NO2 . 250 3 0,92 N2 O4 0 C.
27 C, , 128 . , .
5.5.33. , ,
. P0 T0 .
, , r.
, .
.

5.5.34. r1 r2 .
,
r, P0 .
5.5.35. m,
S l .
P0 T0 . .

5.5.36. ,
P, V -. 12 34 ,
, 14 23 .
T, V -. V3 ,
V1 V2 = V4 .
135

5.6. .
5.6.1. = (i/2)kT , i
, ,
. , , H2 , N2 ,
H2 O, CH4 T .
5.6.2. ( )
1 3 ? 1 ?
5.6.3. T0 T .
. ?
5.6.4. V1 P1
T1 , V2 P2
T2 .
? .
5.6.5. 800 K 1
(CO2 ) 1 (H2 ).
CO2 + H2 = CO + H2 O + 40,1 /.
?
5.6.6. m1 m2 , V0
P0 . .
, .
5.6.7.
m 1 T0 .
3v v. ?
. .

5.6.8. , .
5.6.9. , ( ) .
5.6.10. : ,
. ?
5.6.11. ?
5.6.12. V P . . .
,
, : ) ; ) 2P ?
5.6.13. . 2 6 .
136

5.6.14. ,
,
,
1, 2, 3. (U = cT ). ,
.
5.6.15. , 2
0,8 ,
10 . , .
5.6.16. , 2
0,1 ,
4 . , ( ), . 8 . , .
.
5.6.17. , 0 C, . ,
? ?
5.6.18. , ,
, .
1 3 T1 T3 . , ,
, 2 4 .
5.6.19. M , V0
P0 T0 , u. . , .

5.6.20.
T1 .
. , ,
, T2 .
.
. .
137

5.6.21. 1 . , ,
.
5.6.22 . , M .
T P . S, , , H. . . ,
, ? ?
5.6.23 . . , V0
V1 . ,
, V2 .
V1 ?
V0 V2 , P0 .
V1 ?
5.6.24. 1 3 0 C ,
1 0,5 2 . 973 .
300 C? .
5.6.25. 1 1 K 912 ,
649 . ?
5.6.26 . , 1
T0 P0 , S. P0 . .
F . , ,
. U = cT .
, ? .
5.6.27. ABCD, ,
, . KBCM 1 , (BK = 2h). ,
. ,
h, AK > h? ,
. . ,
S.

138

5.6.28 . , P V n = const. n
? ?
5.6.29. ,
P V 2 = const?
5.6.30 . , ( P0 , V0 , T0 ). . . ,
, . , .
5.6.31 . , M .
P . S,
H . . . ,
? .
5.7.
5.7.1.
. .
, -
0 K. , ,
.
T , , cP .
5.7.2 . 1 2 .
k . T .
5.7.3 . ,
1 /. 131.
. , ,
?
. , ,
?
5.7.4. ,
:
) 2H2 + O2 = 2H2 O + 483 /;
) 2Al + (3/2)O2 = Al2 O3 + 1,65 /;
) Be + (1/2)O2 = BeO + 610 /.
5.7.5.
T = 3000 K, = 30 /. .
cP = 33,4 /( K). .
, M = 1000 .
139

5.7.6 . .
T1 , P1 . P2 .
. , .
5.7.7 . , 1 , ,
0,1 . 300 K.
.
5.7.8 . , 0 C,
400 /. . ?
5.7.9 . P T v
S. , ,
. q. . ?
, .
5.8.
5.8.1. . V 1 2.
. . 1
, 2. 1
, ?
. .
1?
. N . 1?
5.8.2. , -
, .
. . ,
? ?
. ,
?
. . ,
?
5.8.3. V0 N .
. , V ,
V0 , .
. ,
102 ?
5.8.4 . , 0,1 3
- ,
. ,
?
5.8.5. , , a a a, . v.
.
,  1?
? ?
140


.
. , ?
S,
, ?
. ,  1?
, V ?
5.8.6 . 5.8.5 , 1/m, m .
5.8.7. , v,
, .
. . R, H.
,  R/v, H/v
v.
5.8.8 . 1 2 A. , V , v, ,
1 .

5.8.9. .
. , , , . . , 2N , N .
, 5.8.1.
. ! , , ,
.
,
.
?
? , . ,
,
. , 1 T , Q, ,
exp (Q/kT ) 1 (k
).
141

, . , , N
( 1), , ( 2).
, , .
x A . ,
Q = A (
). ,
T Q = A.
A = RT ln (V /V ), ( ), R = kNA , NA , V , V
.
N = NA , V /V = 2. A = N kT ln 2. , 2
exp (A/kT ) = exp (N ln 2) = 2N 1. 5.8.1, .
,
, V , V0 ,
, . , 5.8.3.
5.8.1?
5.8.10. , 1 106 102 ?
5.8.11 . , ,
, T P = nkT , n
.
?
5.8.12 . 2V0 2N .
N V0 V N V0 ?
5.8.13 . 20 C 1 , 0 C . ,

.
.
5.8.14.
, T :
)
exp (U/kT ) , ,
, U ; )
exp (U/kT ) , , , ,
U , .
142

5.8.15.
,
. ,
, ?
, .
5.8.16. ,
. , , . ,
n
n ? n ? .
5.9.
5.9.1. , ,
. . ,
.
5.9.2. 1 .
5.9.3. 1 , 293 K, ?
5.9.4. V 2V : ) ; ) . m.
5.9.5. m
V1 T1 V2 T2 , : ) V1 , ; ) T1 V2 ,
; ) V2 , .
5.9.6 . 0,1 0 C , 2 50 C. .
1,75 /( K).
5.9.7 . 0,5 1 .
273 K, 303 K. . .
5.9.8. V , , .
, .
. , P ,
T . ,
(, )?

143

5.9.9 . , 1 , , .
.

5.9.10 . , , .

5.9.11. ,
?
5.9.12.
?
5.9.13. 14,7 1 8,1
3, 3 107 /. 200 C,
58 C.
.
5.9.14. , , .
5.9.15. ( )?
5.9.16. 1 0 C .
, ,
. 27 C.
5.9.17. , 1 3 ?
, ?
6 .
5.9.18 . T
.
C. ,
T0 . ,
.
144

5.9.19 . T1 T2 .
C1 C2 .
, . , . ,
1 , 1 0 C.
5.9.20. ,
?
5.9.21. . ,
A?
5.9.22. . , 0,1 , -
, . ,
, 104 K 20 C. ,
13 C. (
0,01
.)
5.9.23. , , 0 C
100 C. ,
1 ?
5.9.24 . 1 17 C 23 C.
?
5.9.25 . , 1 , 300 K?
5.9.26. , (,
) . , ,
. , ,
. ?
5.10.
5.10.1. ( 10 C)
. 10 . ?
5.10.2.
1 ?
1 /2 , 0,9.
5.10.3. , , ,
?
5.10.4.
1 . 10 . ,
- .
5.10.5. 0 C.
. ?
5.10.6. 100 10 C 40 ,
10 C.
10

145

, ? 2,5 /.
5.10.7.
.
,
200 C.

0, 1
20 C. 150 C. ? .
5.10.8 . ,
10 C. 2,2 /( K),
0,9 103 /3 .
5.10.9. . , ?
. ?
5.10.10. , , . . .
, ( . . . .: , 1949. . 1). .
5.10.11. ,
. ?
5.10.12.
(81 K)
?
, ?
5.10.13. . ,
.
5.10.14. 1 .
, . 1 , 1 2 . ,
.
5.10.15 . : (CCl4 ) . CCl4
76,7 C, 100 C.

65,5 C. , ( )
.
65,5 C 25,6 .
5.10.16. ?
5.10.17. ?
5.10.18.
. ?
5.10.19. .
.
, ?
5.10.20. 20 2
5 .
, . 15 /. 146

.
0 100 C. .
5.10.21. , , 20
, . ?
? 0,5 3 ,
20 C 1,73 .
101,3 .
5.10.22. 300 K
P0 . 77,3 K, .
, ,
2P0 . ,
. c = (5/2)R,
5,53 /.
5.10.23. , ,
. m, 0 C,
0 C P0 . , ?
? q, , .
5.10.24 . 100 2 , . ,
100 3 . 100 C. . ,
, 0 C
. 0 C
610 . .
42 /. ?
5.10.25. 4 C ,
?
5.10.26. 0 C 100 . ,
13,8
1 C? 2,5 /( ).
5.10.27. .
, h, ? m, T . r, .
. r1 = 1 r2 =
1,1 . ,
?
5.10.28. , , ,
. . 147

,
, = 60 %, t1 = 25 C.
P = 34 . = 2,5 106 /. P = 105 .
5.10.29. P . , ,
?
5.10.30. R r. .
P0 . .
5.10.31 . (. 5.10.30) L
,
. ?
5.10.32. . n , m ?
. 300 600 K
141 . 300 K P0 . 600 K.
5.10.33.
,
100 C. 50 , 1 2 .
5.10.34.
1 2
0 C
100 C.
5.10.35 . 2
1 , ,
1 . 2,6 /3 ,

1 . ,
, , .
5.11.
5.11.1. , T , ( ), : = T 4 , < 1
, = 5,672108 /(2 K4 ) ,
T .
. , ( ).
0,3.
148

. 6300 K.
, .
5.11.2. 0,5
150 . , .
5.11.3 .
T . 3 108 /.
5.11.4. , , ,
.
5.11.5. . , 800 C - ,
( . . .
.: , 1976). .
. ?
5.11.6 . .
, T0 .
. , , T1 T2 .
5.11.7. . R T0 . . ,
L .
. , ,
20 C.
5.11.8. , ,
1,5 108 ; 5,8 107 ; 1,1 108 2,3 108 .
20 C.
. , .
. .
. ,
. ?
5.11.9. 2 106 /2 .
, , .
5.11.10. , , ? .
5.11.11 . ( ),
, T1 T2 ,
1 2 . S,
.
5.11.12. T .
. ?
. ,
T /2?
5.11.13 . r R, 2R 3R. T0 . .
, = 1.
10

149

5.11.14 . , h.
, . T1 , T2 . .
5.11.15. E p
p = E/c, c , . , P w P = w/3.
5.11.16. . . . , , . . .
- . . . ,
( . // . 1967. . 13.
. 27).
.
R , , S,
m?
. ,
R1 R2 ? .

5.11.17. . , .
, , .
. ,
.

150

6.1. .
)
6.1.1. . 1 2 1
.
. 108 ?
?
6.1.2. 1 1 . .
6.1.3. . , 1 , 1 . , , ?
. , 10 , 100 . , , ?
6.1.4. , 10 , 1 20 ?
0,001 ? 1000 ?
6.1.5. , 1 3
, 100 .
?
6.1.6. 1 3 , 1 %
?
6.1.7. q1 , q2 , q3 .
l. .
6.1.8. l q1 q2 .
q, . ?
?
)

,
.
151

6.1.9. m,
l, , .
.
6.1.10. q, Q, q, Q
, . l.
, Q > q.
6.1.11. Q, q, Q, q
, . l.
.

6.1.12. 1016 /. .
6.1.13. q ,
l, m q . q .
.
6.1.14 . q R, m
Q ?

6.1.15. q,
, , 2a
. 2l, l. .
6.1.16 . q
, .
l. .
152

6.1.17. R?
h ? Q.

6.1.18 . l , , x ? .
6.1.19. , ,
, E = /(40 ), ,
, .
, , :
) ,
, ;
) , 1 , 2 ;
) , ;
) ,
1 , 2 , 3 ;
) h, ;
) , ; l.

6.1.20. . n , 153

. , .

, .
. -
, ?
6.1.21. . ,
Q. , ,
. ?
. , ,
?
6.2. .

6.2.1. . E.
l,
30
?
.
, , ,
. , E , h. , h, E,
E.
. ,
.

6.2.2.
, R
r? E .

154

6.2.3. ,
Q ,
, Q/(40 ).
6.2.4. , , .
, , ?
6.2.5. . q ?
? ? .
. q,
? .
6.2.6. ,
:
) , Q;
) , ;
) , ;
) R,
; ;
) R,
;
;
) h,
;
.
6.2.7. :
) R ( E0
); ) R ( E0
).
6.2.8 . ?
? , l.
6.2.9. ?
? ?
6.2.10. .
1 2, ?

155

6.2.11. h . , . .
6.2.12. , . h, . A? B?
OA O.
6.2.13 . R
r, l . .
, . ,
.
6.2.14 . . R, l , , .
, . ,
. .
. 6.2.14 : l 0, , l = const, R,
E.
?

6.2.15 . ,
.
6.3. .

6.3.1. . 300 . ,
?
. 108 /.
10 . . ?
6.3.2. . 1 2, , ,
, 1 109 /, 2
2 109 /. 2, 1
?
156

.
220 . ?
6.3.3. 0,1 0,2 20 .
, ?
6.3.4. l q.
?
6.3.5. 109 10 .
(1)
(2).
6.3.6. 100, 10, 1, 10, 1, 10
2 .

?
6.3.7. R Q.
? ?

?
6.3.8. ?
? , ?
6.3.9. , ,
E E = /0 .
6.3.10. . ,
6.1.20 .
. n . R,
Q.
.
. .
b, .
6.3.11. ,
1 , 2 . .
.

6.3.12. . 1 ,
. 3 /2 . .
. 5 , .
157

1010 /2 .
.
6.3.13.
, , 1 , 2 , 3 ?
h1 h2
.
6.3.14. , . a b. .
6.3.15. .

. a, b (a b
). . , .
. a b
q.
. , , ,
, q , , q. .
6.3.16. R Q. , .
? , ?
L
, q. ? ?
6.3.17 . , r, q. R. L  R. ?

6.3.18. ,
. ,
n ?
6.3.19. 10 , 30 10 , ,
. 105 , 8 105 .
.
158

6.3.20. r, 2r 3r
q, 2q 3q. .
6.3.21. r ( ). 2r . .
.
6.3.22. R1 , , R2 .
, ? ?
6.3.23. R1 R2 . q,
.
.

6.3.24. R1 R2 . q, .
.
6.3.25. R
. .
6.3.26.
R, ? R, ?
h, , ?
6.3.27. r R.
.
6.3.28. Q h . ?
6.3.29. h Q. , , 2h.
6.3.30. , , . I q
l . IIIV? q?
6.3.31 . q L R < L. .
?
159

6.3.32 . , q, L
? R < L.

6.3.33. R q, L
, Q?
6.3.34.
. , .
, , , . . ,
.
? , ,
, .
6.3.35. 1 2 R , q.
, ?
r  R.

6.3.36. 1
Q. , 2, .
2 q. , ,
, 2.
6.3.37. , Q, ,
Q?
160

6.3.38. , 100 /. .
6.3.39. ,
?
6.3.40. .
6.4.
6.4.1. ?
?
?
6.4.2. . .
?
. ,
? n ?
6.4.3. . ,
S d.
. 20 2 , 3 . .
6.4.4. S, d.
. , d/3 S?
. , d/3, S 0 < S?
. ,
?
6.4.5. , R1 R2 ( ).
6.4.6. , d < R1 R2 .
R0 .
6.4.7 . , R1 R2 . l  R1 , R2 .
6.4.8 . a l.
d. , R  d.
6.4.9. , .
6.4.10. E, .
S. ,
?
11

161

6.4.11. . ,
, V1 V2 . , a.
d.

6.4.12 . . ,
? d.

162

. ,
?
6.4.13. d.
, . , , q.
, ,
x?
6.4.14 . (1) (2) S .
(3), (4). ,
. (5), C.
. .
n ?
d .
6.4.15. ,
, ,
1000 , . 100 2 ,
1 . ,
?
6.4.16. , k ?
n ?
6.4.17. ,
V :
) S = 1 2 ,
d = 1 V = 1 ;
) r1 r2 ;
) l r1 r2 .
6.4.18. Q. S, d.
. ,
d?
. ,
x , ?

a a.
. , ? ?
6.5. .

6.5.1. . ?
. S, . ,
( )?
.
, E. 163

.
?
. 104 /,
. .
6.5.2. S, , , P0 . ,
? ,
.
6.5.3.
E 2E? E 2E?
.
?
6.5.4. h.
h, .
, . , h = const?

6.5.5. , R, Q.
6.5.6. , V .
R, r.
6.5.7. R,
P ?
6.5.8 . . , , q.
? (1)? (2)? .
. R .

6.5.9 . R
, h . ,
164

. Q. , ?
6.5.10. S q. , x ,
,
E, 0 E 2 /2.
6.5.11. E S. ,
d. , ?
6.5.12. E
.
.
. ? ?
. h, S.
, ?
6.5.13. , ,
? ,
S, h
.

6.5.14. E0 . E. ,
? S, h .
6.5.15. R
. Q.
6.5.16. W W = mc2 . , .
, . , , .
6.5.17. , 1016 .
6.5.16, ,
1016 .
6.5.18 .
R. Q.
6.5.19. ,
? R, Q.
11

165

6.5.20.
, R Q N
?
6.5.21. , n .
?
6.5.22.
, , A. ,
? n ?
6.5.23 . .
, A. ,
?
6.5.24 . , ,
. A .
,
?
6.5.25.
Q, ,
l?
6.5.26 . 1 2 . ,
, Q1
Q2 .
6.5.27 . a a
d , ,
c a a. , , , : ) Q; )
V .
6.5.28. , ,

? Q, a b. S.

6.5.29 . S d
r q r. , .
, r
.

166

6.5.30 . , q, R, r.
(  r  R).
6.6.
6.6.1. .
?
. ?
. ?
6.6.2. 0 C 1 1,000074.
300 /.
6.6.3. 18 C 2 103 ,
1,0078.
103 /.
0,61 1029 . ?
6.6.4.
d
h, .
, .
6.6.5. ,
E0 . .
6.6.6. ,
?
6.6.7. , . .
6.6.8. C , V .
?

6.6.9. . V . , ?
167

n ,
V , ?
6.6.10. n V .
. , k ? .
6.6.11.
1
2 .
. S, d.
6.6.12.
d1 d2 . 1 2 . S. . ,
q?
6.6.13. S1 d1 S2 d2 .
. .

6.6.14. q. ,

= 0 (1 + x/d)1 , x , d . x.
S.
6.6.15. ,
. V .
.
. ?
. , 2 , , , 1 , F1 ?
.
. ?
?
. , R, , r < R?
, .
6.6.16. Q S, R
. . , . ,
.
168

6.6.17 . E
. d, S. , , .
, ?
6.6.18. r Q , R. .
. .
.
6.6.19 . r Q .
R. .
6.6.20 .
Q,
x? ,
d. , ,
a b.
6.6.21. , .
, V . d,
, . . ? .

6.6.22. .
S.
, Q?
6.6.23. C q. ,
?
6.6.24. C . ,
?
V .
6.6.25 . C 1 .
W 2 > 1 .
169

, ? , .
3,1, 88.
6.6.26. V . .
, .
2/3
. ? , .

6.6.27. (. 6.6.26), V , , ,
, .
. , ?
. , ( = 81)
. c = 4,18 /( ),
= 1103 /3 , d = 1 ,
V = 300 . ,
.
6.6.28. 0 C 1 1,00056. , , ,
.
6.6.29. r, E. 6.2.14.
6.6.30. r. . n.
.

170

7.1.
7.1.1.
?
7.1.2. 200 / 107 /. .
? ,
, 45 .
7.1.3. , q, m,
d t. v
. .
7.1.4. m q > 0 , . E, d. v .
?

7.1.5. -, ,
.
-?
7.1.6. m q l , .
, E.
171

7.1.7. ) v
. V ,
l. ,
d b ,
? n.

7.1.8. 1012
. -
1 /.
490 , 10 .
. 0,1 .
.
7.1.9. . 1 4 . 2, 3
. 2 ,
1, . . ,
, H0 , H . ,
H 4, 3, 4
.
7.1.10 . , v1 ,
1 2 .
, ?

7.1.11. , , 31016 .
1 , 1 .
)

172

, me ,
e.

7.1.12 . , V , , .
R1 R2 . V0 .
7.1.13. , , ,
, . B . B AB, ?

7.1.14 . A q. AB t.
3q, A?
.
7.1.15. (1) , (2) . , .
, ,
n?

7.1.16. R Q,
.
, ? l.
7.1.17 . M R . Q. . , , v m
q, Q. ,
.
7.1.18. E , , , . ,
. m, q,
l.
7.1.19. l m, E.
q.
173

7.1.20. ,
m, q l, E,
/2 .
7.1.21. m, q, Q, Qq > 0. l.
, Q.
7.1.22 . , h Q.
l, m, q.
7.1.23 . m, q,
R, Q.

7.1.24 . , v,
R. , C?

7.1.25 . K ( ).
. , , , ? .

174

7.1.26. , V0 ,
.
d l,
L l. V  V0 x LV
V0 : x k(V /V0 )L. k.
7.1.27. . , ,
r . v. , ,
, p = eq/(20 vr),
q r.
, , .

. , q1 , q2 . r, q1 v .
. , 100 ,
103 .

7.1.28 . , V , . C. V
C V V0 (eV0 ): kCV /V0 .
k.
7.2.
7.2.1. , V0 , ,
V . , ?
7.2.2.
.
, v,
l?
175

7.2.3. , : ) k ?
) k ?
, , .
7.2.4.
1 1
1
+ + ,
a b f
f , a ,
b , , .
7.2.5. ,
A x0 y0 , f , ? y0  x0 , f .

7.2.6. , ,
,
d.
. ?
. , , 7.1.27. .
7.2.7 . , V0 , .
, V ? .
d.
7.2.8 . , d , , . ,
176

V .
, V0  V .
7.2.9. , V0 ,
. , V  V0 ?
7.2.10. , V0 , , R R ,  R? ,
V  V0 , .

7.2.11 . 7.2.10 , , , L  R ,
.
7.2.12. , ,
d .
V . , , , a,
b. .
7.2.13.
,
, , l . V0 .
d.
, l  d.
7.2.14 . , 12

177

, ,

. , E1 , E2 ,
eV ?
7.3.
7.3.1. () . E
.
()?
l.
7.3.2. .
1 2,
V . V 2 3
. 2 3 l. ,
, : ) ; )
.
7.3.3. ,
1 . ,
, ,
200 .
7.3.4. . - , . . , 1
. l, d  l, L  l. V .
. - , V =
10 , L = 30 , l = 3 , d = 5 .
7.3.5.
V1 = V0 sin t, V2 = V0 cos t.
( ) 5/V0 . ?
7.3.6. l, V .
?
7.3.7.
 1/ ( ) . S. .

7.3.8. , V , ,
178

l. . f
.
.
7.3.9 . , V ,
. ,
V0 sin t. d l.
7.3.10. l,
A B.
V0 . d.
. ,
, ?
. A
B,
?

7.3.11. t = 0
E = E0 sin (t + ).
.
7.3.12 . ( -) ?
1011 /, 3 1015 1 .
7.3.13 . E = E0 sin t .
, M  me .
7.3.14 .
0 . .
E = E0 sin t.
7.3.15 . ,
, ,
E = E0 sin t. 0 ,  0 ,
ne .
7.4.
7.4.1. r
, , r ?
7.4.2. a . .
.
179

7.4.3 . a .
. 1840 ,
.
7.4.4. v
. ,
.
7.4.5. v1 v2 . ,
.
7.4.6. . .
, ? , ,
m1 , q1 , R1 . m2 , q2 , R2 .
7.4.7.
v, d
, . ?
7.4.8 . r ,
,
, .
, r ?
7.4.9 .
m, q. ,
d .
, d.
7.4.10 .
m .
d. ,
, q, q.
.
7.4.11. H2+
,
. R.
, r.
7.4.12. Ze r
. , ?
7.4.13 . r.
, ?
7.4.14 . m M .
m n , .
n ?
7.4.15 . . ,
? v.
180

7.4.16. ( )?
7.4.17 .
v. . ,
.
7.4.18 . q u v, , ,
, l . , , , ,
r.
7.4.19 . l .
t0 .
3l .
?
7.4.20. m, q,
. R, Q, M .
,
? qQ > 0.
7.4.21. m, q, v0
,
. R, Q, M . .
, ?
7.4.22 . m, q, , . R, Q, M . .

, ?
7.4.23 . , q
m, ,
l. . .

7.4.24. M R m, q.
l. , .
7.4.25. R m, Q. .
, ? ?

12

181

7.4.26 . H h0
,
. ,
, . ,
m q. ,
?

7.4.27. m, q, l . -
2l . , .
7.4.28. , ,
l1 l2 . l0 ,
q. .
7.4.29. m, q, ,
v .
.
7.4.30. R ,
v. , .
, : ) Q Q;
) q, Q. , .
7.4.31. q. .
.

7.4.32 . h . S, m,
. , ? h
.
182

7.4.33. R
r, .
m, q, , ,
?
7.4.34 . , , , .
l R1 R2 .
v0 . . m.
7.4.35 . , l , ? m q.

7.4.36 . . n.
h v
, .
p

,
:
)
v

he
n/me ; ) v 
p
he n/me , e, me ? .
7.4.37 . , , .
, .

7.4.38 . q . l q.
M , . , ,
, .
?

183

8.1. . .
8.1.1. .
l = 240 .
n = 1011 , .
?
. , ,
r = 0,5 1010 .
8.1.2. l , , q. ,
I.
8.1.3. a = 30
v = 20 /.
,
E = 1,2 106 /. ?
8.1.4. . ,
.
8.1.5. ,
,
1 , 30 ?
8.1.6. , - ,
. v,
. .
v?
8.1.7. - , u, ne . , v, . .
8.1.8.
j = 0,2 /2
30 . s = 104 2 , .
.
184

8.1.9. j = 1 /2 r = 10 . ,
V = 220 . .
8.1.10. j, v. .
8.1.11. r =
3 v = 108 / I = 100 .

,
.
8.1.12 . E. , 0 , v0 . ,
, : ) , ; ) .
0
?
8.1.13. : ,
( ), , . , . ,
. .
8.1.14.
,
. , ?

8.1.15. , ,
, , .
?
8.1.16 . d = 0,5 .
S = 10 2 . V = 5000
I = 1 . ,
.
. ?
8.1.17 .
d 185

, v x.
, , . , ,
= V (x/d)n . (x) v(x), ,
, n.
V . S.
8.1.18 . . , , , (x, y, z).
, n ? ?
8.1.19. .
i. r . r.
8.1.20. . A I,
B I. , -
,
A B.
?
, B I?
. , h

I.
8.1.21 . l
v q. r
; r > l.
8.2. . .
8.2.1 . . , ne ,
. .
. .
8.2.2 . ,
, ,
. O2 , . 10 10 2 ; 10 10
, . 1000 , 1,5 .

, . 107 .
8.2.3. , . .
, . , ,
. ,
186

, n,
v e.
8.2.4. r = 0,1
= 103 /. ,
= 103
. s = 0,5 2 , =
6 107 /.
8.2.5. . ?
8.2.6 .
0 C, .
?

, /(K) , 107 /
385
111

6,0
1,7

, /( K) , 107 /
60
34

0,9
0,5

8.2.7. A B , AB. A B, , , .
A B l. j, .
8.2.8. j
1 2 .
.
8.2.9. , 1
2 , , 1 . ?
?
j.

8.2.10. x: = 0 a/(a+
x). x j,
x?
8.2.11.
Q0 .
. Q0 , ?
. ?
8.2.12. . A
B? ?
187

8.2.13. l S.
V . .
?
8.2.14. 1 2
, . V .
.
8.2.15.
500 , .
,
, . , ,
, .
. ,
2,83 108 ?

8.2.16 . . R0 .
,
. .
.
8.2.17 . r. , ,
V . ,
?
8.2.18 . 30
60 .
300 . 4 /.
.
8.2.19 . , , . , , .

?
,
?
8.2.20.
r1 r2 , q.
,

.
188

8.2.21 .
R. . ?
8.2.22 . ,
?
8.2.23. , , ? ( -)
I = 100 S = 1 2 .
ne = 2,5 1022 3 .
8.2.24 .
.
, R.
, q. l,
.
F . .

.
8.2.25. ,
, .
, .
,
. q, l,
R. :
) v; ) ,
F .

8.2.26. - . , , .
189

W . .
?
?
8.2.27 . , . . ,
I0 ? eV0 . .
8.2.28. , R.
l , , E ,
. l?

8.2.29. : Ag +
Cl = AgCl + e
(1/2)Cl2 +e = Cl ( ). AgCl
3280 .
Ag + (1/2)Cl2 = AgCl AgCl 29 380 .
, . . , (1 2,6 1019 ).
8.2.30. H2 SO4
4,40 105 / , CuSO4
2,34 105 /. , ,
, .
1 %.

( 1,09 .). ?
?
8.2.31. 0,1 8 .
CuSO4 5H2 O ( ).
8.2.32 . , q, m .
. , k ,
km . , .
190

k ! , - .
.
8.2.33.
: j = e+ n+ v+ +
e n v , e , v n ,
. ,
, , ?
8.2.34 . - E. , V  E. , , , ?
8.3.
8.3.1. 150 .
, 3, 15 150 .
50 1 .
?
8.3.2.
, 100 1 180 , 1 ?
8.3.3. 100
10 . , 90 ?
8.3.4.
? ) .

8.3.5. . R.
. ,
)

.
.
191

, ? . r.
. R .
, , ? .
r.
8.3.6. 4 36 .
100 .
, , , 6 .
, 4 6 ?
8.3.7. 20 , 120 .
5 1 , 20 .
.
, 250 100 . ,
.
8.3.8. ( 100 200 ),
. ,
. ?
8.3.9. ? , ) ?
8.3.10. , , A B, .
R1 , R2 , r , rx .
, .
?

8.3.11. : V1 , I1 ; V2 , I2 ; V3 , I3 .
, . , ,
.
)

192

.
, ,

.

8.3.12 . . ,
, , , R, ?
8.3.13. ,
?

8.3.14. . r, r?
. r C D,
A B ?
. I. n- ,
. ?

8.3.15 . , . R1 R2 ,
R1
, ?

13

193

8.3.16. R
IR I.
() E, ( ,
), E, .

. , , .
8.3.17. , 10 , 3 ; 20 , 1,6 .
.
8.3.18. ,
1 5 . 1 .
20 , 2.
?

8.3.19. ,
.
, . : , ? ,
. ,
(). ,
120
20 ) .
8.3.20. 4 120 ,
2 160 .
.
8.3.21. 4 .
12,6 .
6 11,1 . .
8.3.22. , .
, .
, . , . )

194


.
8.3.23. R1 , R2 , R3 , , I3 ,
R3 , . R1 R2 .
8.3.24. , ,
. .

8.3.25. , .
. r.
i. A B,
A B.
. , A B
. r, , , i.
. r. A B. C D?
8.3.26. , ,
. , ,
. , , A B.

8.3.27 . . , ,
A i, B i,
, A B?
, r?

. , r?

. A B
C A, . r.
8.3.28. E1 = 20 , E2 = 30 r1 = 4 , r2 = 60 .
E r ,
?
195

8.3.29.
, E.
(3/2)E. .
.
8.3.30. , ,
. , ?
8.3.31. , ?
, 1,5 ,
2 ?
8.3.32. , 3 , 1 . 1,5 .
8.3.33. .
6 .
, ?

8.3.34. R,
, I. , , V  IR?
.
8.3.35. , ,
N . , , N0 ?
8.3.36. , n r, 10 r.
, , 196

, ,
.
8.3.37. R1 , R2 . , , .
.
8.3.38. , ,
50 , 0,2 2 ,
0,2 , ,
100 108 , 105 .
, ?
8.3.39. 10
5 500 ; 1 %.
? , 100
?
8.3.40. , , I? E, r.
?
8.3.41. 100 20 ?
80 %? 0,1 ,
, ?
8.3.42 . . , . , r r. r
, , R r r.
8.3.43. E ,
V . r. , , ,
. ?
?
8.3.44. 4 1
. , 6 . ,
.
8.3.45. C
V . , , , , , 1.
8.3.46. , , R ,
v. , , I.
13

197

8.3.47. a R . v ,
ne . . (, ).

8.3.48 . : N = (T T0 ).
: R = R0 [1 + (T T0 )], R0
. I?
8.4.

8.4.1. .
. 4 .
. A B ?
5 , A B?
. , .

8.4.2. V0 . V
, ,
. ,
, x
. V , l, .
, V > V0 ?
198

8.4.3. :
, , , , .
. k.
B . A , x.
l;
S  l2 , x2 .
8.4.4. A B.
?
?
, , ?
8.4.5 . , . V , .

8.4.6 . , ,
C C/2
A. q.
8.4.7. ?
?

8.4.8. - , . V0 . .
?
8.4.9. , ?
?
8.4.10. E n . C,
? ?
199

8.4.11 . C q. ,
I. , ,
, . ?
8.4.12. . .
1/1000 ?
8.4.13 .
. ,
,
.
? ,
, , k ,
.

8.4.14 . V0 . , T . ,
. , , .

8.4.15 . C, V0 , R. dV /dt ?
?
8.4.16. , . .
V , . , , 80 ;
1 . 120 , 80 < V < 120 .
( )?
200

8.4.17 . , , V ? V1 V0 < V1 . .

8.4.18. . v , q.
, , d.
. , v q?
8.4.19. , a a,
,
. , d. E. , v,
, ?

8.4.20. V I = V 2 ;
. ,
R E.

8.4.21. - , ,
6 1,5 .
.
?

201

9.1. .

9.1.1. 10 , , 15 , 1,5 .
.
9.1.2. 1 , 1 /,
10 . 30
. ?
9.1.3. l,
, F , I. : )
l + L, ,
I1 ) ; ) R, I2 ,
?
9.1.4 . , , , ,
, . V , B. . a, b.

202

, ( ) , .

9.1.5.
0,16 80 . , ,
. , , 2 , 1 .
9.1.6. , . B. , m, a. .
9.1.7. . B . S, I.
. , , , N = BM , M =
IS .
. , , , , ,
~ = [M
~ B],
~ M
~ , IS,
N
.
9.1.8 . B
. m, I.
OO0 .
9.1.9. OO0 , .
, I. B, . .

9.1.10. , ,
~ = [M
~ B].
~
B, N
9.1.11. .
OO0 . , I. B,
. .

203

.
, OO0 ,
. . ,
, I. B, .
?
9.1.12. R B ,
, /2
B. I. , .
9.1.13 . , , . P , n, R, I.
, , ?
9.1.14. R, I, - .
. B, , . m.
.

9.1.15 . , e
. I. F , ? R. ,
I, .
9.2. .
)
~ ,
9.2.1. E
)
v , , ~ = K[v E].
~ K 0 0
B
1/c , c . ,

.
9.2.2. , ,
v.
)
)

204

,
.
, v  c.

9.2.3. , I.
9.2.4. 2,5 107 .
1 , 1 .
?
9.2.5. ,
( 30 ) , 10 ,
, . .
9.2.6. .
x y,
, I.
9.2.7. . ,
. I.
9.2.8. . , 9.2.1, , q, v,
r . - ~r v .
. l, I, r , l  r. - ~r
.
9.2.9. , l1 l2 , ,
~l = ~l1 + ~l2 , .

9.2.10. R I.
h .
9.2.11. , ? .
9.2.12. , , , .
I. .
9.2.13. I R. .

205

9.2.14 . R. I.
h .
9.2.15. . , I0 .
, ,
. ?
. , , ?
9.2.16. ,
M , h , , , .
9.2.17 . a a
I A, r,
. - ~r .
9.2.18 .
.
.
9.2.19 . . . M0 . , ,
,
, nM0 , n
.
. , a a h (h  a), , .
B.
.

9.2.20. R
h. .
M . l .
9.2.21.
1 10 20 .
,
2e = 1,85 1023 /.
9.2.22. R B.
B0 , .
206

, , , ? ?
9.2.23. , H , F . a a h. ,
h  H, H  a.
9.3. ,

9.3.1. , 9.2.1,
,
v . .
9.3.2. ,
9 / . 10 , 10 .
9.3.3. , i?
9.3.4. , i1 i2 .
.
9.3.5. I. b . ,
?
9.3.6. . a b (a  b) I. E. ,
a .
. 4 107 ,
5 108 . ,
.
9.3.7. , , i. , , i,
Bk = 0 i/4 Bk = i/c , ,
.
9.3.8. Bk = 0 i/4 9.3.7,
.
. 2h h ,
i.
. , i.
. , a, j.
.
9.3.9. I?
n, R.
9.3.10 . R
i.
207

. AA0 .
. x1
x2 ? ?

9.3.11 . . , . ,
,
.
. , ,
, .
?
. l r. M .
r  l? r  l?
. ,
.
9.3.12. , a a h
(h  a), , . B0 .
a a, ?
9.3.13. 0,5 .
1 10, 600. ( A B).
9.3.14. R h  R,
= 1 + ,  1, B0 .
B0 ?
9.3.15.
, , 0 .
, . , , .
. r I. .
.
208

. a ,
. j.
x ?

9.3.16. , N , I.
R, r. .
9.3.17. . I , , .
.
. I . , , I 0 .
.
. ,
. .

9.3.18 . I . r?

14

209

9.3.19.
h . . x.
9.3.20. h h/2, .
, , ,
. OA,
.
j.
9.3.21 .
, ,
d.
. j.
9.3.22 . . R, a
, , .
, j. , .
.
, a 0, j
R,
B0 .
B0 ?

9.3.23 . . I, n,
R.
.
9.3.24 .
r ,
. M .
x ,
?
9.4.

9.4.1. B.
. a, 60 ?
. S, ?
210

9.4.2.
R. B
.

9.4.3. , ,
i, .
9.4.4 . , , ,
.
9.4.5. .
. ,
.
9.4.6. B, ,
. ?
?
9.4.7 . , , . . 1 3
AA0 , B1 B3 . 2
4.

9.4.8. . - , , x:
Bx = B0 x/x0 , x0 B0 .
. x
r? .
.
B = B0 (x/x0 )n . . Br ,
Bx = B0 f (x)?
211

9.4.9. R B0 x/x0 , .
?
9.4.10 . . , i.
R.
. I?
R, n.
9.4.11. , ,
. ,
F . ,
S.
.
9.4.12. I.
n. , , 1 2 . ,
.
9.4.13.
, . : ) r R,
l  r, R; )
r, n , , .

212

10

10.1.
10.1.1. , 20 , 0,1 . , .
10.1.2. , 200 ,
, 70 . ,
, .
10.1.3. . ( )
m q B.
.
1 .
10.1.4.
K1 K2 , ?
10.1.5. , B?
q, m. .
10.1.6. , B, - . -, - R.
, .
10.1.7. l.
v B, .
?
10.1.8. -, 0,1 . A , 10 .
.
1 H+ , 2 H+ , 3 H+ , 4 He+ ?
, 16 O+ 15 N+ ?
14

213

10.1.9. 39 K+
41 K+ , B, .
- V .
V /V0
, ?
10.1.10 . A v , , B
. , A
, . m, q.

10.1.11. B
v m q.
, .
10.1.12 . B
v , . ,
, .
10.1.13. . R
, B. K;
I. I B. B,
.
. I B P1 P2 . ?
10.1.14. v
. 2R, , 214

.
?

10.1.15. R . ,
B, ?
10.1.16. , ,
V ? B, . h R.
10.1.17. . l
E, , E, B.
, . ,
, , L  l. - ,
?
. - B = 1 , E =
5 105 /, l = 5 , L = 50 .
.
p ;
m = me / 1 2 ,
, m . 10.1.17
.

10.1.18. , K, V , B, R.
.
10.1.19. d
B. ,
215

. ? ?
10.1.20. ,
, B
. m q. . .
.

10.1.21. R, , -
v. R BR . , .
10.1.22 . , M 10.1.21
: M = (1/2)Q, Q . ,
.
10.1.23 . , - , , B1 B2 , B2 > B1 ?
R, Q, m.
10.1.24. . . , OO0 . , M + (1/2)e,
, M OO0 , .
. , M + (1/2)e .

216

10.1.25. I II B1 B2 AA0 , .
I R .
I II?
M +(1/2)e (. 10.1.24)?
10.1.26 . , A C -
S1 S2 ,
e/2.

10.1.27 . , , ,
? B1 , B2 > B1 .
10.1.28 . ,
B1 B2 .
. B1 R,
B1 .

10.1.29 . , . , ,
, ,

.

217

10.2.
10.2.1. . B1 , B2 ,
.
v B1 .
. ()
, .
10.2.2 . , Bx = 0, By = 0, Bz = B0 (1 + x).
v, v  eB0 /(me ).

10.2.3.
. B
. E . l
m q. .
.
10.2.4. . , , ? E,
B.
10.2.5. E B
. ?

10.2.6. ,
, E B ?
10.2.7. ,
= qB/m ,
(

).
10.2.8. B, . A ,
218

. , ,
V . ?

10.2.9. , d,
. B, . ,
. , B = 0,1 , d = 2 .
10.2.10. v
B. E,
. .

10.2.11. q
B F .
10.2.12.
, 0,7 104 .
.

219

11

11.1. .

11.1.1. - ?
11.1.2. 0,1
1 / 0,3 .
?
11.1.3. , abc (b  a, c),
v B , .
.

11.1.4 . , r
, Ze. ,
, , B.
11.1.5 . H 40 . ,
, ,
. 0,72 1019 .
11.1.6. B
.
, v.
11.1.7. , a a, .
220

. , , V . ) .

11.1.8. , a b,
B, . .
. , , . R.
. , ?
11.1.9. , b,
, v . , B.
, , , , a
b b, b.

11.1.10 . AB, , v, AB,


OC OD, . OC l AB OC.
B, .
,
O C.
11.1.11. , . S = 1 2 ,
L = 100 . , = 0,25 ,
v = 1 /. , , B = 104 .
,
)

11.1, 11.2 .
221

, . ,
.
11.1.12 . , h = 10
, .
S = 1 2 .
, B = 1 .
v = 2000 /, = 50 /.
.

11.1.13. d I.
B. .
1 2 ,
h, .
11.1.14. . ()
(), , B. A C . I .
( ) , . , m? l. ,
, L.
. B = 1 , l = 0,1 , L = 1 , I = 10 ;
1013 .

11.1.15. a b .
, . , I .
, .
, , .
B .
11.1.16. B, . v. ,
R0 , r0 .
.
222

11.1.17. S
B. R.
, ?

11.1.18 .
0,2 . 1 .
1,7 108 . , , ,
, . , ,
, 104 .

11.1.19. B , , .
, l, m. , R.
?
11.1.20 . 11.1.19
,
: ) R; ) C.

11.1.21. , , , I. , 223


, .
11.1.22 . a
m. R. .
, B = B0 (1+h).
11.1.23 . . L m
. B . ,
,
? .
11.1.24. .
E, B, R, L.
. ,
F .
. ,
, .

11.1.25 .
B, .
, R? , R = 1 ,
r = 0,05 , = 2 50 /, B = 1 .
11.1.26 . O
, a a.
0 , , , . ,
, M , B.
11.1.27. , ?
11.1.28. ,
24 , 20 600 1
0,2 . ,
- 1200 1 ?
11.1.29. , E
R, , -, E0 f0 ? M .
11.1.30. - ,
300 50 ,
224

4 % ? 60 ?
11.1.31 . . E.
0 . , I0 .
, .
, ?
, .
11.1.32 .
V .
, , ,
I1 , I2 , ? F , ,
R.
11.2.
11.2.1. , , 1
1 ? 100 ? 1 ?
11.2.2.
r = 0, 1 : B = t ( = 103 /).
. l = 0,2 ?
11.2.3. , , , , . A B
, C . .

11.2.4. 8 B = t2 ( = 104 /2 ).
. l = 0,1 t1 = 1 ?
t2 = 4 ?
11.2.5.
.
15

225

j = t. .
11.2.6. l0 = 20 r = 2 I = I0 sin(2t), I0 = 10 , = 50 .
n0 = 200. .
l = 5 r = 1 , ?
n = 100.
11.2.7. , , .
. C,
.
. C1 C2 .
.
11.2.8. . ,
, C1 C2 .
ab.
, .
.
C3 , , ?

11.2.9. , , 1 /.
, ,
.
0,1 /(2 ).

11.2.10. R C.
. , , T /(CR2 ),
T ,
.
226

. I, T , ,
, V ,
, I = CV /T .
, k I.
11.2.11. C = 0,01
D R = 100 ,
. V = 0,5 . , .
11.2.12. , m, . h. d , h.
. ,
, a?
bt2 ?
. , 10 ,
1 , 1
10 ? , ?

11.2.13 . n
r , B = B0 cos t.
. .
11.2.14 . m r q. . . l < r
,
.
? , B
? ,
, .
11.2.15 . r0
: B = t. ,
r > r0 ? t = 0 .
11.2.16 .
. ?
227

11.2.17 . z
, .
z , ?
? , ?

11.2.18 . r , .
B .
. .
11.2.19 . ,
, , . a F = m a. m
.
. 0,1 103 1
? 1 2 ,
1023 3 .
. ,
?
1023 3 .
11.2.20 . C, V ,
.
11.2.21 . - , ,
1018 . .
11.3. .
.
11.3.1. I
S, .
n.
.
11.3.2. r
, , /2 .
n.
?
228

11.3.3. r. N .
n. ?
11.3.4 . R r. . n, N .
I = I0 sin t. .

11.3.5. . r l  r?
n.
. , me . S,
ne .
, ?
11.3.6 . r1 = 0,05 , r2 = 0,1 , n = 10 000.
.
11.3.7. v = 10 ,
L = 0,01 . V = 10 .

B = 0,1 ?
11.3.8. ,
d = 0,1 ,
h = 5 . ,
.
11.3.9 . r1 r2 (r1 < r2 ). .
. ,
.
11.3.10 . r1 r2 , 1 .
2 .
.
, .
11.3.11 . . r,
15

229

h  r. , . .
11.3.12. k . ?
11.3.13.
r1 r2 ,
, ? l1 , l2 .
n1 , n2 . ,
.
11.3.14. L1 L2 . L12 .
.
11.3.15 . .
L1 L2 . ?
.
11.3.16 . I = I0 sin t.
, ,

.
.
,
N1 , N2 . ?
S. l.
11.3.17. . ?
11.3.18. ,
I1 /I2 = N2 /N1 , I1 I2 ,
N1 N2 .
11.3.19 . . ?
.
. ?
11.3.20.
,
. K ?
11.3.21.
? , ?
11.3.22. ?
11.3.23.
.
, ?
11.3.24.
. ,
, . 1 230

40 2 10 .
1, 2
10 ?
11.3.25 . 1 : 3.
, 100 .
.
.
11.4.
11.4.1. L. , E. .
?
11.4.2. ,
L R,
: ) : I = t? ) : I =
I0 sin t?
11.4.3 . , . L , t .
I. ,
.
R.
11.4.4 . E = E0 sin t t = 0
L. .
. .
11.4.5. , , D L
K V =
V0 cos t. t = 0 K .
. .
. .
11.4.6. , C0 L, K.
, ?

11.4.7. C
V0 . K1 . ,
231

L , K2
K1 .
R?
11.4.8. . ? , ?
. L,
N , ?
= 50 , L = 0, 1 , N = 10.
11.4.9. E
t = 0 L C. .
11.4.10. L1 L2
K , . C, V .
11.4.11 . , , t = 0 K. , : ) V0 ; ) V0 cos t.
, V0 = 100 , L = 102 , C = 103 , = /2 = 50 .

11.4.12. . I = I0 cos t x I0 , O .
R, L
C? ?
. , L, R
C ,
: I = I0 cos t.
11.4.13. ,
.
, R = 10 .

232

11.4.14. , .

11.4.15. , 100 10
.

11.4.16 . . A B
V = V0 sin t. M N
R0 C0 = RC?

11.4.17. , . .
, , E0 = 200 , R =
100 , C = 104 , L = 1 . = /2 = 50 .

11.4.18.
. .
, 50 .
11.4.19. ,
, .
, ,
, .
.
11.4.20 . C0 V0 ,
C . K C, V ?

11.4.21. . , L1 L2 L1 I1 + L2 I2 .
.
233

. C, V0 , L1 .
L2 , K , L1 ?
11.4.22 . . , L1 I,
K . R
?
. K L1 I1 ,
L2 I2 . ,
L1 L2 K.
11.4.23 . -
, 1
1 , 1 . .
11.4.24 . , R, L C,
. I1 I2 .
?

11.4.25 . LC . , L, C.
LC-, l? ?
11.5. .

11.5.1.
?
11.5.2. r0 ,
B0 , .
,
r. .
234

11.5.3.
?
11.5.4.
,
. , .
11.5.5. I, , , .
? .
11.5.6 . B0 .
, ?
11.5.7. L, I0 ,
B0 . ,
. ; r.
11.5.8. B ,
, .
L, D.
.
11.5.9. ,
. , ,
B0 ?
11.5.10 . S h, , l.
, I0 .
: ) l > h; ) l < h. .
11.5.11 . r l, R
. N .
11.5.12. , ah = 1001 , .
, ?

11.5.13 . , h
, I.
?
?
235

11.5.14. B = 2 .
n = 1000 1 . r = 9 , , , V = 100 ?
.
11.5.15 . 11.5.14 ?
11.5.16 . m, l r, v0 , N , L
R  l, I,
. , ? .
11.5.17 . r I. N , l  r, r/2,
m. , ?

11.5.18 .
S, l m,
B?

11.5.19. , r1
r2 , B.
r0
l  r1 , r2 , , ?
.

11.5.20.
. 236

, .
B . . ,
3v, , v. l, s, m. S.
. .

11.5.21. 11.5.20 , m1 ,
m2 , v1 v2 (v1 > v2 ).
11.5.22 . , L, v , ,
2 /2L,
.
11.5.23 . r m ,
T . L. ,
B, ? , ,
J. .
11.5.24 .
d.
(h  a, l). , , , i. , ,
? m.

11.5.25 .
v0 m, . ,
237

, I0 ,
x. L.
.
11.5.26 .
. v , , .
, ,
B0 = 10 , v = 3 /, r0 = 20 ,
= 0,5 , = 8,9 /3 .
, .
.
11.5.27 . B, ,
- , . . , r
h.
ne . , ,
B = 10 , r = 1 , h = 0,1 , ne = 1020 3 .
11.5.28 . , . , .
11.6.

11.6.1.
. , B
dE/dt. , ,
. ,


.

11.6.2. . v,
. E.
238


abcd .
? ?
. ,
, .
11.6.3. , ,
1
0,001 ?
11.6.4 . , ,
, ,
. .
11.6.5.
10 : E =
t, = 9 1010 /( ).
5 ?
11.6.6. .
? r, h, L,
N .
11.6.7. ,
E, v. .
?

11.6.8. v B. .
11.6.9.
v , . V , h.
. ? ?
. ,
?
11.6.10 . . ,
?
239

11.6.11 . .
, ,
: E = t.
r .
. : V = t. r0 , h.
r .
.

11.6.12 . , r0 , , r.
. I. .
.

240

12

12.1. ,

12.1.1.
. , , .

12.1.2. , : ) ; ) ?
12.1.3.
.
. z
t?
12.1.4.
h 
i
h 
i
z
z
+ 1 ,
+ 2
E1 sin t
E2 sin t
c
c
. ? ?
16

241

12.1.5. , E0 ,
+ ,  , , .
?
.
12.1.6. AB A0 B 0 .
c , AB. E. baa0 b0 , .

12.1.7 . 12.1.6 ,

. c/ .
12.1.8 .
(. 11.6.1), ,
c/ .
12.1.9 . E B
?
12.1.10.
. E B
a/2c, a/c, 3a/c.
?

242

12.1.11. . E AB A0 B 0 , d.
. , E .
. B, AB A0 B 0 ?
B .
12.1.12. . , . . . ,
.
. v
, B
E B = (v/c2 )E.
B/2, .
. .
. , Q, r, ?
v.
. , , , , , , . , , .
12.1.13. , .
I ,
, . . ?
? , 1-
k- .
12.1.14 . , E, v,
B = (v/c2 )E. dv dB = (dv/c2 )E. , ,
:
E (t, x) = cB (t, x) = (cvtx/c /c2 )E = (vtx/c /c)E.
t x/c , x
t x/c. , ,
243

v0 sin t, x t (v0 /c) sin [(t x/c)]E, t x/c v0 sin (t x/c).


E (t, x) = (vtx/c /c)E , v , .

12.1.15 . E = (vtx/c /c)E,


12.1.14, .
. , a,
. d, E.
. i0 .
, .
. ,
. x, ne , .
12.1.16.
,
, .
, , . .
12.1.17.
, , , .
. ( x < x1 ) x, ,
x > x2
E0 . .

244

12.1.18 . . = 1015 ,
ne = 1022 3 .
12.1.19. ,
, .

, . :
, , ,
(
AB). , AB,
. AB
AB , , .
, , AB.
12.1.20. . 105
. . , .

?

12.1.21.
. E.

.
12.1.22. E0 .
?
12.1.23. ?
30 ? 600 /2 .
12.1.24 . ,

.
16

245

12.1.25. , 12.1.19, ,
. : ) E , ,
; ) B
.
12.1.26. ,
, P . .
12.1.27 . (. 12.1.19)

v . , ,
, , v/c .
.

12.1.28 . v .
E.
?
12.1.29. , , .
0 . .
12.1.30. k . .
12.1.31. : )
, ; )
, ? ,
.
12.1.32. ,
1 2 1 < 2 ?
1 > 2 ? .
12.1.33. ,
, ,
, .
246

12.1.34. .
r0 /c. t ? .
12.2.
12.2.1.
.
. , , : )
c , ; )
c.
12.2.2.
n? ?
12.2.3. , : )
(1 = 3 ); )
(sin 1 / sin 2 = v1 /v2 ).

12.2.4. , ,
b.
< b.
12.2.5. . , k ?
12.2.6. , 200
1 , K -
( = 1,541 1010 ) 200 .
.
247

12.2.7. r .  r.
.
? z0 , r2 /  z0  r.
12.2.8. (, ) , , ,
, A
, ,
. ?

12.2.9 . , , .
A (. ). ,
,
, , .
12.2.10. A ,
.
A b. I.
12.2.11 . .
. 248

. ,
A. ?
. ,
, .
A ?

12.2.12 . a . ( a A ,
S, r, . . a = cAS/r. c
- , , , .)
12.2.13. . . , 10 , 105 . (
, ,
, .)
. , 0,01 .

12.2.14. , , 200 ,
, . .
12.2.15. .
, .
.
. ?
. ?

249

13

. .

13.1.
13.1.1. , .
.
13.1.2. 6 26 . . 1 , 2 .
13.1.3. , , , , , , . . . ,
. . . , ,
(. . . ,
). .
13.1.4. , , .
, . (, )?
? ( .)
13.1.5. .
.
?
13.1.6. h.
, ?
13.1.7.
.
, , .
,
.
13.1.8.
90 . ? .
?
250

13.1.9. . , .
13.1.10 . ,
.
( ).
13.1.11.
, . .
13.1.12. h

, . ?

13.1.13.
R.
13.1.14. , l1 l2 , l1 l2 = f 2 , f
.
13.1.15.
60 . ,
.
13.1.16. . ,
80 ,
40 .
13.1.17. .
80 , .
.

251

13.1.18. R/2,
R.
R/2 .
13.1.19. , . .
13.1.20 . ,
?
13.2. .
13.2.1. 60 . ? 1,33.
13.2.2. . , , 3 .
?
.
3 .
?
13.2.3. l. , ? L .
13.2.4.
1,5, 1,33.
?
13.2.5. . (n =
2,4), (n = 1,33) , .
. ?
13.2.6. -
? 1,5.
13.2.7. , n,
, l, ,
, , ?

13.2.8. 2.
.
. 1,5.
13.2.9.
R.
, , . n > 1,5. 2 = 60 ,
252

R. .
?
13.2.10 . , r. ?
n.
13.2.11.
, k ,
, ?
n, N .

13.2.12. h n = n0 h h  n/. R.
, , , , .
13.2.13. ?
13.2.14. . ,
. ,
(n1).
n.
. ,
. ,

(n 1). n.
. . R1 R2 , n.

13.2.15. .
30 , 1,6.
?
253

. , 1,6, , . 1 . .
13.2.16. 1,61 .
1,6 . .
13.2.17 . f r . n. ,
: )
; ) .
13.2.18.
R . n.
13.2.19. . n1 n2 .
. f .
. ,
?
13.2.20 .
45 , , .
?
n.
13.2.21 . ,
R, 2R .
.
13.2.22. , ,
,
. , , . , ?
13.2.23. , n, x .
, ?
13.3.
13.3.1.
. .
?
13.3.2. .
13.3.3. 90 .
, ,
. , ,
, 30 .
13.3.4. ,
?
f .
13.3.5. AB , , 254

A .
, . p f . ,
.
.
13.3.6. l
f , . a .
.

13.3.7. 15 30 , 9
51 . .
13.3.8.
?
. 5 , 10 .
10 ,
0,5 .
255

13.3.9. v.
,
, ,
, k?
13.3.10.
r1 . r2 . , ,
. f , D,
r2 > D/2 > r1 . .
13.3.11 . , 2 4 , 4.
( ) 2 8.
13.3.12. ,
, 20 , ,
, 50 ?
13.3.13. , ?
13.3.14. ,
, . .
13.3.15. 10, . .
.
.
13.3.16 . 2
f , a (a < f ).
?
.
13.3.17. , , .
13.3.18. . f
. ,

, a .
. - ,
f , . .
13.3.19. f1 f2 .
, ?
?
13.3.20. .
l, f .
, l  f .
13.3.21. 30
15 . ,
.
13.3.22. 0,6
, .
, , , .
,
.
256

13.3.23. f R, d .
S, ,
.
13.3.24 . R , .
, , l , l < R.
.

13.3.25 . - ,
f , , .
n. , . , ?
, , n0 .
13.3.26. , , ,
l, f .
?
13.3.27. f . ,
a?
13.3.28. f1
f2 . , l?
, , ,
k ?
13.4.
13.4.1.
100 .
, 100 ?
13.4.2. ,
, 200 . ,
30 , 45 , 60 ?
13.4.3. 2 ,
120 . 1 .
.
17

257

13.4.4. I h . , .
x ?
13.4.5. . , AB,
? .

13.4.6. ,
, ?
. ,
.
400 000 , 2000 .
13.4.7. I - . ,
k, , .
13.4.8 . 1 , ,
300 .
.
13.4.9.
,
. ,

.
13.4.10.
. R
, . . ,
.
13.4.11. , , ?
13.4.12. , - ,
?
13.4.13. R, S, h SO
, . , h  R, l.
13.4.14 . S a < 4f
. A, 258

f , x
? x A ?

13.4.15. ,
D0 , L0 .
D?
13.4.16 . , ?
D R? B.
13.4.17. , 100
. , 1 ,
?
0,14 /2 . .
13.4.18 . ,
. , d = 1 ,
x = 500 , , T 1000 ,
.
13.4.19. , 1 ,
. ,
, 1 ? , .
13.4.20. ?
13.4.21 . , , ,
. , ? .
13.4.22 . k1 , k2 t2 ?
13.4.23 . L -
10 %. r.
.
13.4.24 . 5 0,004 1 3 50 .
1 3 ,
10 , 20 ?
13.4.25 . , ,
104 ( ).
1 .
,
259

5 10 , , (-
).
10 20 ?
13.5.
13.5.1. ,
.
5 105 . 10 10 . , = h (h = 6,62 1027 ),
. 109 .

13.5.2. .
,
.
, , .
.

13.5.3. : - . - ,
?
-?
13.5.4. . - 1 2 ,
. . = pc,
c .
. ,
1 2 . .
260

13.5.5. . - ,
, .
 .
.
. , , . , h  me c2 .
13.5.6 . , v,
. , : ) ; )
? .
13.5.7. , ,
, , . ,
. ?
?
13.5.8. M R,
.
.
?

13.5.9 .
, . R 20 000 M 3 1011 M (
M ), , .

17

261

14

14.1. .
14.1.1.
104 .
?
14.1.2. 108 0 - A A B
-, 0 -: 0 + .
1 . 0 -?

14.1.3. , c , ?
14.1.4 .
,
. ,
?

14.1.5. , , 262

. , ,
, .
?
?

14.1.6 . 14.1.5 , v.
?
14.1.7 . ,
, v .
,
,
, .
, .
?
, ( ) l?

?
14.1.8. .
c.
, , , , K .

.
14.1.9. . . . . . ? , .
? . , .
0,01 %, . 100 000 / .
? . , ,
. , , .
.
, ,
, , . .
. , c
,
. : ,


263

? , , . ,
, .
: . . . . .
- . ?
?
14.1.10 . . ,
, v u. ,
v1 = (v + u)/(I + vu/c2 ).
. v. .
u. ,

(v + u)/(I + vu/c2 ).

.

14.1.11. , 225 000 / , , 240 000


/ .
?
14.1.12.
, n,
, , v
.
14.1.13. l
v. .
,
, ,
? n.

264

14.1.14 . . ,
L , , ,
l . . ?
.

14.1.15. . v, u.
.
14.1.16. v. , ,
u,
?
14.1.17. v .
, l.
.

14.1.18. - , , , ,
. ,
, ,
- , .
14.1.19. , c
.
14.1.20. ,
, l .
.
,
c? , ,
p
c, = 1/ 1 2
?
14.1.21. v , u, v u?
265

14.1.22. , , , . ,
1, ?

14.1.23. . ,
v = 4c/5. ? ? /2 ?
14.1.24 . 0 -, c, : 0 + . - c,
/2?
14.1.25 . ,
,
. ,
,  c.
14.1.26 . c . ,
?

14.1.27. , v,
u
. .
14.1.28 . , c, , , .
, .
266

14.2. , .
14.2.1. , 240 000 /?
14.2.2. .
, l?
14.2.3. - 2 106 ,
-, 30 , . -.
14.2.4. 30 , ,
, ( ), , .
3,2 1015 .
, ?
14.2.5 . , c ? .
14.2.6 . , c.
, n.

267

14.2.7. 0 - v
l.
, 0 - u?
14.2.8. 105 ,
, 0,6 ?
14.2.9. . ,
, ,
, p

v, lv/c2 1 v 2 /c2 ,
; l ( ,
). .
14.2.10. c l.
, ,
. .

14.2.11 . R, x. y
v. x. , ,
. , ,
. .

14.2.12 . c.
,
1 c?
14.2.13. . v
p ,
= 1/ 1 (v/c)2 , , , . , , . t = xv/c2 ,
x . ,

x0 = (x vt),
268

y 0 = y,

z 0 = z,

t0 = (t vx/c2 ),

x, y, z t ,
; x0 , y 0 , z 0 t0 , ,
v.
. : x, y, z, t
x0 , y 0 , z 0 , t0 , . . ,
.
14.2.14. , ,
- , , , v,
v, , , ,
.
. .
, , ,
v? , v ,
. , , v .
, , . , ,
.
, .
, , 1 2 .
0 . ?
14.2.15. , 14.2.5 14.2.6 .
14.2.16. z.
.
c z? ?
14.2.17. v
. l, .
, :
) u? ) a?
14.2.18. . ,
/(2) A
z = A sin t.
, c?
. , , ,
, y = A sin t.
14.3.
)
14.3.1. , , c
, , , E. (
, .)
14.3.2 . 14.3.1 , c
. ?
)

.
269

14.3.3.
~
, c,
, ,
.
14.3.4. .
~ .
c,
.
c?
. ?
. ~ E?
~
B
14.3.5 . 14.3.4 ,
~1 c, 1 = k .
~

14.3.6. . ,
. ) c
c. , ,
- , . ,
, ,
~ E.
~
B
. c B,  1?

~ . B,
~
, 1, c,
q
~ B],
~ = [
~ . E
2, , , . , q,
, . 1, 2,
~
~ c.
B
~ B],
~ = [
~
E
B.
?
)

270

c ,
, c ,
.

~ ,
14.3.7 . . c
~
, B,
~ E].
~ B
~ = [
~
E
~
~ c.
, E
~ . c
~
E,  1, = 1?
~ B
~ 14.3.8. . E
~ :
c
p
~ B]),
~ E]),
~0 = E
~ k + (E
~ [
~
~0 = B
~ k + (B
~ + (
~
E
B
= 1/ 1 2 ,
~0 B
~ 0 ; E
~ k, E
~ B
~ k, B
~ E
,
~ . E
~0 B
~ 0 c
c
. .
. ,
14.3.114.3.3, 14.3.5 .
. ,
14.3.6 , 14.3.7.
~ 0 B 0 .
. , 1 E
14.3.9.
c: ) ? ) ?
14.3.10.
c ?
14.3.11. R Q c. .
14.3.12 . c q.
14.3.13 . c
. E.
?
271

14.3.14 . h c
,
B, . .
.

14.3.15.
, c ?
14.3.16 . 14.3.15 n.
14.3.17. c .
?
14.3.18 . 14.3.17,
.
14.3.19. c. 1 c ?
14.3.20. j. 1 c ?
?
14.3.21 . ?
14.3.22. h,
j. .
c ?
14.3.23 . 14.3.21 c .
14.3.24. M .
v ?

272

14.3.25 . 14.3.24 .
14.3.26. . . . . E ~ E]
~ = [
~ (. , . .
B
. .: , 1960). , ,
, ) .
14.3.27. - . , , ,
E, ( ) B, (
) E. , , , , .
,
. ,
, .

14.3.28 . , , ,
, , .
, ,
c. ?
14.4.

14.4.1. , c
, . . , ,
, ? , : )
)

18

, me , e.
273

, ) ,
,
p
2
c, m = mi / 1 , mi .
14.4.2 . , v
, u, . .
.
14.4.3.
, v
. .
.
14.4.4. 14.4.3,
u: )
? ) ? ) )
?
14.4.5. , E, , 2v v. .
14.4.6 .
E.
, ?
14.4.7. , , p.
c .
, , , c ? ,
, , ?
14.4.8. ez R?
14.4.9 . ,
c, ,
? ?
14.4.10 . m q, E /(2).

14.4.11. , c
, l. . , , ,
? , : )
, ) A
m: A = c2 m.
274

14.4.12 . , v
, u, l.
, .
14.4.13. ,
E = 104 / (1 / = 1/300
), l = 1 ?
14.4.14. E = 105 /,
+ - E = 1010 (1 = 1,6 1012 )?
+ - ?
+ - m+ c2 = 108 , e.
14.4.15.
N = 1000 me c2 .
, ,

?
14.4.16. , E = 106 p = 0, 1 . ,
.
14.4.17. , U = 1 ,
= 30 , ?
14.4.18. , R = 0,3 B = 1 .
14.4.19.
E = 2104 = 60 .
l = 10 . .
,
?

14.4.20.
B = 1 ,
E = 1011 ? E = 1011 ?
14.4.21. , v = 2 106 , B = 10 .
275

14.4.22. R = 6 , , ,
N = 1000 me ?
14.4.23. c B.
.
14.4.24. 14.4.23 , ,
, 1 c.
14.4.25. , ,
h = 1000 , , B = 30 ?
14.4.26. v,
.
,
, B ?

14.4.27. ,
B R h.
14.4.28. E
B .
?
14.4.29 . ~ B
~ (E
~ B),
~
E
c? > k = E/B?
14.4.30 . .
B = 10 ,
. h = 10 . ?

276

14.4.31 . B,
~ E
~ c.
~
c. ,
.
14.5.
14.5.1. M -. -.
14.5.2. W 4 1026 . ,
- .
14.5.3. , M , c. ,
.
14.5.4.
k mp :
p = p + p p + p + M,

M = kmp .

,
. ?
14.5.5. ( )
- : e + e+ p + p? 0 -: e+ + e 0 ?
14.5.6. M m.
.
14.5.7 . M
m.
14.5.8. , ,
m. M .
?
.
14.5.9. m1 m2 , v1 v2 ,
, .
.
14.5.10. , m mc2 (1 = 1,6 1019 ).
, , 0 - -,
0,911 1027 , 1,673 1024 , 2,4 1025 ,
5 1024 .
14.5.11. 0 - -: 0 + . 0 -, ,
, - 270 .
14.5.12 . 0 - -, 0 + , - ?
14.5.13. , ,
Ee = 1, 73 E = 1 . .
, M =
3,9 1022 .
18

277

14.5.14 . ,

p v M , M M v0 ,
= 1/ 1 (v 2 /c2 ). , .
14.5.15. -
, . ?

14.5.16. .
0 -: p +
p p + p + 0 ? -: p + p p + p + ? - : p + p
p + p + (
p + p)?
14.5.17. - : e+ + e p + p?
, -
?
14.5.18. , ,
, ,
- : e+ e p + p.

14.5.19. . M ,
- m ?
?
. . - .
14.5.20. , 0 - 6 : 0 + + e + .
14.5.21.
, -: e + + ?

278

14.5.22. E = 10
Ee = 1010 ?

14.5.23. m . ,
.
14.5.24. , , .

279


1.
1.1.
1.1.1. v = 200 /.
1.1.2. v = 0,7 /; -.
1.1.3. v = 3 /; 1 2 .
1.1.4. 1,15 A.
3tA 2tB tC
1
1.1.5 . AO = L
, tO = tB (tA tC ).
2(tA tB )
2
1.1.6. l0 = l(v u)/(v + u).
1.1.7. v = c(0 )/(0 + ).
1.1.8. 0 = (w u)/(w v).

1.1.9. . t < l/v , vt


, ut. t > l/v
ut u(t l/v)
. . cos = u/v.

1.1.10 . , = 2 arcsin(u/v) A, .

1.1.11 . v = cl/ l2 c2 t2 .
1.1.12. u = v/ sin .

1.1.13. . .

1.1.14. x1 = vt x2 = a + v(t t1 )/2


: t0 = (2a vt1 )/v, x0 = 2a vt1 .

1.1.15. . .;

1.1.16. . .

) v = 0,

) v = 1 /.

1.1.17. . . ) x , . . . ) y /x = m/n, m n .

v sin + c2 v 2 cos2
.
1.1.18 . x = 2lv
c2 v 2
1.1.19. = 2. , .
1.1.20 . tg = 2ma/(nb), m n .
1.1.21. (cx , cy , cz ), (cx , cy , cz ).
p
1.1.22. t/t = (r 2 h2 )/(R2 h2 ).
280

281


1.1.23 . . . . ,
2R, 2R/(L2R) L > 4R; ,
L 2R, 4R > L > 2R.
1.2.
2 Rr
v
; .
R+r
1.2.2. t = 12 , x = 24 .
1.2.1. v =

v0 (t t0 )2
.
2t0
1.2.4. 20
15 /.
1.2.3. L = v0 t +

1.2.5. x > l(v1 /v2 1).


1.2.6. x = (/4)v0 t0 .

1.2.7. , .

1.2.8 . v =p La.
1.2.9. v = N/b.
1.2.10. t = R/q.
v03 t2 tg2
1.2.11 . . v =
.
s
r
1
q
.
. v =
2 ht
1.2.12. q = 126 3 /.
1.2.13. a = 277 /2 ;
28 .
1.2.14. v1 = 43 /;
v2 = 423 /.
1.2.15. . .; v = 600 /.
6 6,9 . x = 6,9 .

t.
1.2.16. 4 16.

1.2.17. . . 2.
1.2.18. . .
1.2.19. v = 0,72 /.

1.2.20 . t = (2 + 2 )t0 .
1.2.21 . t = (2t1 t2 t21 + t22 )/[2(t1 t2 )].

282

1.3. .
1.3.1. t = v/g t/2.
p
1.3.2. . t = 2D/g. . gt2 /2 A.
1.3.3 . /2 .
q
2 + 2gh.
1.3.4. vB = vA
v
1.3.5. t = (sin cos tg ).
g
1.3.6. ) vx = v cos , vy = v sin gt. ) x = (v cos )t, y = (v sin )t gt2 /2. ) y =
gx2
gx2
2v
v2
v2
2
2
x tg 2
=
x
tg

(tg

+
1).
)
T
=
sin
,
H
=
sin
,
L
=
sin 2.
2v cos2

2v 2
g
2g
g
2
1.3.7. L = 2 v /g.
2v 2 cos2
1.3.8. L =
(tg tg ).
pg cos
1.3.9. v = L(a + g).
2u
1.3.10. H =
(v cos u) tg2 .
g
2v 2
1.3.11. L =
.
g(tg + tg )

1.3.12 . m = 7 .
p
v 2 v 4 2gv 2 y g 2 x2
v2
gx2

1.3.13 . ) tg =
. ) y =
2.
gx
2g
2v
q
p
) v = g(y + x2 + y 2 ).
1.3.14. x = (v cos )t; y = (v sin )t gt2 /2 gt t, t ,
. ,

p gt.
1.3.15. v = 2Rgn/ sin 2, n ; = 0
.

2v
1.3.16 . t =
ctg v cos < 2gl sin ;
g
r



v
2gl tg
t = ctg 1 1 2
v cos > 2gl sin .
g
v cos
283

1.3.17. v1 = gt sin ,
v2 = gt cos .
1.3.18 . R = p
gT1 T2 /(2 2).
1.3.19 . v = g[2(H h) + L].
1.3.20. v = 1675 /, a = 0,034 /2 .
v =838 /, a = 0,017 /2 .
1.3.21. v = gR = 8 /.
1.3.22 . a < (4 + 2 )v 2 /(2l).
1.3.23. .
.
1.3.24. ( 3/2) 102 /; 5 105 ;
=p
5 103 1 .
1.3.25. a = k2 + k4 t4 /r 2 .

1.3.26. v =
gr.

1.3.27 . v = 5gR.
1.3.28. 27,5 42,4 ; 18,3 52 ; 0,2 73,4 .
2
1.3.29. a = (v 2 /R)
pcos .
p
p
1.3.30. t = (V /g) 9 sin2 8 sin > 8/9; t = 0 sin < 8/9.
1.4.

1.4.1. v1
v2 . , ,
.

1.4.2. . .
1.4.3. , , A.

1.4.4. . .

1.4.5. .

: tg = u/v.
.
u
=
10
3 /.

1.4.6. v = p
v 3.
2L v 2 u2 sin2
1.4.7 . t =
. .
v 2 u2
1.4.8. ) v = 2(v + u). ) v = 2(v w). (
.)
p

1.4.9. ) u =v. ) u = v 2 + 4vw cos + 4w2 . ) u = v 2 + 4vw cos cos + 4w2 cos2 .
v 2 + u2
1.4.10. =
.
2(R
p r)
2
2
1.4.11. t = 2 u /g + 2h/g.
1.4.12. vx = v 2u;
vy = (2n 1)Lg/(v u).
1.4.13. n = (v1 + v2 )/(2R).
1.4.14 . sin =
u/v.
1.4.15 . u = v 3.
1.4.16. , ,
, . .
284

p
1.4.17.
1 + v 2 /u2 . .

1.4.18 . = 60 , l = 200 3 345 .


1.5.

1.5.1.
1.5.2.
1.5.3.
1.5.4.
1.5.5.
1.5.6.
1.5.7.
1.5.8.

vB = 2vA .
v = R;
v = (R r).
u = v 3.
a = g ctg .
. .
(2,8; 3,1).

. uAB = v/ 2. . u1 = u2 v 2 .
2
2
. .; a = (v /R )r; r = (R + r)2 /r, r = (R r)2 /r.
vR
v
1.5.9 . u =
;=
; cos > r/R, cos < r/R.
R cos r
R cos r
1.5.10. .
1.5.11. . . . 4 .
1.5.12. a = 4 2 R.
1.5.13. u = v cos .
1.5.14 . q
t = a/v.

2 t/
1.5.15. . .; vB = 2vA

2 t2 .
L2 + vA

1.5.16. u = v 2 t/ L2 v 2 t2 .
1.5.17. = (v sin2 )/H.
2
1.5.18 . 0 = /2
sin (/2).
1.5.19. v = uR/ R2 h2 .
1.5.20. d = (R2 r 2 )/(vt).

2.
2.1.
F = 2ml/t2 = 0,16 .
F = me v 2 y/(lL).

a = g(T2 T1 )/(T4 T3 ).
T = F (1 x/l).
t = T (m1 + m2 )/[(2m1 + m2 )].
m = m0 [(t/t0 )2 1].
m2 /m1 = (F2p F )/(F F1 ).
t = nt(1 + 1 1/n ); t 10 .
m1 m2
2m1 m2
2.1.11. a1 = a2 = g
, T1 =
g, T2 = 2T1 .
m1 + m2
m1 + m2
m1 .
2.1.2.
2.1.3.
2.1.4.
2.1.5.
2.1.6.
2.1.7.
2.1.8.
2.1.9.

285

2.1.12. a = 3,5 /2 , T 1,1 103 .


2.1.13. 3g, .
F m1
F
F m1
2.1.14. x =
; a1 =
, a2 =
.
k(m1 + m2 )
m2 + m1
m2 (m1 + m2 )
2.1.15. x = ma/(k1 + k2 ); F1 = k1 x , F2 = k2 x .
2.1.16. x = F (k1 + k2 )/(k1 k2 ).
2.1.17. F = m0 g/; a = g(m m0 )/m.
2.1.18. . .
2.1.20. . .; F = mg sin tg 6 ; F = mg cos tg > .

2.1.21. a = g( cos sin ).


2.1.22. t = 2v sin /[g(sin2 2 cos2 )].
2.1.23. a = (F/m)(cos
+ sin
) g, , a = 0.

cos

6 tg sin ; a = 0 > tg sin .


2.1.24. a = g sin
sin(/2)
2
2
|m2 m1 |g F
|m2 m1 |g > F .
2.1.25. a = 0 |m2 m1 |g 6 F ; |a| =
m1 + m2
2.1.26 . = + arctg .
m2 g
1 2
, = arctg m1 > m2
;
2.1.27 . T = p
1 + 2
1s+ 2
m21 + m22
m2 m1
1 2
, = arctg
m1 < m2
.
T = g
2
(m1 + m2 )
1 + 2

2.1.28. 10 .
2.1.29. 0,4; l 50 .
s
2lm2
F0 m1 g
m1
2.1.30 . . F > (m2 + m1 )g; t =
. . a1 =
, a2 = g
.
F0 (m2 + m1 )g
m1
m2
2m1 g(m1 + m2 )
F
F0 a1 = a1 = a2 =
;
2.1.31. F 6
m2 + 2m1
2(m1 + m2 )
F m1 g
m1 g
F > F0 a2 =
, a1 = a1 = a2 =
.
m2
m2 + 2m1
2.1.32. F = mg cos (sin cos ) 6 tg ; F = 0 > tg .
2.1.34. 0,7 /.
p
2.1.35 . v = u (/)(mg F ) u2 > (mg F ), v = 0.

2.1.36 . F = mv.
2.1.37. ; v 5,5 /.
2.1.38. v1 0,25 /; v2 0,01 /.
2.1.39 . Rv 4 104 2 /.
v p 2
2.1.40. d =
v + u2 .
2g
2.1.41. .
q
2 F 2.
2.1.43 . v = RF/ F
286

2.1.44 . v = u tg /
2.1.45 . u = v/2.
2.1.46. F = 2T .

p
2 tg2 .

FM
F
F (M + 4m)
; a2 =
; a3 =
.
2m(M + 2m)
2m(M + 2m)
M + 2m
m2 g tg(/2)
m2 g
2.1.48. a1 =
; a2 =
.
m2 + 2m1 tg2 (/2)
m2 + 2m1 tg2 (/2)
2.1.49 . a = g tg ; m = m0 sin /(1 sin )2 .
2.1.50. M = m tg /(tg tg ).
2.1.51 . a = g sin(/2).
2.1.52. m2 = m1 a1 /a2 .
2.1.53. N1 = mg/2, N2 = 3mg/2.
2.1.54. F = me v 2 /r. ,
; .
2.1.55. T = 2mv 2 /l.
2.1.56. T = M 2 l; Tx = M 2 l + m 2 (l2 x2 )/(2l).

2.1.57. cos = g/( 2 R) g/( 2 R) < 1; = 0


g/( 2 R)s
> 1.
2.1.47 . 13: a1 =

g tg
.
l(sin + sin )
2
2.1.59. l = (1 m /k)R.
p
p
2.1.60 . R = R0 /(1 m 2 /4 2 k) < 2 k/m; > 2 k/m .
2.1.61 . = mg/(2T m 2 R).
v2
.
2.1.62. = arctg
Rg
2 /R2 2 )1/4 < g/R.
2.1.63. 1 = 0 > g/R; 1 = (2 gs

u
+ tg
=
.
2.1.64. v = gR; = arctg ;
v
(1 tg )
s
Rg sin (tg + )
2.1.66 . v =
.
tg 1
q
2.1.67 . > g 2/R.
2.1.58. =

2.2. .
2.2.1. u = 5v/2.
2.2.2. F = F/3.
2.2.3. t = 2p sin (/2)/F ; = ( + )/2 .
2.2.4 . m = F t2 /(16L). .
2.2.5 . t = mv(sin cos )/[(m + M )g] tg > ; tg 6 .
2.2.7. m1 /m2 = (u2 u1 )/(v1 v2 ). .
2.2.9 . u1 = F0 t0 /m1 ; u2 = v F0 t0 /m2 .
2.2.10. m = m0 /3.
2.2.11. u1 = u2 = 0,2v.
2.2.12. 4L .
2.2.13. S/L = 35/36.
m1 p 2
2.2.14. w =
u + v2 .
m2
q
2.2.15. p = p21 + 2p1 p2 cos + p22 .
q
m21 v12 + m22 v22 + m33 v32
2.2.16. V =
.
m1 + m2 + m3
287

2.2.17. l1 = lm2 /(m1 + m2 ); l2 = lm1 /(m1 + m2 ).


2.2.18. 2 , 2m.
2.2.19. , .
:
R1 = Rm2 /(m1 + m2 ),

R2 = Rm1 /(m1 + m2 ).

2.2.20. l = L 2/4 , L ; ; , ,
l = dr2 /(R2 r2 ) .
2.2.21. u = Svl/m.
2.2.22 . v = uV (0 )/(V + 0 V0 ).
p
2.2.23. T = 2 2R/3g.
m1 m2 v 2
2.2.24 . F =
.
(m + m2 )l
s 1
2m1 m2 (T1 T2 )
m2 T1 + m1 T2
2.2.25 . =
; m=
.
Lm1 m2
m1 T2 m2 T1
m
m
1 2
2.2.26 . T12 =
l 2 , m1 m2 ;
m1 + m2 + m3
.
2.2.27. F = mg V a.
2.2.28 . F = SLa.
2.2.29 . u/4 .
2.2.30. n = (m2 g tg )/(m1 v).
2.2.31. F = N mg. .
(M + N m)2
2.2.32 . H = h
.
N m(N m + 2M )

2
2.2.33 . F = mv /R; p = F/S = N mv 2 /3.
2N

M
(v2 v1 ).
2.2.34. vN =
M +m
2.2.35. F = Su2 .
2.2.36. = M g/u; 0 = M (g + a)/u.
2.2.37. F = 2 (u v) + 1 u.
2.2.38. v = Su/(S + k).
p
2.2.39. v = F ( 0 )/(r2 0 ).

2.2.40. . .
2.2.41. F = 3mg(1 x/l).
2.2.42. F =
m(v 2 + gl)/l.
2.2.43. v = gh.
p
2.2.44 . N = 2(F v 2 ) cos
; v > F/.
2
n
2.2.45. K = k .
2.2.46 . u = v ln n.
2.2.47 . m 5,5 105 ; 7,4 .
2.3. . .
2.3.1. m = 2F l/v 2 .
2.3.2. F = m(v22 v12 )/(2l); F > 0,
, F < 0, .
2.3.3. F p
2,5 106 .
2.3.5. v = F0 (l1 + 2l2 + l3 )/m.
2.3.6. v > 2gL.
2.3.7. A 0,8 .
288

2.3.8. x = v

q
p
m/k; x0 = x20 + mv 2 /k.

2.3.9. E = F 2 /(8k).
2.3.10. , , , . , , .
2.3.11. K = mgl cos ; K 0 = mgl(cos sin ).
2.3.12. h = v 2 /[2g(1 ctg )].
p
2.3.13. v = 4gh 2A/m.
2.3.14. A = mgl.
2.3.15. A = mgl/2.
p
2.3.16. v = r g/l.
2.3.17. n = mv 2 /(4F R cos ).
p
2.3.18. v = 2 (l h)T /m.
2.3.19.
p.
2.3.20. sin = v sin / v 2 + 2gh.
sin
mv 2
cos2 .
2.3.21. sin = p
F l >
2
2
1 2F l/mv
1
2.3.22. . . = arctg .
3
T 3mg
.
2.3.23. x = l
T mg
2.3.24. F = 5mg ; F = 6mg .
R
2.3.25. L =
.
2(tg )
2.3.26. h = 2R/3
2.3.27. h = 2,5R.
2Rmg p 2 2
2.3.28 . F =
4 R + h2 + 16 2 H 2 .
4 2 R2 + h2

2
2.3.29 . F = mg(1 1/k ).
2.3.30. F = mg cos (3 sin 2) sin > 2/3; F = 0 sin 6 2/3.

2.3.31. v = 2gl.

2.3.32 . A = 2mv 2 .
2.3.33 . K = K0 e2 .
2.3.35. F = l/(m2 ).
2.3.36 . A = x2 /(2m2 ) px/(m).
2.3.37.
U = F0 x2 /(2x0 ) |x| 6 x0 ; U = F0 (|x| x0 /2) |x| > x0 . :
p
|x| 6 2Kx0 /F0 K 6 F0 x0 /2; |x| 6 (K/F0 + x0 /2) K > F0 x0 /2.
2.3.38. K = kqQ/r 2 ; qQ > 0 , qQ < 0 .
2.3.39. .
p
V
2.3.40. E > 0 r > l (1 + 1 + E/V ); E < 0 r
E
p
V
r1,2 = l (1 1 + E/V ).
E
2.3.41. h = 2mg/k.
p
2.3.42. h = 2mg/k; v = g m/k.
2.3.43. H1 = 3h/2; H2 = 4h/3.
mgx0
2.3.44. k = q
.
2
2( l + x20 l)2
p
2.3.45. F = mg(1 + 1 + 2k(h l)/(mg) ).
2.3.46. F = (m1 + m2 )g.
p
2.3.47 . x = (m/k)(g a); x = (m/k)(g + 2ga a2 ).
19

289

2.3.48 . F = g(m1 + m2 /2).


2.3.49. m = m0 /2.
2.4. . .
2.4.1. . .
2.4.2. K = m1 v 2 /2 F l.
2.4.3 . K1 = k(x1 + x2 )x1 /2; K2 = k(x1 + x2 )x2 /2.
2.4.4 . A1 = mu2 /2 mv 2 /2; A2 = mu2 .
1); = 60 .
2.4.5. A = 2F r(2 sin
2
2.4.6 . .
p
2.4.7. x = v m/(3k).

2.4.8. v 0 = 2 v.
2.4.9 . v = gl.
2.4.10 . x = 4l/3.
s
s
2M gh
2M gh
+ 2g(H h).
;
v
=
2.4.11 . vm = tg
M
M + m tg2
M + m tg2
p
2.4.12 . v = (4/3) gR/3.
2.4.13 . F = 7mg/9.
2.4.14 . h 0,25 .
2.4.15 . K 0 = 0,01K.
p
2.4.16. vx = (l l0 ) k/(2m) cos ; x = (l l0 ) sin .
2.4.17. . .
, .
2.4.18. = v 2 /(2gl).
p
2.4.19. v = 2h(g T /m); K = mgh, E = T h.

2.4.20 . x = Lm2 /(M 2 m2 ).


2.4.21. l = l0 ; l = l0 + F/k.


q
mg cos
2.4.22 . x = mg cos /k 6 tg ; x =
1 + 1 2(1 tg /)2

2k
tg 6 6 3 tg ; x = 2mg sin /k > 3 tg .
2.4.23. K = m(u + V )2 /2, u , V . muV .
p
2.4.24. K = F 2 /(2k); U = 2F 2 /k; v = F (m1 + m2 )/(km1 m2 ).
2.4.25. , .
2.4.26. W = F l.
2.4.27. W = F (l F t2 /2m).
2.4.28 . W = F 2 m22 /[k(m1 + m2 )2 ]; U = W/2; K = F l + F 2 m1 m2 /[k(m1 + m2 )2 ].
2.4.29. A = mu2 . .
2.4.30. W/A = /(tg + ).
m1 m2
2.4.31. W = W1 + W2 +
(V1 V2 )2 ; .
2(m1 + m2 )
2.4.33. Q = m(v 2 /2 gh).
2.4.34. Q = m1 gh(m1 m2 )/(m1 + m2 ).
p
p
2.4.35. Q = 2mgR(1 1 l2 /(4R2 ) ) 1 l2 /(4R2 ).
2.4.36. E 200 .
2.4.37. m 3 .
2.4.38. 8 .
s


N
2N
N
2.4.39 . v = gt t 6 t0
;
v
=
t
t > t0 .
2
2
2
2
m g
m
2m g
2.4.40 . N = m0 g(1 /0 ); m = m0 /2.
290

2.4.41 . m = n2 m0 /(2n1 ).
2.4.42 . v 20 /; = arcsin
2.4.43 . N = S(v R)2 R.
2.4.44. = 2v/(v + u).
2.4.45. N = mgu/2.

2
.
4

2.5.
2.5.1. m1 /m2 = 1; .
2.5.2 . = /2.
2.5.3. u1 /u = (k 1)/(k + 1); u2 /u = 2k/(k + 1).
2.5.4. (mn md /2),
,
.

2.5.5. m = m1 m2 .
q
2.5.6. cos = v1 v2 cos /(u1 v12 + v22 u21 ).
2.5.7. v10 = 2v v1 ; v20 = 2v v2 .
2.5.8.
v10 =

(m1 m2 )v1 + 2m2 v2


;
m1 + m2

v20 =

(m2 m1 )v2 + 2m1 v1


.
m1 + m2


r .r
m2 m1
m2 m3
.
; v3 = v
2.5.9 . v1 = v
m3 (m1 + m3 )
m1 (m1 + m3 )
m1 + m2
.
2.5.10. tg = tg
m2 m1

2.5.11. d = 2 2 R.
2.5.13. v1 = v cos v2 = sin ,
, . ,
.
2.5.14. t = tn tn1 = 2t.
2.5.15. t = (2R cos )/v.
sin2 ( + ) sin2
m1
=
; m1 , m2 2.5.16.
m2
sin2
.
2.5.17 . sin = m2 /m1 .
2.5.18 . u = 2m1 v cos /(m1 + m2 ).
2.5.19. m2 = m1 (p2 + p20 2pp0 cos )/(p20 p2 ).
p

v
2.5.20. u = v 2 + (v + u0 )2 ; = + arctg
.
2
u
0+v
p

2.5.21 . v = 2gh(1 + m2 /m1 ).


p
2m2
;
2.5.22. v1 = 0; v2 = v v > v0 2gh(1 + m2 /m1 ); v1 = v
m1 + m2
m2 m1
v < v0 . v1 , v2 .
v2 = v
m1 + m2
p
m2 p
2.5.23 . v1 =
2gRm1 /(m1 + m2 ); v2 = 2gRm1 /(m1 + m2 ); N = m2 g(3 + 2m2 /m1 ).
m
1
s
s
k(m1 + m2 )
m1 m2
k(m1 + m2 )
; u1 =
x
;
2.5.24. u = x
m1 m2
m1 + m2
m1 m2
s
k(m1 + m2 )
2m1
; u1 = u; u2 = 0 . .
u2 =
x
m1 m2
m1 + m2
291

m2 U
m1 U
; h2 =
.
m1 g(m1 + m2 )
m2 g(m1 + m2 )
1,5 .
K = 35,7 .
E = E(1 + me /m).
E s
27,2 .
s
2Em1
2Em2
.
v1 =
; v2 =
m2 (m1 + m2 )
m1 (m1 + m2 )

2.5.25. h1 =
2.5.26.
2.5.27.
2.5.28.
2.5.29.
2.5.30.

p21 m22 + p22 m21 2p1 p2 m1 m2 cos


.
2m1 m2 (m1 + m2 )
2.5.32. E = K sin 1 sin 2 .
2.5.33. E = 4,1 .
2.5.34 . cos = (p2 2mE)/(p2 + 2mE), 2mE < p2 ; = /2, 2mE > p2 .
2.5.35. h/h0 = [(m1 m2 )/(m1 + m2 )]2 .
2.5.36. Q/K =p
(3 m1 /m2 )/4.
p
2.5.37. Q1 = 2 Q2 m (v 2 Q2 /m ).
s
r
r
Fl
Fl
Fl

(1 + 1/n); un =
; vn
n .
2.5.38 . vn =
m
m(1 + 1/n)
m
2.5.39. tg = tg 2 tg > 2; = 0.
2.5.31. E =

2.6. .
2.6.2. a = K/R2 , R , K .
2.6.3. h 700 .
2.6.6. = r 2 a/(2M ) 5 1011 2 /2 ,
.
2.6.7. M 6 1024 .
2.6.8. M 2 1030 .
2.6.10. 0,3 .
2.6.11. 6 .
2.6.12. R = (m / 2 )1/3 .
2.6.13. T1 0,7 p
.
2.6.14 . T = 2 R3 /(m1 + m2 ).
2.6.15. m/m = = r 3 /T 2 .
2.6.16. 2 = 3m/l3 .
2.6.17. v1 7,9 /; v2 1,7 /. T1 = 84 ;
T2 105p.
.
2.6.18. p = 2m0 m/R sin
2
2.6.19. F = 3mv 2 /(4R).
2.6.20. N 9 103 .

2.6.21. . R 42 103 . . , 60-


.
M mh
mgh U
h
2.6.22. U =
;
= .
R(R + h)
U
R
2.6.23 . v =q
4,6 /.
2.6.24. u = v 2 v02 .
2.6.25. 10 .
2.6.26. v1 11,2
/; v2 2,4 /.
2.6.27. v = ( 2 1)v.
2.6.28. h = 59 .
2.6.29. p
= 3H 2 /(8).
2.6.30 . v = 3m/R.
2.6.31. v 42 /.

292

2.6.32 . v 16,7 /.
2.6.33 . v = 29 /.
2.6.34. U = 2K.
2.6.36 . S = (1/2)vrt sin .
2.6.37. /
p45.
2 ).
2.6.38 . = R  1 + 2M/(Rv

2M
r
2.6.39 . V = v
1 ; R=
, M .
rv 2
2M/rv 2 1

2.6.40 . E = M m/(r + r ).
2.6.41 . R1 /R2 = 2u2 /v 2 .
2.6.42 . dv = M d/(v r ). dv .
2.6.43 . ( -, ) , ;
. : up = (v V sin )r , r = p/(1 sin ), = V /u. < 1
, = 1 , > 1 .
pu
u
pu
, r =
; = arcsin .
2.6.44 . V < u; r =
u+V
uV
V
. V ,

2.6.45

V0 = u2 V 2 .
1
pu2
a = (r + r ) = 2
,
2
u V2
a = M/V02 ( u2 /p = M/p2 , pu2 = M ).
p
V0 = M/a.

dS
1
1 p
= bV0 = b M/a. (.
dt
2
2
2.6.45.) T = 2ab/(bV0 ) = 2a3/2 / M .
, 2.6.45. R = a/k2 = b2 /a.
s
r
v2
v2 a
M
M b2
dS
1
1
M
= 2 = 2 vr =
,
= vr = b
.
R
b
r
a
dt
2
2
a

2.6.46 .

T = 2

ab
a3/2
= 2
.
dS/dt
M

2.6.47. 1910p
.
2.6.48 . t = R/g [(1 + Rc /R)/2]3/2 .
2.6.49 . t 65 .
2.6.50 . v 70 /.
M m(R23 R13 )
2.6.51. F =
.
(R1 + R2 )R12 R22
m2
mM (3R2 r + r 3 )
2.6.52 . N =

;
4r 2
R(R2 r2 )2

12
2.6.53 . 1,8 10 .
19

R=

12 R0 .

293

2.7.
2.7.1. K2 /K1 = 32.
2.7.2. K = mR2 2 /2. .
2.7.3. M = mR2 /t; M = mR2 2 /(4N ).
2.7.4. t = R/(g), n = 2 R/(4g).
2.7.5 . J = m1 r12 + m2 r22 .
2.7.6. n = 2 R(1 + 2 )/[4g(1 + )].
2.7.7. n = 2 R(1 + 2 )/[8g(1 + )].
2.7.9 . w = |m1 m2 |gR/(J + m1 R2 + m2 R2 ).
2.7.10. P1 = mg/2 Jw/l; P2 = mg/2 + Jw/l.
2.7.11. a = F/(m1 + m2 ); w = F/(m2 R).
2.7.12. a = (1/2)g sin . F = (1/2)mg sin .
2.7.13 . T = p
(1/7)mg sin .
2.7.14 . v = gl(sin 2 cos ).
2.7.15. a = 2m2 g/(2m2 + m1 ).
2.7.16. J = mr2 [gt2 /(2h) 1].
(m1 R1 m2 R2 )R2
(m1 R1 m2 R2 )R1
, a2 = g
;
2.7.17. a1 = g
J + mR12 + m2 R22
J + m1 R12 + m2 R22
J + m2 R2 (R2 + R1 )
J + m1 R1 (R2 + R1 )
T1 = m1 g
T2 = m2 g
.
J + m1 R12 + m2 R22
J + m1 R12 + m2 R22
g
1
mg
2.7.18. a =
; T =
.
1 + J/mr 2
2 1 + mr2 /J

2.7.19 . a = g/2.
2.7.20 . cos > r/R.

2.7.21 . . . t = 0 R/(2g).
Q/E = 1/2.
2.7.22 . t = v0 (3g). Q/E = 1/3.
2.7.23 . t = v/(g).
2.7.24. > 3v/R.
2.7.25. 1 = 3 = /3; 2 = /3.
2.7.26. = 60 . .
2.7.27 . N = 4m1 m2 g/(m1 + m2 ).
2.7.28 . N = mgl2 /(l2 + 3a2 ).


2g(m1 m2 )
2g(m1 m2 )
< 1;
2.7.29. cos = 2
2
l(m1 + m2 )
l(m1 + m2 )
= 0 .
J1 1 + J2 2
J1 J2 (2 1 )2
2.7.30. =
. Q=
.
J1 + J2
2(J1 + J2 )

2.7.31 . = v/(2R).
2.7.32 . 10 = (31 2 )/4; 20 = (32 1 )/4.
2.7.33. u m2 v/m1 ; = 2m2 vh/(m1 R2 ).
2.7.34. = 2m2 vr/(m1 R2 + 2m2 r2 ).
2.7.35. = mR2 /J. (1 + mR2 /J) .
2.7.36. n = 33/8 1 .
2.7.37. . - .
2.7.38 . m 4 1016 .
2.7.39 . .
.
. ,
.

2.7.40 . v = 3gL.
1
2.7.41. Q = /10 mv 2 .
294

3m22 v 2
.
gl(4m1 + 3m2 )(m1 + m2 )

2.7.43 . 2l/3 .
2.7.44. F 0 = F (mRx/J 1). x = J/(mR) F 0 = 0.
2.7.45. (v1 v2 )/2,
(v1 +v2 )/l. l/2(v1 +
v2 ) ; ,
.
2

3m2
2.7.46 . h = H
.
m1 + 6m2

2.7.47 . M = (u R)R.
2.7.48 . N = (u R)R. = u/R M/(R2 ).
2.7.42. cos = 1

2.8.
2.8.1. T = 98 , F = 138 .
2.8.2. F = 0,98 .
2.8.3. h 700 .
2.8.4. 120 .
2.8.5. m2 = m1 sin / sin(l/R ).
2.8.6. T 2,6 ; = arctg(3 3 ).
2.8.7. x = 5F/k.
2.8.8. l0 = 2l2 l1 .
2.8.9. T = mg/(2 tg ); T 0 = mg/(2 sin ).
2.8.10. FA = mg sin / sin( ); FB = mg sin / sin( ).
2.8.11. FA = mg tg ; FB = mg cos 2/ cos .
2.8.12. = tg( /2).
p
2.8.13. d = d0 + 2R(1 1/ 1 + 2 ).

2.8.14 . tg =(1 2 )/(1 + 1 2 ).


2.8.15. = 1/ 3.
2.8.16 . fn = F (f /F )n .
2.8.17 . F = F0 e .
2.8.18. . F1 = F2 = 98 ; . F1 = 24,5 , F2 = 171,5 .
2.8.19. m 6 7,5 .

2.8.20. m = m1 m2 .
2.8.21. m = (h/L)m0 tg .
2.8.22. m = (M + 2m)r/(L r), .
2.8.23. = arctg (1/3).
p
2.8.24. T = mgL/2h; P = mg 1 + (L/2h)2 .

2.8.25 . Tn = (2n 1)mg/ 3.


2.8.26. P = (1/4)mg ctg .
2.8.27. > 1/3. p
2.8.28. l < L < l 1 + 2 .
2.8.29. 6 arctg 2.
2.8.30. > /3.

2.8.31 . cos = ctg / 3, > tg .


2.8.32. tg 6 1/.
mg p 2
1 2
5 4 + 1, tg =
= 1/2.
2.8.33 . F = mg/2, = 0 > 1/2; F =
2

2.8.34. tg > (1 1 2 )/(21 ).


2.8.35 . F 0 = F (l + h)/(l h).
R
p
2.8.36. sin =
.
(l + R) 1 + 2
2.8.37 . = vh/R2 .
2.8.38. .
295

2.8.39. F = mg;

F = mg/4, , .

2.8.40. m = M r/(R r).


2.8.41. T = 3mg.
2.8.42 . S = (N l/)(t2 t1 ) tg .

2.8.44. F = mg( 2 1).

3.
3.1.
3.1.1. f = 2F x/l.
3.1.2. F = kx.

U = F x2 /l.

v = x0

p
2F/(ml).

U = kx2 /2.

3.1.3. . k = mv02 /x20 .


3.1.5. F = mgx/l.
p
3.1.6. v = x0 g/l.
p
3.1.7. n = R/r.

. F = kx, U = kx2 /2, k = m(v0 /x0 )2 .

, .

U = mgx2 /(2l).

3.1.8 . m = 2qEx20 /(lv02 ).


mg x2
.
Rr 2


1
qQ
qQ
2qQ
qQ
3.1.10 . U =
+

x2 .
40 L + x
Lx
L
20 L3
3.1.11. m = 2(k cos2 )x/g.
2mg
R
6mg
x. . R0 = . F 0 = 0 x.
3.1.12. . F =
R
R
3
m p
3.1.13. v =
gR/2.
m
lp
g/h.
3.1.14 . = 0
L
r
r
g
g
3.1.15. v1 = x0 (1 + y0 /x0 ) , v2 = y0 (1 + x0 /y0 ) .
l
l
3.1.16. mg(1 + x20 /l2 ) > F > mg(1 x20 /2l2 ).
p
3.1.17. x0 = R 2/(3N ).
3.1.9. U =

l=v

p
0 L3 m/(qQ).

3.2.
3.2.1. . , F = m2 x.
T = 2/.
p , ,
. . = k/m, R = x0 .
p
3.2.2. T = 2 l/g.
3.2.3. .
p
3.2.4. , : T = 2 m/(k1 + k2 );
p
: T = 2 m(1/k1 + 1/k2 ). .
3.2.5. l = 24,4 .
p
3.2.6. T = 2 l/(g sin ).
296

3.2.7. . F = mg[(T0 /T )2 1].


. F = mg (T0 /T )4 1, cos = (T /T0 )2 ,
.
3.2.8. r 30 .
3.2.9. t1 = 2 ; t2 7 .
3.2.10. F = m 2 l/2.
p
3.2.11. = qQ/(0 ml3 ).
3.2.12. t = 42 .
3.2.13. t = 42
p .
3.2.14. = 2g/l.
3.2.15 . t = 22 .
1/4

l2
3.2.16 . T = 2
.
g 2 + a2 2ag cos
p
2
3.2.17 . T =
l/(R + l).
p
3.2.18. = k/m 2 .
p
3.2.19. = g(M l mx)/(M l2 + mx2 ).
3.2.20. 2 = g/l + k/4m. k/4m.
3.2.21. M = m(g/2 R 1).

2 = g/ R2 l2 , ). 2 = g R2 l2 /R2 .
3.2.22. )
k
p
3.2.23. = k/(2m).
p
3.2.24. = g/(2R).
p
3.2.25. = k/, = m1 m2 /(m1 + m2 ).

3.2.26. HD /H2 = 3/2.


p
3.2.27 . 2 /1 = 11/3.
p
3.2.28. T = 2 l(M + m)/(M g).
p
3 p
3.2.29. ) t =
m/(2k). ) t =
m/(2k).
2
2
2
3.2.30. I = I0 [(T /T0 ) 1].
p
3.2.31 . = 6k/m.
p
p
3.2.32 . 1 = M g/(ml), 2 = 21 = 2 M g/(ml).
p
3.2.33. T = 2 l/(2g).
p
3.2.34 . T = 2 H/g.
p
3.2.35. = g/H.
q
3.2.36 . = k/[m + lR4 /r 2 ].
3.2.37 . m 900 .
3.3.
3.3.1. v = A sin t, a = A 2 cos t, F = m 2 x = Am 2 cos t, k = m 2 .
3.3.2. ) x = 5 sin(3,13t). ) x = 5 cos(3,13t). , .
3.3.3. T = 0,06 .
3.3.4. t = /(4).
p

3.3.5. T = l/g (1 + 1/ 2 ).
p
p
3.3.6. T = ( R/g + r/g ).
p
3.3.7 . t = /2 l/g; .
p
3.3.8 . t = m/(2Rp).
p
3.3.9. l = (n + 1/2)v0 m/k, n .
297

l p
3.3.10.
g/R.
v0
p
3.3.11. T = (4/3) l/g.
3.3.12. l = A cos[(1 T /T0 )].


r
p
mg
3.3.13. t = + 2 arctg
m/k.
2k(H h)
p

v0
v0
p
l/(g) v =6 gl, t =
+ l/(g) arccos
v > gl,
3.3.14. t =
2
g
v
p
v 0 = v 2 gl.
3.3.15 . w = 2R/(A) A  R, w = 1/3 A = 2R. .
d p
3.3.16. u =
k/m, n .
2n
3.3.17. t = T /4 + /2.
 mg

mg
3.3.18. ) x =
(cos t 1). ) x =
+ l (cos t 1). x k
k
, .
s
s
mv
k
k
mv
t, x = p
sin
t.
cos
3.3.19. v =
m+M
m+M
m+M
k(M + m)
3.3.20.
.
, ,

p
v. T = 2 m/(2k).


m1
m2
m1
3.3.21 . v1 =
v 1+
cos t , v2 =
v(1 cos t).
m1 + m2
m1
m1 + m2
3.3.22. F = 2F ; = T /2.
s
F2
2A0 F

3.3.24 . A =
A20 + 2
cos t0 . t0 = (2n + 1), n ,
k
k
; t = 2n/ .
p
3.3.25 . x0 = u m/k.
p
3.3.26 . u > g m/k A =
p
mg/k, u A = u m/k.

3.3.27 . = kl/(4M gn).


3.3.28. BC = g(M + m)/(M 2 ).
3.3.29. F = m 2 x = m 2 A cos(t + ), m 2 A
t = (n )/, n .
3.3.30. 2 A > g ,
.
3.3.31. A = F/(m 2 ).
3.3.32. h = A + g/(2 2 ) + 2 A2 /(2g) 2 A > g.

3.3.33 . A = (g/ 2 ) 2 n2 + 1, n .
3.3.34 . A  1011 g = 0,8 /, ,
, . A < 1011
. v = g/(2) 1,57 106 /.
3.3.36 . u = v0 tg /(2).
3.4.
3.4.1.

p
p 1 = 2k1 /m 2 = 2k2 /m. k1 6= k2
.
298

3.4.2. . r, p
, , v = r, = k/m.
T = 2/.
3.4.3. . A v/. v/
A.
.
s 

q
v2
1
A2 + 2 (A2 + v 2 / 2 )2 4(xv/)2 .
2

3.4.4. 2 = /6.
3.4.5 . 2 = n, n ,
; 2 = /2+2n
. A 2 cos A 2 sin .
3.4.6. .
3.4.7. , ,
, . , , ,
. . 2/ .
3.4.8. Tx : Ty = 1 : 2, , Tx : Ty = 2 : 1.
3.4.9. Tx : Ty = p : q, p q , pTy = qTx
. Ty = Tx .
3.4.10. y : x = p : q = 3 : 4.
p
3.4.11 . = 2F/(M + m1 + m2 ), m1 /m2 = p/q, p q
.
k
3.4.12 . F = k[A2 cos(t+2 )A1 cos(t+1 )]. E = [A21 +A22 2A1 A2 cos(2 1 )].
2
k
E = [A21 +A22 2A1 A2 cos(2 1 )]. 2 1 =
4
, 2 1 = 0 .



2 + 1
k
2 1
t sin
t . E = (A21 + A22 ).

3.4.13 . F = 2kA sin


2
2
4

3.4.14. N = (1/2)F
p 0 A sin . p
3.4.15. . 1 = 3k/m, 2 = k/m.
. v1 = v(cos 2 t + cos 1 t)/2, v2 = v(cos 2 t cos 1 t)/2; x1 = x2 = v(1/1 + 1/2 )/2;
x = v/1 .
. v1 = v(2 cos 2 t + cos 1 t), v2 = v(2 cos 2 t cos 1 t); x1 = x2 = v(2/1 + 1/2 );
x = 2v/1 .
3.4.16 . : )
v0 ; ) ,
p
(1)
: v0 = v1 cos 1 t, 1 = k/M ; )
v2 cos 2 t ,

p
2M
v2
cos 2 t, 2 = k(1/M + 2/m).
m





|v1 |
M |v2 |
1
1
1
x =
+ 1+2
= v
+
.
1
m
2
2
1
2
299

v(1 + 2 )[l(12 1 2 + 22 ) g]
g2
,L=
.
1 2 [l(12 + 22 ) 2g]
l(1 2 )2
2
2
3.4.18. k = m( 0 )/2.
3.4.19. A1,2 = (A B)/2; 1,2 = 2/ /T .
3.4.17 . x =

3.5.

3.5.1. . .

3.5.2. . .

3.5.3 . . . T0 ,

q
An = [v0 / + np/(m)]2 + x20 .
T0 /2,
q
An = [v0 / + p/(m)]2 + x20
An =

300

q
v02 / 2 + x20

n,

n,

= 2/T0 .

3.5.5. 63 .
3.5.6. , .
3.5.7. .
3.5.8. .
- , .
2
3.5.9. N = bv
 .

mv 2
dv
d kx2

3.5.10 .
+
= bv 2 , m
= kx bv.
dt
2
2
dt

3.5.11. . . : ;
. : , ,
pv ,
bv 2 T , .

3.5.14. 0 1.
3.5.15. n2 , n3 .
3.5.16. 2 . 2 /2 .

3.5.17. . .
3.5.19. = 102 1 , = 103 1 . 0
/0 .
3.5.20. . 102 1 . . 0 = /4.
3.5.21 . . Q = 0 /(2), n = Q/(2). . 50 Q = 108
1,5 Q = 109 .
p
2
3.5.22 . v =
. v 2p/m 2/  1;
m 1 exp(2/)
v 2p/(2m) 2/  1.
3.5.24. A = F0 /(m 2 ).
p
p
3.5.26. . A = F0 /[m( 2 02 )], 0 = k/m. . A = F0 /[m(02 2 )], 0 = k/m.

2
2
3.5.27 . A = F0 /[m( 0 )]. B ,
t = 0 x(0) = x0 , v(0) = v0 .
3.5.28 . x0 = F0 /[m(02 )], v0 = 0, B = 0.
3.5.29 . , ,
, .
3.5.30. , .
, , .
301

. , , . ,
, .
.



.

2F0
0
+ 0
t
sin
t
.

3.5.32 . . . x(t) =
sin
m(02 2 )
2
2



F0 t
+ 0
sin
t .
m( + 0 )
2
F0 t

3.5.34 . x(t)
sin 0 t.
2m0

3.5.35 . | 0 |  - et ,
0 . = 0 .
= 1/, e
.
3.5.33 . x(t)

. F = 2Am0 sin (0 t ). . A = F0 (2m0 ); 0 /(2) .


= F0 /(2x0 m).
0 = 550 1 , = 50 1 , Q = 5,5.
105 .
v = 0 /(2).
F0

3.5.41 . t v =
(1 cos t);
m
v = F0 /(m); V = 2F0 /(m)
F

(2n + 1) , n .
m 2
302
3.5.36.
3.5.37.
3.5.38.
3.5.39.
3.5.40.

F0
(cos t 1);
m
v = F0 /(m); v = 2F0 /(m)
.
F0
, t = /(2), v =
sin t; m
v = 0; v = F0 /(m)
F0 /(m 2 ) .
3.5.42 . ; v = F0 /(m) x. t = 0
vx = F0 /(m), vy = 0, r = F0 /(m 2 ).
, t = /, v =

3.6. .

3.6.1.
3.6.2.
3.6.3.
3.6.4.
3.6.5.

F/k; (N 1)F/k.
1014 .
k = ES/L, F = ES(L/L).
k = Ea.
. . l = 3 .

3.6.6. 108 0,5 108 .


3.6.7. F = 5 104 .
3.6.8. 1,2 104 .
3.6.9. l = mal/(2ES).
3.6.10. w = E2 /2 = 2 /(2E).
2 Ea4
.
3.6.11 . A =
6 l
3.6.12 . = k/(k + 2k0 ).
3.6.13 . = k/(k + 2k0 ).
3.6.14. . = 0,5.
3.6.15. = 3(1 2)/E.
3.6.16. 30 . 50 /3 .
2,5 106 /3 .
3.6.17. F ; ,
.

303


3.6.18. . . , 1, 2, 3: F1 = F0 b/L, F2 = F0 (b/L +
b/l), F3 = F0 b/l.

3.6.19. u = c.
3.6.20. . dp/dt = c2 .

. F = F0 ; c =

p
F0 /.

p
3.6.21. . = b/L, w = Eb2 /(2L2 ); u = c = cb/L. . c = E/.
p
3.6.22. . dp/dt = cuS = c2 S. . = E, c = E/.
3.6.23. 5 /. . . ,
.
3.6.24. 550, 1400 340 /.
3.6.25. c2 = (P P0 )/[0 ( 0 )].
3.6.26 . , ,
, .
, , .

3.6.27 . . .
u/c = h/h.
;

hcu = ghh. c = gh.


l
.  0 c = 0 l, 0 0,5 1014 .
3.6.28. c =
2 arcsin(/20 )
3.7.
3.7.1. p = cbS.
c l

;x=
l.
L

3.7.3. P (t0 r/c), r .


3.7.4. qp = cu(x0 ct).
3.7.5. F = 1400 .

0 = [1 + F/(SE)]. p = 0,5F , p0 = F ;
3.7.6. u = F/(S E),
= F/(SE);
2
W = 0,5F /(S E), W 0 = F 2 /(S E).
3.7.7. A = 12,5 103 , K/A = 0,25.
3.7.2. . qp = c2 .

304

. v =

q
q
c1 c2 F
, c1 = Fk /01 , c2 = Fk /02 .
c1 + c2 F k
p
3.7.9. F1,3 = (v 2 F )b/L F2 = 2(F v 2 )b/L. v F/ ,
,
.
p
, v F/
.
3.7.10. , . .
3.7.11. , , .
3.7.13. .
(sin = c/v).
3.7.14. 1 = , sin 2 = (c2 /c1 ) sin .
3.7.15. ,
.
3.7.16. sin 0 = c1 /c2 .
3.7.17. :

3.7.8. . .; u =

c2 sin 1
,
c1 + v sin 1
c1 c2 , v , 1
.
3.7.18. . , .

. . .
sin 2 =

3.7.19. .

3.7.20 . . ., , ; , .

3.7.21. = 0 /(1 v/c).


3.7.22. 1,2 = 0 (1 v/c); 3 = 0 [1 (v/c) cos ].
20

305

3.8.
3.8.1. (. . 3.8.1) ,
U = 2E. (. . 3.8.1)
K = 2E, .

3.8.2. = 0,5 103 .

3.8.3. . .

3.8.4. . . P = 2cA cos t. = 2c/.


. /4.
3.8.5. . . .
.

3.8.6. A = v0 /2. .
/4 (. .).
3.8.7.
().

3.8.8 . u = 2P/(c) = 250 /; l = c /2 = 1 .


c

1

1
L
3.8.9 . l =
L arcsin
=
1 arcsin
. l = L/2 0  , l = L/4
2

0
2

0
0 .
3.8.10. P = cu = 3,9 104 . , ,
. , . .
, , ,
. , .
= 2l/c = 4 104 .
306

3.8.11. vl = v, vL = v|1 2l/L|.


3.8.13 . v1 = 0, v2 = vl1 /l2 .

u
1 E1 2 E2 u
2 E1 1

3.8.14.
=
,
=
.
u
1 E1 + 2 E2 u
E1 1 + E2 2
3.8.15. D 41 c1 /2 c2 1,1 103 .
3.8.16. , , 0,25 0,41. (-), 2l/c, . - ,
.


1 c 1 2 c2 2
, L = 2lc1 /c2 .
3.8.17. n =
1 c 1 + 2 c 2

3.8.18 . L = 2lc1 /c2 . n = 1. .


3.8.19. l1 = 1,25 , l2 = 2,5 .
3.9. .
3.9.1. = c/ = 6,6 .
3.9.2. l = c/4 = 82,5 .
3.9.3. c = 2l/.
3.9.4. v1 = 6,8 /, v2 = 6,8 108 /, x1 = 0,11 , x2 = 1,1 1011 , P1 = 3 104 ,
P2 = 3 1012 .
3.9.5 . I > 3 /2 .
3.9.6 . F = 2L2 cv.  c/L
, .
3.9.7. E = 2R2 2 A2 c. .
3.9.8 . . :
u=


F0
x
cos t
2Sc
c

( x , F )
u/c.
. :


l
x
F0
cos t
cos
;
u=
Sc
2c
c
:
u=


F0
l
x
cos
cos t
Sc
2c
c

( x , F ). l
,
.
3.9.9 . l = (1/4 + n); l = (3/4 + n), = 2c/.
3.9.10. L = 2, c = L/4.
3.9.11 . . , /2.
0 S
F0 =
.
sin(2L/)
307

. . .; = 2nc/(2L), n , c = /(2) . .

3.9.12. n = n 2500 . 25 .
3.9.13. .
2
A0
. =
.
3.9.14 . A =
| sin(L/c)|
| sin(L/c)|
3.9.15. = c/(2L) = 8,25 .
3.9.16. , , .
.
3.9.17. 50, 250, 450 . .
(1)
(2)
3.9.18. 0 = 300 ; 0 = 150 .
3.9.19. . .
3.9.20. .
- ,
. . ,
.
3.9.21. F = 4l2 2 = 144 .
3.9.22. l/6 l/3 .
3.9.23. - .
, , ,
.
3.9.24 . ()
().
D = 4 c / c = 0,7 104
(. 3.8.15). 104 .
3.9.25 . , ,
. .
,
, . (
; .)
, , .
, c ( 330 ).
.
21 c1
P1 =
P2 ,
1 c 1 + 2 c 2
P2 .
150 .
308


3.9.26 . M =

mk
ctg

m
.
k

4.
4.1.

4.1.2. F1 = 2000 2 . F2 = 0.
4 F
4.1.3 . P = 2 .
3a
4.1.4. .
4.1.5. F = 2r2 P .
4.1.6. F = (R2 r 2 )P .
(R )2
4.1.7 . = 2
P.
R (R )2

4.1.8 . F1 , , F2 , .
d2 d2
4.1.9. l = 2 1 2 3 2 a .
d1 + d2 + d3
4.1.10. h = 727 .
4.1.11. F = 4392 ; F =
4314 ;
F = 4353 ;F = 78 .
4.1.12 . F = (1/12)ga2 (3 3 h 2 a) + (1/4)P a2 3.
4.1.13 . , 

:


1
h
cos + sin +
+ P a2 .
Fk = a3 ( 0 )g sin , F = a3 0 g
0
2
a
4.1.14. x = H (R2 /r 2 )(1 + a/r)(/0 1)h.
4.1.15. h = 85 .
4.1.16. h = 10,1 .
4.1.17 . m = R3 /3.


1 r2
4.1.19 . A = r 2 h + l 2 gl.
2 R

4.1.20 . Pr : , ,
, ,
Pr =

2
2 (R2 r 2 ),
3

P0 =

2
2 R2 .
3

4.1.21. .
4.1.22. = arctg .

4.1.23 . P (x) , m 2 y, y
, m :
P (x) = 2 [(R x)2 R2 /4]/2.
1 2 2

4.1.24. y =
x .
2 g

20
309

4.2. .
4.2.1. P = mg/S + P0 .
4.2.2. h = H( 1 )/(2 1 ).
4.2.3. H = (m 1 hS)/[S(2 1 )].

4.2.4 . , O,
, , ,

, .
r
a

> 6 (1 /0 ).
b
0
4.2.5. A = 34.
4.2.6. V = 147 3 .
4.2.7. = 1,5 /3 .
4.2.8. 01 /02 = 1 /2 .
4.2.9 . x = 4m/[(d21 + d22 )].
4.2.10. F = 0,8 103 .
4.2.11. F = (2/3)r 3 g(1 + 2r/l).
4.2.12. F = 1,2 102 .
3.
4.2.13. = 2/3 /

4.2.14. F = mg/ 3.
4.2.15 . . F = gR(H + L/2)2 . . F = gL(H + R)2 /2.
4.2.17. m = 520 .
4.2.18 . m = (4/3)(R2 + r2 )3/2 .
4.2.19 . m1 =a3 (6 + 5 tg + tg3 )/24; m2 = a3 (6 5 tg tg3 )/24.
4.2.20 . T = 3 mg/72.



1 h
r2
4.2.21. . Q = 1 . . Q = r2 ghH 1 +
1 2
.
2 H 0
R
3
4.2.22. Q = (4/3)R gH = 410 , .
4.2.23. A = 2,5 106 .




4.2.24 . . . [/3 ] = 1 +
1
.
2R 2H 2 + l
l

3
2
4.2.25 . F = (4/3)r (R r) .
p
4.2.26. = (g tg )/[R (l + r) sin ].
4.2.27. F (m1 m2 ) 2 R/2.
4.3.
4.3.1. 28,5; 27,0; 25,6 /. .
310

4.3.2. T = 2ghS.
4.3.3. N = V [gh + V 2 /(2S 2 )].
4.3.4. . - 1 2 ,
, ,

, A.
. F = v 2 S1 (1 S2 /S1 )2 /2.

4.3.5.
P = P0 + gx, P = P0 + g(x H).

4.3.6. F = 2 (P + pv 2 )S.
p
4.3.7. v = 2F S/[(S 2 s2 )].
4.3.8. x = 5l. "

2 #
mg
1
2
4.3.9. h =
v
.
2g
vSN
R2
0
g, P =
0 gh.
2
2
2
2
+ 0 r /(R r )
(R r 2 ) + 0 r2
4.3.11 .
2 .


2 . 2 2 .
4.3.12 . .
p
H/h , H/h .
p
(H/h)2 H/h = (H/h)5/2 .
4.3.10 . a =

4.3.13. , 2, 3 1 v,
,
h1 = h(1 + cos )/2, h2 = h(1 cos )/2.

4.3.14 . ,
. ,
. OO0 ,

4.3.13. (). v1 = v tg , v2 =
2

v ctg .
2

311

4.3.15 . ; cos = (R2 r 2 )/(R2 + r 2 ).


4.3.16 . 4.3.15, ,
. v = 1 /.
p

4.3.17 . h = l cos(t g/l ). P = xg/2 . P = yg/2 .

4.3.18 . a = g(s/S)2 .
4.3.19. E = Ps
V.
2P

(R3 /r 3 1), .
4.3.20 . v =
3
4.3.21 . , v, ; v > 14 /.
4.4.

4.4.1. ,
dv
AA0 , F = dx
. , AA0 BB 0 , .
v0 /h, x v0 x/h, 0 < x < h, F = v0 /h.

P
P 3
x(h x), 0 < x < h; Q =
h .
2
12
3
h g
4.4.3 . . Q =
sin . . 8 108 .
3
4.4.4 . v = 2mg2 /(r 2 h).

4.4.5. )
dv
dv
xP
P x2 2xl .
=
,0<
dx
dx
2l
P
x < R. ). v =
(R2 x2 ). , , Q =
4l
R4 P 2 /(8l).
4.4.2. v =

312

4.4.6. t = T .
4.4.7 . t = 32l/(gd2 sin ).

4.4.8. ) ,
, x,
, ,
.
d
d
M
= M,
=
,
dx
dx
2x3




M
1
1
M
1
1
2 ,
2 .
.
2
2
4 x
R
4 r
R
4.4.9. F = P2 S2 P1 S1 v12 S1 (1 S1 /S2 ).
Mx = x 2x x

r < x < R.

4.5.
4.5.3.
4.5.4.
4.5.5.
4.5.6.
4.5.7.
4.5.8.
4.5.9.

r 0,5 .
F = 2(1 2 )l.
= k(2R l)/(2R).
. A 2V /. . n 4.
a = 2,1 .
= rgh/2.
0,2 /2 .

4.5.10 . , ( ), , ( ): Fx , mg ,
, F0 = gxl Fk = gx2 /2 , ,
. ,

Fk = gx2 /2 = cos , cos = 1 gx2 /(2).



q
Fx = F0 + mg + 2 sin = mg + gx(l + 2 /g x2 /4 ).
313

2
4.5.11. m = 0,55
p / .
4.5.12. . h = 2(1 sin )/(g). . h = 3,9 .
p
/(g). . x = 5,4 .
4.5.13. . x = 2 sin
s2
2 ( + .. )
4.5.14 . . x =
, 6 + .. ; x = 0, > +
( )g
.. . 2,5 2 .
4.5.15 .
, cos . , , ,
.
4.5.17. P = 2/R + g(h + R), P = 2/R + g(h R).
4.5.18. R = /(gx).
4.5.19. P = P0 + 2[1/R + 1/(R h)].
4.5.20. 3s
.
1 + 2
2

.
4.5.22 . h = 2r
2
1 + 2 + (1 2 )2
4.5.23. R = rR0 /(R0 r). = 120 .
4.5.24 . m = r2 (h + 2/Rg).
4.5.25. h = 0,14 .
4.5.26. A = 1,4 105 .
4.5.27 . v 2 /2+gh+/r ( , v ,
, r h ) .
h 2 .

4.6.
4.6.1. . .
4.6.2. h = 2/(gR); A = 4 2 /(g); U = 2 2 /(g). .
4.6.3. r = 1,5 .
4.6.4 . . V = r 3 /(4h). . V = 1,1 3 /.
4.6.5. = 0,4 %.
4.6.6. r2 = 1,5 , r4 = 1,5 .
4.6.7. x = 2/(gr), 0 < x < h 2/(gr); x = h x, h 2/(gr) < x <
h + 2/(gr); x0 = h + 2/(gr).
4.6.8. rx = 2r/ cos .
2p
4.6.9 . =
/(r).
l
4.6.10. x = 2h, l > h; x = l + h, l < h.
4.6.11 . t = 17 C.
4.6.12. , s
.
!
16
16
16
1

4.6.13 . x = H 1 1
,>
; x = H, <
.
2
gH 2
gH 2
dH 2

(cos 1 + cos 2 ).
4.6.14. x =
g
4.6.15. F = a(1 cos ).
4.6.16 . F = 2a 2 /(g2 ).
p
4.6.17 . h = l/(Sg), T = 2 h/g.

314

5.
5.1.
p
5.1.1. K 5,8 1021 , hv 2 i 1,5 104 /.
5.1.2. .
5.1.3. p
m > 0,01 .
5.1.4. hx2 i 6,4 108 .
5.1.5 . r 7 . T = 100 d 4 .
5.1.6. , . . , . , ,
,
.

5.1.7. PHe /PH2 = 1/ 2.


5.1.8 . N1 /N2 = (2 /1 )(n1)/2 .
5.1.9 . 0 = n .
5.1.10. L2 /R2
.

5.1.11 . N1 = N T2 /( T2 + T1 ), N2 = N T1 /( T2 + T1 ). .
5.2.
5.2.1. . N = 1,3 1015 1 . . N1 = 1,3 1017 3 ; N2 = 1,3 1019 3 .
5.2.2. N1 = 6,2 104 3 , N2 = 1,2 104 3 .
5.2.3. ) N1 1016 3 , ) N2 2 1022 3 .
5.2.4. T = 21 .
5.2.5. n = 0,13.
5.2.6. n = 6 103 .
5.2.7 .
.



2t
5.2.8 . L = l
1+
.
t0
t0 + 2
5.2.9. v = 300 /.
5.2.10. v = h/(2).
1 v
1 v
5.2.11. . f 0 = f
. . f 0 =
f
.
l
l
kl
kl
5.2.12. f (v) = 1/v0 v0 6 v 6 2v0 , f (v) = 0 v.
f (v) v = F /m
.
r
F
r
F
r
Fl
, v = v +
+ v, n0 = n. . v = v 1 + 2
,
5.2.13 . . v = v +
2
m
m
mv
s
r
r
r
.
.
Fl
Fl
Fl
Fl
.
, n0 = n
1+2
+ v
1+2
v 1+
v = (v + v) 1 + 2
mv 2
mv 2
mv 2
m(v + v)2

5.2.14 . . exp(2F l/m) . . = 0 exp(mgh/kT ); .


5.2.15. m = 1024 , r = 109 .
5.2.16. h1 111 , h2 123 .
5.2.17 . . n n0 exp (q/kT ). . n = 1015 3 .
5.3. .
5.3.1. d 0,3 .
5.3.2. l 60 .
5.3.3. 1 6 1028 1 3 ; 2 3 1028 1 3 .
5.3.4. 1,5 .
5.3.5. l1 1 [4R12 n1 + (R1 + R2 )2 n2 ]1 ; l2 1 [4R22 n2 + (R1 + R2 )2 n1 ]1 .
5.3.6 . t 30.
5.3.7 . n = 2 2 rAB /(rA + rB ).
315


5.3.8. . AB, h, ,
p
p
W1 vz nh+ /2, vz ,
(
vz2 ) =
kT /m,
nh+ = (h + )
p h + . , , W2 = vz nh /2 kT /m (h )/2.
p
W = W1 W2 kT /m. . D 12 /.

5.3.9. D = nD1 D2 /(n1 D2 + n2 D1 ).


5.3.10 . t L/D; m = DS/L.
p
5.3.11. . 5.3.8: W nk kT0 /m. .
. 6,2 .
5.3.12. W 12 , t 2 . - .
1
2
5.3.13 . =


 p
1/2 2 +
 p
1/2 2 .

1
2
1 + 4
1 + 1 1 /2
1+
1
+

/
2
1
4

5.3.14. t0 = nt.
5.4. .

5.4.1. 1024 1 2 , p/t 10 .
5.4.2. 1 k/2 .
5.4.3. F r2 nmv 2 .
p
5.4.5. F 4r2 P v /(RT ), R .
5.4.6. F = P s/2.
p
5.4.7. F P Sv /(RT ).
5.4.8. ,
. = (1 /P1 )P .
5.4.9. 0 = (r1 /r2 )2 .
5.4.10 . ,
. .
.
5.4.11 . F 102 .
5.4.12. v 1 /.
5.4.13 . P F T1 /[S(T2 T1 )].
p
5.4.14 . P0 = P T0 /T .

5.4.15 . P 0 = P (1 + 2 ) 25/4 , T 0 = T 2.
p
5.4.16 . w = 1,5knT 3kT /.

5.4.17. 14 .
5.4.18. m 0,1 .
5.4.19 . r (W1 /W2 n)1/2 .
5.4.20. : N .
316

5.5.
5.5.1. .
5.5.2. V = (P2 V2 P1 V1 )/(P2 P1 ).
5.5.3. m = m(k 1)n/(n 1).
5.5.4 . P = P0 + mgh/(2r2 L).
5.5.5. V = 885 .
5.5.6. x = L(1 + gL/2P )/2.
5.5.7. P = 1,166 .
5.5.8. t = 140 C.
5.5.9. , ,
.
5.5.10 . T = 9T0 /8.
5.5.11. n = (P P0 )V0 /P0 V .
ln (P0 /P )
5.5.12. n =
.
ln (1 + V /V0 )
5.5.13. .
5.5.14. - . .
5.5.15 . P = 137 .
2V0 + S(l + 2x)
.
5.5.16. T = T0
2V0 + S(l 2x)
5.5.17. P = 1146 .
5.5.18. V /V = 1,9.
s



1
P0
P0 2
5.5.19. x =
l+H +

l+H +
4lH .
2
g
g


gh0
5.5.20. x = (H0 H) 1
.
P0 + gH


h
2gh
h
P0
1+
. . H = +
.
5.5.21. . H =
2P0 + gh
2
g
2
33 5
5.5.22. x =
a.
2

5.5.23 . P = P0 + gH.
5.5.24 . P1 = 0,17 , P2 = 0,18 .
5.5.25. N2 O3 .
5.5.26 . m = 210 /3 .
5.5.27. FHe /FH2 = 25/27.
5.5.28. M = 13,5m
5.5.29. r = 15 .
5.5.30. m = P0 V (T T0 )/(RT T0 ).
5.5.31. V = 15 3 .
5.5.32. N = 0,28.
(P0 r + 4)
5.5.33. T = T0
,
P0 r 3RT0
R .
P r13 + r23 r3
.
5.5.34. =
4 r2 r12 r22
p
5.5.35. T = 2 ml/(2P0 S).

5.5.36. V3 = V22 /V1 .


5.6. .
5.6.1. H2 = N2 = (5/2)kT , H2 O = CH4 = 3kT .
5.6.2. U1 = 0,25 , U2 = 0,2 .
5.6.3. .
317

P1 V1 + P2 V2
P1 V1 + P2 V2
, T = T1 T2
.
V1 + V2
P1 V1 T2 + P2 V2 T1
5.6.5. .
s
s
3P0 V0 m2
3P0 V0 m1
5.6.6. v1
, v2
.
m1 (m1 + m2 )
m2 (m1 + m2 )
5.6.7. T = T0 + 2mv 2 /(3R), R .
5.6.8. v 10 /.
5.6.9. .
5.6.10. .
5.6.12. ) A = P V ; ) A = 3P V /2.
5.6.13. A = 460 .
5.6.14. Q = (c/R)(P2 V2 P1 V1 ) + P2 (V2 V1 ), R .
5.6.15. A = 2,6 .
5.6.16. A = 240 .
5.6.17. Q 7,94
.A 2,27 .
5.6.18. A R( T3 T1 )2 .



3/2
3P0 V0
M u2
5.6.19. T = T0 1 +
, V = V0
.
2
3P0 V0
3P0 V0 + M u
5.6.20. A = 7R(T1 T2 )/2.
5.6.21. t 10 C.
s
h
PS
PS
PS i
5.6.22 . u = 2gH 1
+
ln
;
Mg
Mg
Mg
s
h
5  P S 3/5
3 PS i
u = 2gH 1
.
+
2 Mg
2 Mg

5.6.23 . V1 = V0 V2 , A = 5P0 V0 [(V0 /V2 )1/5 1].


A /2.
5.6.24. Q = 450 . U = 321 .
5.6.25. .
5.6.26 . T = T0 + Q/c Q 6 Q1 = cT0 F/P0S;

Q + cT0 + RT0 (1 + F/2P0 S)
F
T =
1+
Q > Q1 .
c(1 + F/P0 S) + R(1 + F/2P0 S)
P0 S
2
5.6.27. Q = 10gSh .
5.6.28 . c = (1/(1 n) + 3/2)R, n = 5/3; n = 1.
5.6.29. .
5.6.30 . c = 2P0 V0 /T0 .
5.6.31 . x = 3H(1 P S/M g)/5.
5.6.4. P =

5.7.
p
5.7.1. v = 2cP T /.
p
5.7.2 . v = 7(k + 1)RT /(k1 + 2 ).

5.7.3 . . T 3150 . . v 3 /.
5.7.4. ) v 5,2p
/; ) v 5,7 /; v 7 /.
5.7.5. m = M g/ 2cP T / 3,8 /.
"
(
 (1)/ #)1/2
2RT1
P2
5.7.6 . v =
1
.
( 1)
P1
5.7.7 . T 120 , v 1370 /.
5.7.8 . T 193 . P
0,33 .

s

1
P
P 2
2( 2 1)q
0

5.7.9 . v = v
1 2 +
+ 2

.
1+
v
v
Sv 3
F = Sv(v 0 v), = P /(RT ).
318

5.8.
5.8.1. . t = /4. . t = /8. . t = /2N .
5.8.2. . p1 = 1/4, p2 = 1/2. . p = 1/2. . p2 = 3/8, p0 = 1/8.
5.8.3. . p = (1 V /V0 )N . . V = V0 (1 102/N ).
15
5.8.4 . p 1010 , V 1017 1018 3 .

5.8.5. .
. . :

i

1 h 
k
x
v0
v v 2;
tg
+ 1 =
,
0
0
2A B
2
4
2n
k n , ,
tg(/4 + ) 1 = k/n;

h1 2a/k,

h2 = 0.

. , tg(/4 + ) 1 , 0,03,
,
0,03 + 2/n, n , 0,03.
, ,
, . , .
p = S/a2 .
. p = V /a3 .

p
5.8.6 . . v 0 v 1 1/m2 ; tg = m, tg( + ) 1 = k/n, h1 = 2/k, h2 = 0.
2
3
. p = S/a . . p = V /a .
5.8.7. R/v; 0 H/R H  R, 0 R/H H  R 0 H R.
5.8.9. p = (V /V0 )N .
5.8.10. A = 200 .
5.8.12 . (1 V 2 /V02 )N .
5.8.13 . 104,810

22

5.8.14. . , ,
. . ,
.
319

, , .
S = N U/T + N k ln c,
N , c ,
. S . , c = exp (U/kT ).
5.8.15. .
5.8.16. . .
5.9.
5.9.2. S = 1,2 /.
5.9.3. S = 7 /.
5.9.4. , . S = (m/)R
(ln 2.  )
V2 T2 3/2
m
R ln
.
5.9.5. . S =

V1 T1
5.9.6 . S 20 /.
5.9.7 . S 60 /.
5.9.8. S = (P V /T ) ln 2.
Q1
Q2
5 (T1 T2 )2
5.9.9 . . S =
+
= R
, Q1 Q2 ,
T1
T2
2
T1 T2

.


P2 V2
P1 V1
V2
V1
R
3
+3
+2
+2
10 .
. S =
2
P1 V1
P2 V2
V1
V2
 2/3
V
2(T2 T1 ) ln(P2 /P1 )
1
5.9.10 . . = 1
. . =
.
V2
5(T2 T1 ) + 2T2 ln(P2 /P1 )
5.9.11. .
5.9.12. .
5.9.13. 10,8 %, = 30 %.
5.9.14.
Q /T + Q /T > 0,

Q Q = A,

= A/Q ,

T T , Q Q , , A
. , 6 (T T )/T ,
Q /T Q /T = 0, . . .
5.9.15. .
5.9.16. A 33 .
5.9.17. A 3 1016 , t 60 .
5.9.18 . A = C[T T0 T0 ln(T /T0 )].
C /(C1 +C2 ) C2 /(C1 +C2 )
5.9.19 . A = C1 T1 + C2 T2 (C1 + C2 )T1 1
T2
32 .
5.9.20. .
5.9.21. Q = A(1 )/.
5.9.22. N = 0,29 , N 0 = 0,11 .
5.9.23. m = 5 .
5.9.24 . N = 138 .
5.9.25 . A = 46 .
5.9.26. . ,
.
5.10.
5.10.1. t 1 .
5.10.2. .
5.10.3. , .
5.10.4. p 108 .
320

5.10.5. 13 % .
5.10.6. 100,5 30,5 0 C.
5.10.7. m = 98 /.
5.10.8 . x 0,11 .
5.10.9. . , 100 C. . .
5.10.11. ,
.
5.10.12. ,
.
5.10.13. .
5.10.14. v 8 /.
5.10.15 . 25 .
5.10.16. .
5.10.17. .
5.10.18. .
5.10.19. .
5.10.20. m = 11,7 .
5.10.21. P = 0,2 , A = 35 .
5.10.22. P = 0,37P0 .
5.10.23. v = mRT /[P0 (q + RT )]. A = mRT /(q + RT ).
5.10.24 . h 580 .
5.10.25. 5 % .
5.10.26. 6 % .
5.10.27. . n = exp(mgh/RT ) = exp(2m/r). . h = 15 .
5.10.28. t = 2H2 O P /(7RP ) = 23 C.
5.10.29. .
5.10.30. P = P0 (R/r)2 .
2
5.10.31 . P = 2P
0 (R/L) .
5.10.32. . m n . . P = 200P0 .
5.10.33. a = 1,0 /2 .
5.10.34. m1 = 1,7 /, m2 = 170 /.
5.10.35 . T 1720 .
5.11.
5.11.1. . 0,2 . . = 89 /2 .
5.11.2. T1 600 C, T2 2000 C.
5.11.3 . w = 7,56 1016 T 4 /3 .
5.11.5. . , , ,
.
. , ,
.

q .

5.11.6 . . T = T0 / 4 2. . T = 4 (T14 + T24 )/2.


p
5.11.7. . T = T0 (R/2L)2 . . = 1,7 /2 .
5.11.8. . T = 200, 70, 35 C. . 4 1026 . . T = 140 C.
5.11.9. T = 2,4 .
5.11.10. T = 20 C.
1 2
5.11.11 . =
S(T14 T24 ).
1 +
2 1 2
5.11.12. . T 0 = T / 4 2. . n = 32.
T0
5.11.13 . T = p
.
4
6,5 + 4R/r
s



h 4
4
5.11.14 . T = T14 T24 + T1 +
(T14 T24 )
.
2

21

321


r
S
1
1
S
. v =
.

.
2R2 mc
R1
R2
mc
5.11.17. . .
5.11.16. . a =

. r 1 .

6.
6.1. .
6.1.1. . F = 1,8 104 . . F = 2,3 108 . 4,17 1042 .
6.1.2. q 1,05 105 3,16 104 .
6.1.3. . E = 1 / = 3,3 105 . . E = 3 105 / = 10 .
6.1.4. 1 E1 = 9 1010 / = 3 106 ; 20
E2 = 2,25 108 / = 7,5 103 .
0,001 F1 = 9 107 , F2 = 2,25 105 ;
1000 F1 = 3 109 , F2 = 7,5 106 .
6.1.5. F = 2,56 109 .
6.1.6. q = 3,5 103 .
q1 (4q2 + q3 )
q3 (4q2 + q1 )
6.1.7. T12 =
, T23 =
.
160 l2
160 l2

6.1.8. x = l q1 /( q1 + q2 ) q1 . . .

6.1.9. q = l 80 mg.


1
q2
Q2 .
6.1.10. T =
2
40 l
3 3
6.1.11. = 2 arctg(q/Q)2/3 , = .
6.1.12. r = 1,4 108 .
q
6.1.13. = q (3 2 4)/(80 ml3 ).
6.1.14 . q = 320 mgR2 /Q.

q 2 a2 + l 2

6.1.15. k =
.
320 a3 ( a2 + l2 l)
!

3
q2
9
.
6.1.16 . T =
+
3
80 l2 4
6.1.17. E1 = 0, E2 = Qh/[(40 (R2 + h2 )3/2 ].
6.1.18 . E = l/[40 x(l + x)].
6.1.19.
) E = /(60 ); ) E = (1 2 )/(40 );
q
12

22

32

+
+
1 2 2 3 1 3 /(30 );

) E = 3 l/(120 ).
6.1.20. . .
6.1.21. . q = 10 Q. . q = 9Q.

) E =

) E = /(20 );

E = h(1 cos )/(20 );

6.2. .
6.2.1. . = El2 /2. . = Eh2 , = Eh2 .
6.2.2. = E cos (R2 r 2 ).
6.2.4. F = .
6.2.5. . F1 = F2 = q/(20 ), E = /(20 ). . F = q/(40 ).

6.2.6. ) E = 0 r < R, E = Q/(40 r 2 ) r > R; ) E = /(20 r); ) E = /(20 );


) E = r/(30 ) r 6 R; E = R3 /(30 r2 ) r > R; ) E = r/(20 ) r 6 R; E =
R2 /(20 r) r > R; ) E = x/0 x 6 h/2 (x
); E = h/(20 ) x > h/2.
322

6.2.7. ) = 2E0 0 /r; ) = E0 0 /r.


R
R
6.2.8 . , , F = En ds, En ds
, .
,
.

= q/0 = 6l2 /0 , 0 = l2 /0 .
0 =

En dS + l2 /(20 ),

,
Z
En ds =

l2
l2
l2

=
.
0
20
20

, F = 2 l2 /(20 ). ,
F =

3 2 l2 /(80 ).

6.2.9. E1 = /0 , E2 = 0. E10 = 0, E20 = /0 .


6.2.10. E1 = (/0 ) sin(/2), E2 = (/0 ) cos(/2).
6.2.11. E = h/0 .
6.2.12. EA = h/(60 ), EB = h/(30 ), E(r) = r/(30 ).
6.2.13 . ,
, E = l/(30 ).





r3

r3
E =
x+

0
<
x
<
l

r;
E
=
x

l + r <
30
(l x)2
30
(x l)2
 2

2

R
r
x < R; E =

x > R.
30 x2
(x l)2
6.2.14 . . E = l/(30 ). . = 30 E cos ,
, . = 30 E.
323

6.3. .

6.3.1. . v = 107 /. .
v = 1,25 106 /.
6.3.2. . = 850 . v = 3 107 /. . v = 8,8 106 /.
6.3.3. =
2,7 108 .
6.3.4. = 2 q/(0 l).
6.3.5. 11,9 .
6.3.6. = 13,5 = 45 .
6.3.7. = Q/(40 R). . .
6.3.10. . E = nQ/(40 R2 ), E = Q/(40 R2 ). . E = (2b).
6.3.11. 10 = (1 + 2 )/2, 100 = (1 2 )/2, 20 = (1 2 )/2, 200 = (1 + 2 )/2.
6.3.12. . = 37,7 = 11,3 . . = 18,8 = 5,65 .
6.3.13. 3 1 = [(3 1 )(h1 + h2 ) + 2 (h1 h2 )]/(20 ).
6.3.14. E12 = /a; E23 = /b.
6.3.15. . E = b/[0 (a + b)],
E = a/[0 (a + b)]. = b/(a + b),
= a/(a + b).
. qa = qb/(a + b); qb = qa/(a + b).
6.3.16. Q0 = Q, = Q/(4R2 ), E = (Q + q)/(40 L2 ). . .
6.3.17 . q, q,
( )
. E = 0 0 < x < r, E q/(20 xL) r < x < R, E = 0
x > R; x .
6.3.18. .

6.3.19. . .

6.3.20. 1 = q/(40 r), 2 = q/(80 r), 3 = 0.


6.3.21. qr = 80 r, q2r = 160 r.
6.3.22. 1 = (R2 R1 )/R2 , 2 = R1 /R2 .


q
1
1
q
6.3.23. E = 0, = 0 r > R2 ; E =
,

R1 < r < R2 ;
40 r2
40 r
R2


1
1
1
E = 0, =

r < R1 .
40 R1
R
2



q
R1
q
R1
qR1
1

=
1

r > R2 ; E =
,
6.3.24. E =
2
4
R2
40 r
R2
4R2 r 2
0r

q
R1
=
1
R1 < r < R2 ; E = 0, = 0 r < R1 .
40 R2
r


r

r2
R3
R3
6.3.25. E =
,=
R2
0 < r < R; E =
,=
r > R.
2
30
20
3
30 r
30 r
2
2
2
6.3.26. = R /(60 ), = R /(40 ), = h /(80 ).



R
r2
x2
r 2
R
6.3.27. =
r 2 ln
+

0 < x < R; =
ln
r < x < R.
20
r
2
2
20
x
2
2
6.3.28. F = Q /(160 h ).
2
6.3.29. F = 3Q2 /(32
0 h ).
6.3.30. . F = q 2 (2 2 1)/(320 l2 ).
324

6.3.31 . = q/(40 L).


6.3.32 . Q = qR/L.
6.3.33. F = Qq/(40 L2 ) L > R; L < R.
6.3.35. h = 3q 2 R2 /(16 2 0 gr6 ), h .
6.3.36. Q0 = qQ/(Q q).
6.3.39. .
6.3.40. C = 40 R.
6.4.
6.4.2. . . . ; n .
6.4.3. . C = 0 S/d. . C = 5,3 = 5,9 .
6.4.4. . . . 1 + S 0 /(2S) . . .
R1 R2
6.4.5. C = 40
.
R
1
 1 R2
1
d
1

.
6.4.6. C = 40
R1
R2
R0 (R0 d))
6.4.7 . C = 20 l/
ln(R
/R
).
2
1

ld
0 al
6.4.8 . C =
1+
.
d
2R2
0
0
6.4.9.
. C = C1 C2 /(C1 + C2 ), C = C1 + C2 . . C = 4C/3. . C = 7C/5.
. C 0 = ( 5 1)C/2. . C 0 = 6C/5.
6.4.10. q = 0 SE.
a
6.4.11. V =
(V1 + V2 ).
d+a

6.4.12 . . . . .
6.4.13. q = qx/d.


V
Cd + 20 S 4n
6.4.14 .
=
.
V0
Cd + 0 S
2
6.4.15. F = 4,4 10
. .
6.4.16. k . n2 .
6.4.17. ) W = 4,4 ; ) W = 20 r1 r2 V 2 /(r2 r1 ); ) W = 0 lV 2 /[ln(r2 /r1 )].
6.4.18. . A = Q2 d/(20 S). . A = Q2 dx/[20 a2 (a x)]. . A = Q2 d/(40 A); A =
Q2 dx/(20 a3 ).
6.5. .
6.5.1. . F = 2 S/(20 ). P = 2 /(20 ). . = 0 E. P = 0 E 2 /2 ( ), P = E 2 /(8)
( ). . P = 4,325 , = 8,85 /2 .
6.5.2. 1 + Q2 /(2P0 0 S 2 ) .
6.5.3.
: = 0 E. ,
E 0 /(20 ) = E,

E 0 + /(20 ) = 2E,

: E 0 = 3E/2. , , , . .
P = E 0 = 30 E 2 /2. E 2E, , = 30 E
E 0 = E/2. ,
, .
: P = E 0 = 30 E 2 /2.
6.5.4. P = 2 h2 /(20 ).
6.5.5. P = Q2 /(32 2 0 R4 ) (. 6.5.3).
2 V 2 /[2r 2 (R r)2 ].
6.5.6. P = 0 R
6.5.7. = 2R 20 P .

2 q/(80 ), F3 = 3 q/(160 ); E1 = /(40 ), E2 =


6.5.8 . . F
1 = q/(40 ), F2 =

2 /(80 ), E3 = 3 /(160 ). . E = R/(40 ).


6.5.9 . F = Q2 (R2 h2 )/(320 R4 ); q = Q/2.
21

325

6.5.11. A = 2EdS.
6.5.12. . = 0 E, P = 0 E 2 /2. . A = 0 E 2 hS/2.
6.5.13. A = 2 Sh/(20 ).
6.5.14. A = 0 ShE0 (E0 E).
6.5.15. W = Q2 /(80 R) ( ); W = Q2 /(2R) ( ).
6.5.16. r = 1,4 1015 .
6.5.17. 1400 .
6.5.18 . W = 3Q2 /(200 R).
6.5.19. A = Q2 /(80 R).
Q2
6.5.20. A =
(1 N 2/3 ).
80 R
6.5.21. n .
n(n 1)
6.5.22. A3 = 3A; An =
A.
2
6.5.23 . A0 =
6A.
6.5.24 . A0 = 2 A.
6.5.25. W = Q2 /(40 l).
6.5.26 . A = (Q1 1 + Q2 2 )/2.
6.5.27 . ) F = 2Q2 dc(d c)/[0 a3 (2d c)2 ];
) F = 0 acV 2 /[2d(d c)].
6.5.28. A = Q2 ab/[2S0 (a + b)].
6.5.29 . F = q 2 Sd/(8 2 0 r 5 ).

6.5.30 . , , ,

E(0) = E + E = 0.
 r  R , O
E (0) = q 0 /(20 R3 ).  r, ,
,
E(0) = E E = q 0 /(20 R3 ).
q 0 = qr 2 /(4R2 ).
E(0) = q 2 r 2 /(80 R5 ).
6.6.
6.6.2. p = 7,4 1037 .
6.6.3. p = 1 1034 .
6.6.4. = ( 1)/. :
E = /(0 ) ,
E = /0 .

p V = (/0 )(d h + h/).
6.6.5. E = E0 sin2 +(cos2 )/2 .
6.6.6. .
6.6.7. = 2.
6.6.8. q = ( 1)CV .
1
1
V . V =
V.
6.6.9. V =
+1
(n 1) + 1
6.6.10. V = k( 1)V /n.
6.6.11. C = 0 (1 + 2 )S/(2d).
0 1 2 S
1 2
6.6.12. C =
; q =
q.
2 d1 + 1 d2
1 2
0 d2 ( 1)(S2 S1 ) + 0 d1 S1
6.6.13. C =
.
d1 (d1 d2 ) + d1 d2
326

6.6.14. = q/(1 Sd).


6.6.15. . .

. F2 =

1 (2 1)
F1 .
2 (1 1)

. F V 2 , F 1/r 3 .

. (R/r)3 .

( 1)SQ2
.
8 2 0 R5
0 ( 1)SdE 2 sin2
0 ( 1)SdE 2 sin 2
; A=
.
6.6.17 . M =
2
2
( 1)Q
( 1)Q
6.6.18. =
, =
. . .
4r2
4R2
6.6.16. F =



( 1)Q2
1
1

.
32 2 0
r4
R4
2
Q d
1
6.6.20 . F =
.
20 b [a + x( 1)]2
6.6.21. h = 0 ( 1)V 2 /(2gd2 ).
6.6.22. h = ( 1)Q2 /(20 gS 2 ).
q2 1
6.6.23. W =
.
2C
V 2C
6.6.24. W =
( 1).
2
1/2
2W
6.6.25 . V =
. E 109 /.
(1 2 )C
6.6.26. ; = 2.
1
6.6.27. . V 0 =
V . . T 105 .

6.6.28. r = 0,12 .
6.6.29. p = 40 r 3 E.
6.6.30. = 1 + 4r3 n.
6.6.19 . P =

7.

7.1.
7.1.1. .
7.1.2. t = 0,56 ; x = 2,8 .
2
7.1.3. E = 2(d
q vt)m/(qt ).
p
7.1.4. u = v 1 + 2qEd/(mv 2 ), tg = tg 1 + 2qEd/(mv 2 sin2 ).
7.1.5. .
7.1.6. K = qEl/[2 cos2 (tg + tg )].
327

7.1.7. N = neU bl2 /(2me vd).


7.1.8. q = 1017 .

1 + sin2
cos 1 + cos2 cos2

; tg =
.
cos
cos 1 + cos2 + sin2
p
7.1.10 . tg = tg 1 + 2e(2 1 )/(me v 2 sin2 ).
7.1.9. = arctg

7.1.11. V = 19 .
7.1.12 . V = (V0 /2)/ ln(R2 /R1 ).
7.1.13. , .
B, K0 e0 .

7.1.14 . t0 = t 3.

7.1.15. n.


1
1
4
eQ
7.1.16. K 0 l 2R; K =
+

l > 2R. 40 R
lR
l
, ,
.


qQ(m + M ) 1/2
2R
1
.
7.1.17 . t =
2
v
20 RmM v
p
7.1.18. v = qEl/m.
p
7.1.19. T = 2 ml/(2qE).
q
p
p
7.1.20. T = 2 ml/(mg + qE) mg + qE > 0; T = 2 ml/ (mg)2 + (qE)2 .
p
7.1.21. = qQ/(20 ml2 ).
s
g
qQ(h + l)
qQ(h + l)

7.1.22 . =

< mg.
l
40 mh3 l
40 h3

1/2
g
qQ
qQ
+

> mg.
7.1.23 . T = 2
3
R
320 mR
320 R2
7.1.24 . t = 0 R(me v 2 )2 /(2e2 C).
7.1.25 . = /4; 4K()2 /(eE).
7.1.26. k = l/(2d).
7.1.27. . p1 = q1 q2 /(20 vr).
7.1.28 .

. l = 3,4 1013 .

k = 1/(20 ).

7.2.
7.2.1. .
p
7.2.2. x = v
me 0 /(e) x 6 l;
2
7.2.3. . k .

s
x=l+v

2me 0
ctg
e

 r

1
e
R x > l.
v (2me 0 )

. k .

7.2.5. y = y0 f /(x0 f ).

7.2.6. . . . , AA0 ,
, , p =
e/(2rv), = r 2 E r , v .
f = r me v/p = 2me v 2 /(eE) = 4d. ( f ,
.)
328

s
!#
"



4
V0
V0
V0 V0
+1 1
V < 8V0 .
f = d
1+
2
+2
V
V
3
V
V
8dV0 /V V > 8V0 .
7.2.8 . f = d(4V0 /V )2 .

7.2.9. m, q v , p = qq/(20 vx), x  R


, q x2 2R
mv 1
R V0
=
.
, x; f =
p x
2 V
7.2.7 .

7.2.10. f = 2R(V0 /V )2 .
 2
1
V0
, f = 2R
.
7.2.11 . x
(1/f 1/L)
V
2
2
7.2.12. E = (a b) /(16d ).
p
7.2.13. V = V0 2d/l.

7.2.14 . f = 4V (2E2 E1 )/(E2 E1 )2 .


7.3.
7.3.1. t = 2me l/(eE ).
s
s
2eV
2eV
eV
eV
7.3.2. )

6v6
+
;
me
me l
me
me l
s
s
2e(V + V )
2e(V V )
; v2 =
V < V .
) v1 =
me
me
7.3.3. 109 .
7.3.4. . S = Ll/(2V d). . S = 0,09 /.
7.3.5.
5 .
p
7.3.6. > l 2eV /me .
7.3.7. V = /(2S).
329

e
l2 f 2
=
, n .
me
2V (n + 1/2)2
s
(

 r
)
V0
2e
me
7.3.9 . = arctg
1 cos l
.
d me V
2eV
7.3.8.

7.3.10. . v = l/(2n). . b = 4eV0 n/(me 2 d), n .


2eE0
eE0
7.3.11. |u | =
| cos |, v =
cos .
me
me
7.3.12 . K = 0,4 .
7.3.13 . - ,
, ,


2eE02 E0 2
.
. V =
me 02

q
7.3.14 . A = eE0 /[me ( 2 02 )2 + 4 2 2 ].
q
7.3.15 . = 1 + 4ne e2 /[me ( 2 02 )2 + 4 2 2 ].
7.4.
r
e
1
7.4.1. v =
.

4
m
r
e
0
q

7.4.2. v = e2 (4 + 2 )/(80 me a).


q

7.4.3 . vp /ve = (me /mp )(4 2 + 1) 0,01. , , .


7.4.4. r = e2 /(40 me v 2 ).
7.4.5. r = e2 /[0 me (v1 + v2 )2 ].
p
7.4.6. v = q1 q2 (m1 + m2 )/[20 m1 m2 (R1 + R2 )].
7.4.7. r = de2 /(e2 + 40 me v 2 d cos ).
7.4.8 . = /2.
p
7.4.9 . v = q 2 /(80 md).
s

mv02
q 2 (2 2 1)
q 2 (2 2 1)

7.4.10 . v = v0 1

>
. 80 mv02 d
2
160
A +q, ,
, . , . q
7.4.11. v =

4e2 r 2 /[0 me (4r2 + R2 )3/2 ].

7.4.12. K = Ze2 /(8


0 r).
2 (2 2 )/(4 r).
7.4.13 . K = e
0

7.4.14 . n = ( 2 1)m/M + 2.

7.4.15 . v = 2v.
7.4.16. .
s

2
e2
e2

2+
7.4.17 . r =
+

.
20 mp v 2
20 mp v 2
4q 2 (l r)
h
i.
2
2
2
rl u + v + 2uv cos( + ) rl 2 (u sin v sin )2

7.4.19 . t = 2 2 t0 .
p
7.4.20. v > qQ(m + M )/(20 RmM ) qQ > 0; qQ < 0.
s
2
mv0
M V0
QqM
+

.
7.4.21. v =
m+M
m+M
20 Rm(m + M )
7.4.18 . m =

330

7.4.22 . v =

p
3qQ(m + M )/(40 mM R) qQ > 0; v = 0 qQ 6 0.
p
p

7.4.23 . v = q 2 /(60 ml); v = q 2 /(240 ml).


p
7.4.24. v = q 2 m(2R l)/[20 RlM (M + 2m)].
!


p
R
Q2
Q
7.4.25. x =
1 .
1 , v = gR p
2 40 mgR2
40 mgR2


2
2
q
h0 cos
7.4.26 . h =
(1 ctg ) mg(1 tg ) .
mg
80 (H h0 )H sin
7.4.27. W = 3q 2 /(320 l).
7.4.28. k = [q 2 /(2
p 0 l1 l2 ](l1 + l2 + 2l0 ).
7.4.29. v = v 1 + q 2 /(40 Rmv 2 ).
Q2
4
7.4.30. ) W = R3 v 2 +
+ 4R2 (2 22/3 ).
3
40 R
Q2 + q 3
(Q q)2
4

) W = R3 v 2 +

+ 4R2 (2 22/3 ).
3
80 R
80 3 2R
7.4.31. . O.
p
7.4.32 . v = 2gh[1 S 2 /(40 mg)] mg > S 2 /(20 );
p
v = 20 mg 2 h/( 2 S) mg < S 2 /(20 ).
s


q2
1
1
7.4.33. v =

.
40 m r
R
s
2 l
R1
ln
.
7.4.34 . v = v0 1
20 mv 2
R2
q

7.4.35 . T = 2 40 ml3 /( 2 q 2 ).

7.4.36 . ) . Ei =
neh/(20 )
t 20 me v/(e2 hn);

e2 hn/(80 me v).

) (. .),
p

T = 2 e2 n/(0 me ). t = T /4 =
p
(/2) e2 n/(0 me ), = 1/4t.
7.4.37 . n = q
8 sin2 (/2).
7.4.38 . x =

l2 + l02 l + l0 , l0 = q 2 /(80 M g).

331

8.
8.1. . .
p
8.1.1. . I nec/l = 0,02 . . I = e4 /[160 me (r)3 ] = 0,0012 .
8.1.2. v = Il/q.
8.1.3. I = 20 Eav = 1,3 104 .
8.1.5. v = 0,4 /.
8.1.6. j = e.
8.1.7. j = ene u.
8.1.8. I = sj sin = 10 .
8.1.9. t = 8 106 .
8.1.10. = j/v.
8.1.11. E I/(20 vr) = 6 105 /; L [8me rv 2 /(3eE)]1/2 0,1 .
0 v 0
8.1.12 . ) = q
, x .
v02 2eEx/me

) 2 = 21

x < x0 = me v02 /(2eE); = 0 x > x0 . 2 x


:
1
E1 =
20

Zx0
2 dx =
0

0 me v02
.
0 eE

, E1  E. E1 E,
. . 0 me v02 /(0 eE) E, . e0 E 2 /(me v02 ).
8.1.14. T1 , T3 .
8.1.15. ,
, .
r
me d 1
1
I

8.1.16 . =
= 1,75 106 /3 .
S 2eV x
x
x 0 , ,

Rd
( = dx), : = 3,5 106 d.
0

: E 0 = /(20 ). E 0  V /d
.
s
4
4
2e V 3/2
8.1.17 . n = ; j = 0
, I = jS.
3
9
me d2
8.1.18 . n , n3/2 .
8.1.19. j = i/(2r).
s
2I
l2
2I l
8.1.20. . j1 =
1 2 ; j2 =
, l AB
2
4r
r
4r2 r
, j; r A B .
, .
I 0.
s
2I
h2
. j =
1 2 , r , j.
2
4r
r
8.1.21 . j = qvl/(2r 3 ).
332

8.2. . .
8.2.1 . . = e2 ne /me . . = 2,4 1015 .
8.2.2 . N/N = 1,5 1010 .
8.2.3. f = ne2 v/.
8.2.4. I = me rs/(e ) = 1,7 .
8.2.5. .
8.2.6 . / . : / =
2 k2 T /(3e2 ), k , T , e .
8.2.7. E = j/; V1 = (jl/) cos ; V2 = jl/(2).
8.2.8. = 0 j(1/1 1/2 ).


2
8.2.9. tg 2 =
tg 1 ; = 0 j cos 1 1 1 .
1
2
1
8.2.10. = 0 j/(a).
8.2.11. . I = Q0 /(0 ). . Q = Q0 exp[t/(0 )].
8.2.13. I = SV /l; 
R = l/(S).



l
1
1
1
l1
l2
8.2.14. RI =
+
,
R
=
+
; II = V /R1 , III = V /RII
II
r 2 1
2
r12 1
r22 2
|r2 r1 |  l1 ,l2 .
8.2.15. R = 0,0566 .
8.2.16 . R = R0 / cos2 .
8.2.17 . I = 4rV ; R = 1/(4r).
8.2.18 . R = 0,14.

1
1
1
q
8.2.20. R =

;I=
.
4 r1
r2
0

8.2.21 . C = 0 /(R); .

8.2.22 . .

2
1
I
8.2.23. K = me
= 2 1015 .
2
ene S
8.2.24 . I = F l/(qR); v = F l2 /(q 2 R).
8.2.25. . = qvR/l. . = F l/q.
2
8.2.26. V = W/e; I
p = e. R < W/e .
8.2.27 . I = I0 (1 V /V0 ).

8.2.28. . . W = E l.
8.2.29. E = 1,13 .

8.2.30. E = 1,07 . .
8.2.31. = 1,4 102 .
8.2.32 . - -,
.
8.2.34 . k = V /(2E).
333

8.3.
8.3.1. r = 1,5 50 .
8.3.2. r = 20 .
8.3.3. V = 1 .
8.3.4. , ,
b . ,
.
8.3.5. . V /V = R/(R + r). . I/I = r/(R + r).
8.3.6. IV /I6 = 10/64, V 40 .
8.3.7. 100 .
8.3.8. .
8.3.9. V = 48 ; I = 15 .
8.3.10. rx = rR2 /R1 ; .
8.3.11. R = V1 /I1 ; R = V2 V1 /(I2 V1 I1 V2 );
RA = (V1 V3 I2 V3 V2 I1 V2 V1 I3 )/I3 (V1 I2 V2 I1 ).

8.3.12 . , . A O
, C O , C A, C B ,
A B . R ICA + ICB . R =
V /(ICA + ICB ), V .
8.3.13. R = 7 .

8.3.14.
r = 3 R. . r = ( 3 1)R. . In = I(2 3 )n1
2R;
a. n1

0
In = I(2 3 )
( 3 1) R, n , R0 = ( 3 + 1)R.
8.3.15 . R1 = 9r; R2 = 10r/9.
8.3.16. : V = E I(r+R); : V = E I(r+R); : V +E1 +E2 I(r1 +r2 +R);
: V = E1 E2 I(r1 + r2 + R).
8.3.17. E = 34,3 ; r = 1,43 .
8.3.18. E = 10 r = 14 .

8.3.19. . .

334

I = 10 , r = 20 ; E = 200 , r = 20 .
I = 80 .
I2 = I3 R3 /R2 ; I1 = I3 (R2 + R3 )/R2 ; V = I3 (R1 R2 + R1 R3 + R2 R3 )/R2 .
. .
. V = 5ir; R = 5r/6; I = 6i. . . . I = 7i/2; R = 12r/7;
. RAB = 13r/7; RCD = 5r/7.
8.3.26. I = 8 .
8.3.27 . . I = i/2; R = r/2. . R = r/3. . RAB = 2r/3; RAC = r.

8.3.20.
8.3.21.
8.3.23.
8.3.24.
8.3.25.

8.3.28. E = (E1 r2 + E2 r1 )/(r1 + r2 ) = 21 , r = r1 r2 /(r1 + r2 ) = 3,75 .


8.3.29. . .
8.3.30. .
8.3.31. V = 0; I = 0,75 .
8.3.32. V = 0,75 .
8.3.33. 12, 54 27 .
8.3.34. N = I 2 R.
8.3.35. N 0 = N0 (N N0 )/N .
8.3.36. R =
9(n 1)r.
8.3.37. r = R1 R2 .
8.3.38. 2 100 ; 20 0,1 .
, .
8.3.39. S = 42 2 ; 10 .
8.3.40. N = (E Ir)I; R = r.
8.3.41. N1 = 125 ; N2 = 80 ; N3 = 45 .
8.3.42 . r = R.
8.3.43. N = (V E)E/r; N = (V E)2 /r.
E > V /2, .
8.3.44. N = 4 .
8.3.45. N = CV 2 /0 .
8.3.46. N = I(me v 2 /2e IR).
8.3.47. q = 4 2 0 a3 ene Rv, v  a2 e2 ne R/me .
8.3.48 . T = T0 + R0 I 2 /( I 2 R0 ), > I 2 R0 . < I 2 R0 T .

8.4.
8.4.1. . q = 8 104 . . V = 60 . . 30, 30, 60 .
8.4.2. V = V0 x/(2x l);
.
s


x
kx
8.4.3. A = B + 2 l
.
2
0 S


C2
R1
8.4.4. A B = E

. R1 + R2
C 1 + C2
, q1 = C1 R1 E/(R1 + R2 ); q2 = C2 R2 E/(R1 + R2 ).
.
CV 2
R1
CV 2
R2
8.4.5 . W1 =
; W2 =
.
4 R1 + R2
4 R1 + R2
2
8.4.6. W = A q /C.
8.4.7 . q = CE; W = CE 2 /4.
8.4.8. W = C(E V0 )2 /2, E > V0 ; W = 0, E < V0 .
8.4.9. W = C(V E)E; W = C(V E)2 /2.
8.4.10. , . . 1/n .
8.4.11 . N = Iq/C > N = Iq/(2C). -
, .
8.4.12. 103 RC.
E1 R2 + E2 R1
E1 R2 + kE2 R1
; q=C
.
8.4.13 . q = C
R1 + R2
kR1 + R2
335

8.4.14 . V = V0 R /(rT + R ).
 
dV
V
8.4.15 .
=
; V = V0 exp
.
dt
RC
RC
V0

I=
exp
.
R
RC
8.4.16. R < 40 .


V V0 1
.
8.4.17 . = RC ln
V V1
8.4.18. . I = qv/d. . .
8.4.19. I = 0 ( 1)Eav/d.
"
#1/2

E
1
E 2
E2
1
8.4.20. I =
+

+
2
.
2R2
R
2R2
R
R

8.4.21. - I = (E V )/R;
2 .
, , R > 0,3 R > 3
- .

9.
9.1. .
9.1.1. B = 100 .
9.1.2. B = 20 . s

I1
L2
L
RI2
1 + 2 2 cos . ) F2 = 2F
.
I
l
l
lI

9.1.4 . h = aV B/(bg).
9.1.5. = 45 .
mg
9.1.6. I =
ctg .
2aB
p

9.1.8 . = 6BI/m.
9.1.9. tg = IB/(4g).

9.1.10. I ,
. , , h 0
, :
!
 X
X
~
~ B].
~
N
[Mi B] =
Mi B
[M
9.1.3. ) F1 = F

h0

336

h0

IB
(4 + )IB
. . tg =
.
2g
4(1 + )(2 + )g
2
9.1.12. N = R IB(sin + cos )/2.
9.1.13 . B = P/(RIn).
9.1.14. a = 2RIB sin /m.
9.1.15 . B = F/(RI).
9.1.11. . tg =

9.2. .

9.2.2.
9.2.3.
9.2.4.
9.2.5.

B = 0 v/(2r), r .
B = 0 I/(2r), r .
= 1,25.
B = 1,88 105 .


0 I 1
1
9.2.6. B =
+
.
2
x
y

0 I
9.2.7. B =
sin , l .
2l
2
0 Il
0 qv
sin
. . B =
sin .
9.2.8. . B =
4r2
4r2
2
9.2.10. B = 0 I/(2R); Bh = 0 IR /[2(R2 + h2 )3/2 ].
9.2.11. n = sin (/2).
0 I 

9.2.12. B =
1+
.
2R
2
9.2.13. B = 0 I/(4R).
0 I( + 1)
;
9.2.14 . B0 =
2R

1/2
0 I
1
R4
2R3
Bh =
+
+
.
2
2 (R2 + h2 )
(R2 + h2 )3
(R2 + h2 )5/2

9.2.15. . I = I0 10. . I = 2I0 10.


9.2.16. B = 0 M/(2h3 ).
p
9.2.17 . B = 0 M 1 + 3 sin2 /(4r3 ), M = Ia2 .

9.2.18 . I, , ,
, .
, , h 0
, . ,
. h 0
,
.

22

337


9.2.19 . . M0 I =
M0 /a2 . ,
,
I, I,
. M = Ib2 = M0 b2 /a2 =
nM0 .
. I = hM ,
M
.

B I B = 0 I 8/(a). M = Ba/(0 h 8 ).

9.2.20.
9.2.21.
9.2.22.
9.2.23.

B = 0 M R2 h/[2(R2 + l2 )3/2 ].
B = 4,9 102 .
B0 . N = 2BB0 R3 /0 .
p
M = HF/(20 ah2 ).

9.3. ,

9.3.1. B = 0 v/2.
9.3.2. B = 1010 .
9.3.3. 0 i/2.
9.3.4. B = 0 (i1 i2 )/2, B = 0 (i1 + i2 )/2.
9.3.5. F = 0 I 2 /(2b).
9.3.6. . = 0 aI 2 /(8Eb2 ). . B1 10 . B2 35 .
9.3.7. Bk = 0 e0 E v = 0 i/(4), E = /(40 )
, ,
, v .

9.3.8. . B = 0 i/4. . B = 0 i; . . B = 0 aj/(4 3 ).


2
9.3.9. T = 0 nRI /2.
9.3.10 . . Bk = 0 i/(4), ,
(. 9.3.7). AA0 = 2, Bk = 0 i/2.
!
1
1
1
. B = 0 i 1 p
, B =
0 i(R/x1 )2 .
2
x

2
4
1
1 + (R/x1 )
!
1
1
B = 0 i 1 + p
, B = 0 i.
x2
2
1 + (R/x2 )2
338


9.3.11 . .
, . , M (M
); (. 9.2.19 ).
.
. , 8/3 ( ,
14) 4, . . n = 1,5 .
0 M
. B = p
;
1 + 4(r/l)2
. B = 0 M

(r/l)0

0 M ,

1 p
1 + 4(r/l)2

(r/l)0

(r/l)

20 M r2
,
l2

0 M l
.
2r
B

(r/l)

0 M .

9.3.12.
B, 4
, . B = aB0 /(2 2h ).
9.3.13. Bk = 6,28 104 , B = 0,377 .
9.3.14. B = B0 h/(2R).
9.3.15. . B = 0 Ix/(2r 2 ), 0 < x < r;
. B = 0 xj, x = a/2;

B = 0 I/(2x), x > r.

B = 0 aj/2, x < a/2.

9.3.16. B = 0 N I/(2r), B = 0 N I/(2R).


9.3.17. . B = 0 I/(2x),
; B = 0.
. B = 0 I/(2x), B = 0 (I I 0 )/(2x).
. B = 0 I/(2x), B = 0.
0 I

tg .
2r
2
9.3.19. . . B = 0 hj/2.

9.3.18 . B =

9.3.20. B =

0
h
jx, 0 < x < ;
2
2

B =

0
hj
2


1


h
h
, x > , x
4x
2

O.
9.3.21 . B = 0 jd/2.

9.3.22 . . B = 0 ja/2.

. i = 2B0 sin /0 , i = 2B0 /0 . . .

9.3.23 . Bk = 0 nI,
, B = 0 nI tg .
339


9.3.24 . (.
9.3.11 ) , .
. B = 0 M/2
x < r; B = (0 M/2)(r/x)2 x > r.
9.4.

9.4.1 . = 3 Ba2 /2 . = BS sin .


9.4.2 = B R2 (sin2 sin2 ).
9.4.6 n = sin /sin, i = (B/
s 0 ) cos (1 tg ctg).

9.4.7 . B2 = B4 = B1

9.4.8. . Br =

1
r
B0 ,
2
x

a1
=
a2

tg =

B12 + B32 + 2B1 B3 cos


.
2 cos(/2)

1r
; . .
2x

. Br =

1
r
nB0
2
x0

x
x0

n1
, Br =

1
f
rB0
.
2
x

9.4.9.
, R/r, r
340

, = B0 R/(2x0 )
.

9.4.10 . .
B0 = 0 i, R2 R2 B0 . (1 )
AA0 , (2 ) : R2 B0 =
1 + 2 . 2 = R2 B0 1 . AA0 Bk = B0 /2 (.
9.3.10 ), 1 = R2 Bk = R2 B0 /2 2 = R2 B0 /2 = 0 iR2 /2.
. , , Fk = B S nI = nI = ,
. Fk = nI , = 0 nIR2 /2.
, Fk = 0 (nIR)2 /2.
p
9.4.11. B = 20 F/(R2 ).
9.4.12. F = nI(1 2 ).
9.4.13. . L = 0 (rR)2 /l3 . . L = 0 nr2 .

10.

10.1.
10.1.1. R = 0,2 .
10.1.2. R = 0,68 .
10.1.3. . = qB/m. . = 1,75 1011 1 .
p
10.1.4. R1 /R2 = K1 /K2 .
10.1.5. t = 2m/(qB).
10.1.6. K = 3(eBR)2 /(4mp ).
10.1.7. sin = eBl/(me v) eB/me 6 v/l; = eB/me > v/l.
10.1.8. x1 = 0,29 , x2 = 0,41 , x3 = 0,5 , x4 = 0,58 , l = 3,7 .
10.1.9. V /V0 < 0,025.
10.1.10 . l = 2mv/(qB), z = mv()2 /(4qB).
10.1.11. R = mv sin /(qB), h = 2mv cos /(qB).
3
10.1.12 . x = 2me v/(eB), y =
me v() /(4eB).

10.1.13. . . . B > B0 = 2 2me k /(eR). . P2 > P1 .


10.1.14. B = me v/(eR) + e/(160 vR2 ).
10.1.15. = 0 eB/(2mep
).
10.1.16. V 0 = 2V h/R Bh 2eV /me .
me E 2
10.1.17. . y =
z .
eB 2 lL
. y[] = 1,1 s
104 1 z 2 .


me E
eBlL 2
2+
. y =
z
.
z
eB 2 lL
me c
 2 2 2

e B R
mp
K .
10.1.18. t = 2
e BV
2mp
eB 2 d2 1
10.1.19. V =
, k = 1,2, . . . .
2 2 me k2
.
mg
10.1.20. v =
(sin cos ) 6 tg ; v = 0 > tg .
qB
10.1.21. M = 2R2 vBR .
10.1.23 . v = Q(B2 B1 )R/(2m).
10.1.25. M = QR2 (B1 B2 )/2. .
22

341


10.1.26 . t = l/v,
v , . , , p = eB vl/v = eB l = e/(2R),
. M = Rp =
(e/2). M2 M1 = (e/2)(1 2 ).

p
10.1.27 . n = (1 B1 /B2 )/2.
p
10.1.28 . r = R B2 /B1 .
10.2.
10.2.1. v = 2v(B1 B2 )/[(B1 + B2 )].
10.2.2 . v me v 2 /(eB
0 ).
s
1
2mEl

10.2.3. . . R =
,
B
q

2 ql E

v =
.
2 ql B + mE
10.2.4. v = E/B.
10.2.5. v = E/B.
10.2.6. v = (E/B) sin .
10.2.8. v 6 eBh/(4me ) v = V /(hB).
10.2.9. V = eB 2 d2 /(2me ); V = 3,5 105 .
10.2.10. , E/B, 
1/2
me v 0
E
E2

, v 0 = v 2 + 2 cos + 2
.
eB
B
B
10.2.11. v = F/(qB).
10.2.12. ve 8 107 /, vp 1,5 103 /.

11.
11.1. .

11.1.1. .
11.1.2. V = 0,03 .
11.1.3. V = vbB; = 0 vB.
11.1.4 . v < Ze/(40 Br2 ).
11.1.5 . V < 7 .
342

11.1.6. E = vB.
11.1.7. B = V /(a2 ).
11.1.8. . . . . M = (a2 b2 B 2 /R) sin2 t.

11.1.9. W = B 2 vab/(2), a < b; W = B 2 vb2 /(2), a > b.


11.1.10 . W = B 2 l2 v tg /(2).
11.1.11. N = (vB)2 SL/(4) = 1 .
11.1.12 . I = BvS = 10 , V = vBh = 200 .
11.1.13. V = IB/(h).
p
11.1.14. . v = 2BIlL/m. . v 1,1 107 /.
p
11.1.15. v = IB/(b).
11.1.16. It = 2r02 Bv/[R0 (r0 + vt)].
11.1.17. Q = SB/R.
11.1.18. B = 1,1 102 .
11.1.19. v = gmR/(Bl)2 . .



B 2 l2
mR
t ; v(t) = gtm/(m + CB 2 l2 ).
11.1.20 . v(t) = g 2 2 1 exp
B l
mR
11.1.21. k = I.
11.1.22 . v = mgR/(B0 a2 )2 .
11.1.23 . I = (mg/BL) cos t.





2E
2F R
2F
3B 2 L2
. (t) = 2E
11.1.24. . =
1

,
I
=
.

exp

t
.
BL2
BEL
BL
BL2
4mR

2
11.1.25 . I = Br /(2R) = 0,4 .
11.1.26 . = 0 4M /(a3 B 2 ).
11.1.27. , .
11.1.28. E = 40 
.

E
2M Rf0

.
11.1.29. f = f0
E0
E02
11.1.30. E = 120 . N = 240 .
11.1.31. M = 2EI0 /02 .
2V (I1 I2 ) + R(4I12 I22 )
I2
11.1.32 . l =
, v=
[2V I2 (2l + R)].
2(I22 I12 )
2F
11.2.
11.2.1. = 1 , 100 , 300 .
11.2.2. E = r 2 /(2l) = 2,5 105 /.
11.2.3. C -
. .
11.2.4. E1 = 6,4 106 /, E2 = 2,56 105 /.
11.2.5. E = 0 x, x .
11.2.6. E = (0 n0 I0 /l0 )x cos(2t), x ; E = 0,12 .
C1 C 2
11.2.7. . q = C. . q1 = q2 =
.
C1 + C 2
343

C3 (C2 C1 )
, q 2 = C 2 . . q 3 =
.
2
2
C1 + C 2 + C 3 2
11.2.9. . I = 1,44 . . I = 2,5 , . . I1 = 2,79 ,
I2 = 1,77 , I3 = 0,96 .
11.2.10. . I = IkT /(RC).
11.2.11. = p
V RC = 5 107 .
p
11.2.12. . V1 = t 20 ma3 /(hd), V2 = t3 320 mb2 /(9hd).
8
. V1 = (8,7 10 /)t, V2 = (1,2 1014 /3 )t3 .
11.2.13 . E = (r 2 /3)nB0 sin t.
11.2.14 . = qBl2 /(2mr2 ). .
11.2.15 . B(t) = t(1 + r2 /r02 ).
11.2.16 . . . ,
.
11.2.17 . l = 3r0 /4. 100 . r < l,
, r > l .
11.2.18 . = 2B/[r( + 20 2 )].
11.2.19 . . 2,6 1012 . . nSr 7 1014 2 , n
, r , S .
11.2.20 . m.. = 0 0 CV 2 = CV 2 /c2 , c .
11.2.21 . m.. 1027 .
11.2.8. . q1 = C1

11.3. . .

11.3.1. = 0 ISn sin , L12 = 0 Sn sin .


11.3.2. L12 = (0 r2 n/2)(cos + sin ).
11.3.3. L12 = 0 r 2 nN .
11.3.4 . V = 0 r 2 nN I0 cos t.
11.3.5. L = 0 r 2 n2 l. .


L dI
dv
e E
= me , l = 2rN.
l dt
dt
ene Sv = I.


dI
me l
El = V = L + 2
.
e ne S dt
, L1 = L + me l/(e2 ne S). .
11.3.6 . L =
0 (r12 + r1 r2 + r22 )n2 /3 = 2,3 /.

11.3.7. t = B v/(V 0 ) = 8,9 102 .


11.3.8. h  dL = 0 h/d = 6,3 108 /.
0
r1
ln .
11.3.9 . L =
2  r2

r1
0
1 + 22 ln
.
11.3.10 . L =
4
r2
0
h
ln .
11.3.11 . L =

r
11.3.12. k .
2
11.3.13. L1 = 0 (n1 r12 l1 + n22 r22 l2 + 2n1 n2 r12 l2 ); L2 = 0 (n21 r12 l1 + n22 r22 l2 2n1 n2 r12 l2 ).
11.3.14. L = L1 +
L2 + 2L12 .
11.3.15 . L12 = L1 L2 .
11.3.16 . E2 = (0 N1 N2 S/l)I0 cos t. V1 = (0 N12 S/l)I0 cos t.
11.3.17. V2 = const.
11.3.21. = 100 .
11.3.22. .
11.3.24. V = 10 .
11.3.25 . V = 60 .
344

11.4.

11.4.1. I(t) = Et/L, A = E 2 2 /(2L). .


11.4.2. ) V = (Rt + L). ) V = I0 (R sin t + L cos t).
11.4.3 . W = (LI)2 /(RT ).
11.4.4 . I(t) = (E0 /L)(1 cos t).
11.4.5. . .

C(t) = C0 [1 t2 /(2LC0 )].


p
V = V0 R C/L.
. . . C = 1/[(2N )2 L] 1 .
p
I = E C/L, q = 2EC.
s
s
CL2
CL1
11.4.10. I1 = V
, I2 = V
.
L1 (L1 + L2 )
L2 (L1 + L2 )
p

11.4.11 . . I = V0 C/L sin 0 t, 0 = 1/ LC.


V0
V0
. I =
(0 sin 0 t sin t); I =
4,8 .
L(02 2 )
L| 0 |
11.4.12. . . . VR = RI0 , VL = LI0 , VC = I0 /(C).
p
L 1/(C)
.
. V0 = I0 R2 + [L 1/(C)]2 , = arctg
R
11.4.13. E0 = 208 .
E0 ( 2 LC 1)
11.4.14. I(t) =
cos t.
L(2 2 LC)
11.4.15. L = 2,8 .
2C0 R0
11.4.16 . V = V0 sin(t ), = arctg
.
(CR)2 1
11.4.17. . IL = 0, IR = (E0 /R) sin t, N = 200 .
. IR = (E0 /R) sin t, IC = E0 C(sin t + cos t), N = 200 .
11.4.18. L = 0,16 .
11.4.19. . .
11.4.6.
11.4.7.
11.4.8.
11.4.9.

345

11.4.20 . VC0 VC C0
p
C, I , VC0 VC = LdI/dt = V0 cos t, = LCC0 /(C + C0 ).
(V0 VC0 )C0 = VC C.
VC = (1 + C/C0 )1 V0 (1 cos t).
V < 2V0 (1 + C/C0 )1




C
V
1
= arccos 1 1 +
,

C0 V0
V > 2V0 (1 + C/C0 )1 C .
11.4.21.
. I1 I2 L1 L2 ,
p
= 1/ (L1 + L2 )C I0 = V0 /(L1 ), L1 I1 + L2 I2 = LI0 , I1 I2 = I0 cos t.

s
L1
C
I2 =
(1 + cos t)I0 , I = 2V0
.
L1 + L2
L1 + L2
11.4.22 . . L1 I1 +L2 I2 = L1 I = (L1 +L2 )I0 , I0
L1 L2 .
W =

1
1
L1 L2
L1 I 2 (L1 + L2 )I02 =
I2.
2
2
2(L1 + L2 )

. I1 I1 2(I1 I2 )/(1 + L1 /L2 ); I2 I2 + 2(I1 I2 )/(1 + L2 /L1 ).


11.4.23 . R = 1,4 103 .
11.4.24 . W = (L + CR2 )(I12 I22 )/2.

11.4.25 . = 2 arcsin( LC/2). v = l/ < 2/ LC; v = l/ LC  1/ LC.


11.5. .

11.5.2. B = B0 (r0 /r)2 .
11.5.3. .
11.5.4. .
11.5.5. .
11.5.6 . .
(1/2) B0 cos , (1/2)B0 cos .
11.5.7. I = I0 (r2 /L)B0 cos .
11.5.8. I0 = D2 B/(4L).
11.5.9. B0 /2,
B0 /2.

11.5.10 . . . x ,
.
I0
I0
. I =
. . I =
.
1 /S
1 l/(Sh)


2
2
N
r
11.5.11 . L = 0 r2 1 2
.
R
l

346

p
11.5.12. I = a 2Cu gh/0 = 380 , Cu .

11.5.13 . AA0
, I
(I), AA0 .
AA0 . P = 0 I 2 /[2(h)2 ]. I ,
2h , .
f = 0 I 2 /(4h).

11.5.14. v = V /(r 2 nB) = 2 /.


11.5.16 .
1
1
1
1
B 2 (W w) + mv02 =
B 2 W + mv 2 ,
20 0
2
20
2

B0 (W w) = BW,

B0 = 0 N I/L B
, W = R2 L w = r 2 l .

v = v02 + 0 (N I/L)2 r 2 l[1 r2 l/(R2 L)] v0 .


p
11.5.17 . v = N Ir 0 /(12lm).

11.5.18 . , ,
2
.
p A = B Sl/(20 ) . v = B Sl/(0 m).
11.5.19. :
r12 B = (r12 r02 )B1 ,

r22 B = (r22 r02 )B2 .


K = (lB 2 /20 )[r14 /(r12 r02 ) r24 /(r22 r02 )].
11.5.20. v1 = v, v2 = 3v, mv 2 < B 2 lSs2 /[40 (2S s)(S s)]; v1 = 3v, v2 = v,
mv 2 > B 2 lSs2 /[40 (2S s)(S s)].
(v2 v1 )2
B 2 lSs2
11.5.21. v10 = v1 , v20 = v2 ,
>
;
1/m1 + 1/m2
20 (2S s)(S s)
2m1 v1 + (m2 m1 )v2
2m2 v2 + (m1 m2 )v1
, v20 =
,
v10 =
m1 + m2
m1 + m2
(v2 v1 )2
B 2 Ss2

<
.
1/m1 + 1/m2
2
0 (2S s)(S s)
p

0
2
4
2
11.5.23 . T = T / 1 + B r T /(4LJ).
p
11.5.24 . = 2i 0 ah/[m(l d)].
q
11.5.25 . v = v0 (1 + 1 + LxI02 /(mv02 ) ).
11.5.26 . B = B0 + 20 v 2 /(Br0 ) 500 , P = B 2 (20 ) 1011 .
11.5.27 .
me

dv
r d(B B 0 )
= eE = e
,
dt
2
dt
347

B B 0 , . me v = er(B B 0 )/2. , ene vh = j, B 0 = 0 j, j


. j = e2 hB/(2me + e2 r0 ne h),
me B
B B 0 =
= 5,7 105 .
me + e2 r0 ne h/2
11.5.28 . B = 2me /e.
11.6.
dN
1 dN
( ); CB = 0 0
( ). CB c dt
dt
, N , c , 0 0
.
dN
dN
1 dN
11.6.2. .
= vlE, CB = 0 0 vlE, CB = 0 0
( ), CB =
( ).
dt
dt
c dt
5
11.6.3. N = 9 10 .
11.6.4 . N = Q/0 ,
1 dQ
1
dN
=
=
I, I
Q ,
dt
0 dt
0
DN
. CB = 0 0
= 0 I
dt
, I.
11.6.5. B = 2,5 106 .
11.6.6. n = 2N r/L.
11.6.7. B = 0 0 Ev cos .
11.6.8. = B/(0 v).
11.6.9. . B = 0 0 vV /h , B = 0 0 vV /h .
. ( + 1)/( 1) .

11.6.10 . . . - ,
abb0 a0 , , .
.
11.6.1. CB =

11.6.11 . . , ,
B1 = r2 0 0 /(2r) = 0 0 r/2.
, , 1 :
B2 = ( 1)B1 . B = B1 + B2 = B1 = 0 0 r/2.
0 0 V r
0 0 V 2
. B1 =
, B2 =
[r ( 1) + r02 ].
2h
2hr0
0 Ix
, r0 > r > r

11.6.12 . . . B0 = 0 Ir/(2r02 ). x < r B =


2r02
2
2
2
0 Ir (r0 x )
B =
, x > r0 B = 0.
2x2 r0 (r0 r2 )
348

12.
12.1. ,
12.1.1. z.
12.1.2. ), )
.


2
12.1.3. E = E0 sin
(z ct) .

E12 + E22 + 2E1 E2 cos(1 2 ),



E1 sin 1 + E2 sin 2
z
+ arctg
.
= t
c
E1 cos 1 + E2 cos 2
h

i
z
1 2
E cos2 t
+ .
12.1.5. E = 2E0 , w =
2 0
c
12.1.6. B = E/c( ), B = E ( ).

12.1.7 . B = E e/c ( ), B = E ( ).

12.1.9 . B = E /c ( ), B = E ( ).
12.1.10. . . 1/2, 1/2; 1,0; 1/2, 1/2.
12.1.4. E0 =

12.1.11. . , . d,
E/2.
. , AB A0 B 0
. cB/2.
1v
12.1.12. . E =
E.
2c

. .

, (v/c) sin . , ,
Q2 /(2r), (v/c)2 ,
349

sin . -
. ,
Q2  v 2
Q2  v 2
W =
( ), W =
( ).
3r c
120 r c
. . . .
12.1.13. .


c
1
c
k+
,
k0 = k, k00
d
d
2
k .

12.1.15 . . . . t A
E = E1 + E2 , E1 E2 ,
:



1
x
1
x+d
1
1
E1 =
Evtx/c =
Ea t
, E2 = Evt(x+d)/c = Ea t
.
2c
2c
c
2c
2c
c
, E = E1 + E2 = adE/(2c2 ).
. E = 0 ci0 /2 = i0 /(2c0 ) ( );

E = 2i0 /c ( ).

eE0
cos t.
me
, ,
eE0 ne e
E =
. k = (E /E0 )2 = [ne e2 x/(4me 0 c)]2 .
me 2c0
, ,
. , ,
. - , , , , , -
, /2, .
. ,
.
. E0 sin t ( = 2) v =

12.1.16. = 4 105 .
12.1.17.
( x < x1 ).
, . x > x2 .
12.1.18 . 4me 0 c/(ne e2 ) 105 .
12.1.19. E = 0, B = 2E/c.
12.1.20. = 4 105 , x = 2 105 .
12.1.21. j = 20 cE, P = 20 E 2 ( ); j = cE0 /(2), P = E 2 /(2) ( ).
12.1.22. P = c0 E02 .
12.1.23. P = 2 , P = 0,5 .
350

12.1.24 . r 1 .
12.1.25. . . ) E 0 = E, B 0 = B. ) E 0 = E.
p
12.1.26. E = P 0 cos2 .
12.1.27 . ,
, 
v 
, F = e E B = 0. E/B = v/c.
c
1 2c+v
2c+v

( ); P =
E
( ).
12.1.28 . P = 20 E0
cv
8 0 c v
12.1.29. v = c/(20 + ).
k1
12.1.30. v = c
.
k+1

12.1.31. . , .
.
. , . .

12.1.32.

E0 = En , B + B0 = Bn , B = E 1 /c, B0 = E0 1 /c,
: E

Bn = En /c (. 12.1.7 ). E0 /E = ( 2 1 )/( 2 + 1 ).

1 < 2 E0 E , 1 > 2 . ,
, .

12.1.34. . . W1 = 7E02 r03 /3, W2 = 2E02 r03 .

351

12.2.

12.2.2. n , .
12.2.4. sin k = k/b, k .
12.2.5. k2 .
12.2.6. = 13,50 .
12.2.7. l = 2r 2 /, lk = 2r 2 /[(k + 1)k].
12.2.8. .
12.2.9 . .
12.2.10. . .

12.2.11 . 100 () 324 ().


12.2.12 . c = i/, a = ASi/(r) ( i
/2).
12.2.13. . R 1 . . R 1,5 .
12.2.14. l 1 , 0,5 , 150 .
12.2.15. .
.
. -
.

13. . .

13.1.

352

13.1.1. . .
13.1.2. . .
13.1.3. . .

13.1.4. . ,
.
13.1.5. .
,
, 180 .
13.1.6. H = h/2.
13.1.7. .
13.1.8. .
.
13.1.9. : .
12.3.10 . = 120 .

13.1.11. .
13.1.12. x = h/2.
13.1.13. f = R/2.
13.1.15. f = 36 .
13.1.16. l = 20 .
13.1.17. f = 48 .

13.1.18. . .

13.1.19. . .

12.3.20 . , .
13.2. .
13.2.1. = 48 .
13.2.2. . h = 4 .

. h = 4 .
L+l
.
L + l/n

13.2.3.

13.2.4. n = 1,13.
13.2.5. . = 24,6 , = 49 , , = 33,7 .
. - .
13.2.6. .
13.2.7. R = ln/(n 1).
23

353

13.2.8. > arccos(2/3) .


< arccos(2/3) .

2, .
13.2.9. r = R/2.
13.2.10 . r 0 = r/n.
13.2.11. sin = n/kN 1 , n/kN 1 < 1; , n/kN 1 > 1.

1  n0
13.2.12. H =
R .
2


1
1
1
13.2.14. .
= (n 1)
+
.
F
R1
R2
13.2.15. . F = 0,25 , D = 1/F = 4 . . R = 0,6 .
13.2.16. R = 0,26 .
1
n1
1
1
n1
1
13.2.17 . )
=

; )
=
.
x
nf
nr
x
f
r
2
1
R
13.2.18. f =
.
(n 1)
13.2.19. = (n1 n2 )f .
12.2.20 . ,
= 2 arcsin(1/n).
12.2.21 . n = 4/3.
13.2.22. n = 3/2.
13.2.23. y = x/n.
13.3.

13.3.1. . .: ) k = 1/2;

13.3.2.
13.3.3.
13.3.4.
13.3.5.

. .
f = 20 .
f = 2f .
v = 2f .

f2
.
(a f )2 l2 /4
13.3.7. f = 3/7 .
13.3.8. t = 5 .
df
vk
db
13.3.9.
=
,
= vk.
dt
1 + k dt
13.3.6. k =

354

) k = 3/2;

) k = 1/4;

) k = 3/4.

r1 r2
f.
D/2 + r2
13.3.11 . 2,3 < l1 < 3,2 ; 1,6 < l2 < 8 .
13.3.12. D1 = 5 , D2 = 2 .
13.3.13. .
13.3.14. , 25 . k = 25/x, x
.
13.3.15. f = 2,5 .
13.3.16 . tg 0 = (1 /f ) tg .
13.3.17. k = 2.
13.3.18. . l1 = f (a 2f )/(a f ), l2 = a f . . f 0 = f /2.
f1 f2
1
1
1
13.3.19. f 0 =
;
=
+
.
f 1 + f2
f0
f1
f2
13.3.20. f 0 f /2 + 3l/4 .
13.3.21. , 10 .
13.3.22. f > 0,6 .
(d R)f
df
fd
13.3.23. x1 =
, x2 =
d > R + f ; x =
f < d < R + f ;
dRf
df
dF
d < F .
13.3.24 . n = (R l/2)/(R l).
n
n(n0 1)
13.3.25 . f1 = f ; f2 =
f.
2
2(n0 n)
13.3.26. h2 /h1 = (l f )f .
13.3.27. a/(a f ) .
13.3.28. 25l/f1 f2 ; k .
13.3.10. l =

355

13.4.
13.4.1. h = 1 .
13.4.2. E1 = 130 , E2 = 71 , E2 = 25 .
13.4.3. E = 41 .
I[h2 + (h + 2x)2 ]
.
13.4.4. E =
h2 (h + 2x)2

13.4.5. . .
13.4.6. 80 000 .
13.4.7. I 0 = I(1 k)/(1 + k).
13.4.8 . x 5 . .
13.4.9. N 0 /N (R/r)2 , R ,
r .
13.4.10. x R2 /r, r .
13.4.11. : -
, .
13.4.12. .
13.4.13. E /E = (R/2l)2 .
a2 f 2
. x = a/2.
13.4.14 . E = E0
[xf (a x)(x f )]2
2
2
13.4.15. L = L0 D /D0 .
13.4.16 . . E = BD2 /R2 .
13.4.17. D = 1,85 .


4x2
T 4
13.4.18 . N 2
770, T 6 103 .
d
T
13.4.19. .
13.4.20. .
13.4.21 . d2 = 5d1 /6.


k1 + 1 2
13.4.22 . t1 = t2
.
k2 + 1

2
1
13.4.23 . n (10r L) .
13.4.24 . = 0,2 /3 .
13.4.25 . .
13.5.
13.5.1. E1 106 /2 , E2 4 106 /2 , E3 4 105 /2 .
13.5.2. W = h eV0 .
13.5.3.
. = 1,24 1020 .
1 2
sin(1 + 2 )
13.5.4. . v = c
. . v = c
.
1 + 2
sin 1 + sin 2
h 2
h 0
13.5.5. . m = (1 cos ) 2
. . = (1 cos )
.
c
me c2
13.5.6 . )
mv 2
m(v v)2
=
+ hv + ,
2
2

mv = m(v v) +

hv
.
c

(1)

, ,
mv 2
m(v + 0 v)2
=
+ h + ,
2
2

mv = m(v + 0 v)

h 0
.
c

(2)

(1) (2) m , v 0 v ,
, 0 . v, 0 v  v (1) (2) ,
1 v/c
0 =
. ) 0 = (1 v/c).
1 + v/c
356

13.5.7. .
13.5.8. = M/(Rc2 ), . 109 .

, .
13.5.9 . f R2 c2 /(6M ) 109 .

14.
14.1. .
14.1.1. l = 15 .
14.1.2. v = 6 107 /.
14.1.3. tg = v/c.
.c

v
1
+ cos ' sin = 104 sin , v = 30 /
14.1.4 . tg = sin
2
v
c
.

14.1.5. ,
l , 3l/c, l/v, v
. : 3l/c = l/v. , v = c/3. , c.
v.
, , l0 /(c + u), l0 , .
l0 /(c u),
l0 /(c + u) + l0 /(c u) + l0 /(c + u),
l/u. l0 /(c + u) + l0 /(c u) + l0 /(c + u) = l0 /u , u = c/3. ,

, c/3.

14.1.6 . L/(c + u), L


. .
l
2
u
c+u
cu
. (. ) l = L
c+u
l
cu
=L
,

c+u
(c + u)2
3c + u
L
. , (c + u)2
L
v1
. ,
v1 + u
3c + u
L
cu
=
, v1 = c
.
L
(c + u)2
v1 + u
3c + u

23
357


L0
L0

.
u (. ),
c
c
L0 L0 u/c
L0 c u

=
.
c+u
c c+u
. ,
L0 c u
L0 3c u
L0
+2
=
,

c
c c+u
c c+u


,
. 3c u L0 
c+u
0
: v1 ' L

=c
.
c+u
c
3c u
: cu
c+u
c
, c
.
3c + u
3c u

14.1.7 . .

, , . ?
, c + v, c v.
, .
, . ,
. ,
, , ,
(c + v)/(c v) , , .
,
. ,
.
358

x l x
. :
x
x 
v
lx
x 
v
+
1+
=
+
1
.
c
cv
c
c
c+v
c
1
v
1
v
x =
1
l, l x =
1+
l.
2
c
2
c

1
v l
lx
1
v l
x
1 = =
1
, 2 =
=
1+
,
c
2
c c
c
2
c c

1 2 =

v
l.
c2


 1 , 2 , x l x: x1 = x2 =
v2
1
1 2 l.
2
c


1
(k 1)(1 2 )
c, v2 =
c.
k
(k 1)(1 + ) + 1

14.1.9. . . 1 2 , (1 + 2 )/2
, a c(1 2 )/2 ,
. ,


14.1.8. v1 =

v=

k1
c(1 2 )/2
=c
,
c(1 + 2 )/2
k+1

k 1 /2 . , ? . .
.
v0 =

(c u)1+ + (c u)2
2(1 + 2+ )

1,2
1,2
1 2 .
:
+

1,2
+ 1,2
= 1,2 ,

1,2
/1,2
= (c u)/(c + u),

, 1,2
= (1 u/c)1,2 /2,

v0 =


1

u2
c2


(k 1)c

 .
.h
i 
u
u2
vu 
k + 1 (k 1) = 1 2 v
1 2 .
c
c
c

v 0 v , , k, u .
359

v 0 , ?
1 2
10 20 , , : 1 : 2 = 10 : 20 .
, v 0 ,
. , , , ,
.
, , ,
, u.
u
v0 v =

vu(v u)
v(1 u2 /c2 )
v = 2
= v,
1 uv/c2
c uv

= 104 . u ' c2 /v = 90 /. ,
:
v0 = v 0 = u = v

c2 u2
vu
u=
' 100 000 / 90 / = 99 910 /.
c2 uv
1 vu/c2

14.1.11 v = 2,9 108 /.


14.1.12 u = (v + c/n)/(1 + v/nc).
2nl
14.1.13 T =
.
c(1 v 2 /c2 ) s
14.1.14 . v =

L c2
l(l + 2L)

1+

!
l3 (l + 2L)

1
; l/ , L/  c v = l2 /(2L ).
(L c)2

p
14.1.15 v0 = (c2 vu (c2 v 2 )(c2 u2 ) )/(v u).

 

u
u2 .
vu
u2
14.1.16 N = 1 + 2 + 2
1+ 2 + 2 .
v
c
c
c

14.1.17
.
l.
2l/c,
p
(2l/c) 1/ cos = 2l/c 1 2 , = v/c =
sin . ,

p
1/ 1 2 .
360


14.1.18 .
. . ,
, . , , :

.
. 3 : 2.
, , .

14.1.20
p
p
1/ 1 2 , 1/ 1 2
. l0 ,
, l0 :
1 =
1/
,

l0
l0
2l0
+
=
.
c(1 + )
c(1 )
c(1 2 )

p
1 2 2l/c, .

2l0
2l
= p
.
c(1 2 )
c 1 2
p
, l0 = l 1 2 .
p , ,
, 1/ 1 2
c.
.
.
p , 1/ 1 2 ,

1 =

361

p
1/ 1 2 .
,
, , , c c, c. , ,
c + c, , , c.
.
, , ,
.

p
14.1.21. 1 u2 /c2 + u2 /v 2 .
14.1.22. .
14.1.23. . . + = /2, = 2, = 5/4.

14.1.24 . N =p
(1 + )/2.
14.1.25 . ' /c.
sin + 2 + 2 sin
.
14.1.26 . sin 1 =
1 + 2 sin + 2
14.1.27. , u sin , , vp
362

vk ; vp vk
vp = u sin

q
1 (u/c)2 cos ,

vk = (v u cos )

.
vu cos 
1
.
c2

, , v
vk , vk ,
q
v = vp /

1 (vk /c)2 ,

vk = v k ,

v1
q
.
q
vu cos 
2 + v2 =
v1 = v
u2 + v 2 2vu cos (vu/c)2 sin2
1
.
k
c2
p
14.1.28 . tg = tg (/2), = 1/ 1 2 .
14.2. , .

14.2.1. 2,5 .
p
14.2.2. v > c/ 1 + ( c/l)2 .
14.2.3. v = 6 104 /.
14.2.4. = 107 .
14.2.5 . , ,
.
0
/(1 + ) = 0 /(1 ),

0 = (1 )/(1 + ).

14.2.6 . , , . 0
/(1 + ) = 0 /(1 + n),

0 = (n 1)/(1 + n).

p
14.2.7. = l(1 vu/c2 )/v 1 u2 /c2 .
4
14.2.8. 5 10 .
p 14.2.10. c . l/ ( =
1 1 2 ) 2 l. , , .
c , , c/, .
.

14.2.12 . tg = 1 /

q
1 12 .

14.2.14. v = c02 (12 22 )/(12 02 )(22 02 ).


14.2.16. cos = (cos + )/(1 + cos ),

0 = (1 + cos )/

p
1 2.
363

p
14.2.17. ) = L/(v + u), 2 = 1 (1 + vu)/ 1 (u/c)2 ;
s
!
r

v
2al
va1  .
v2

) 1 =
1 + 2 1 , 2 = 1 1 +
1 2.
2
a
v
2c
c
14.2.18. c.


 0
t0
z 0
A
0
0
sin
1+
; ) y 0 = A sin t
,
t : ) z =

c
p
= 1 1 2.
14.3.
p
14.3.1. = 1/ 1 2 ,
.
:
E 0 = E,

B = E 0 = E.

p
14.3.2 . E = E cos , Ek = E sin , B = E cos = E , = 1 1 2 .
p
14.3.3. Er = 2/r, Br = 2/r, = 1/ 1 2 , r .
p
14.3.4. . e = /, i = , = 1/ 1 2 .
. .
.
0 = / = 2 .
E = 2 s/r,
B = s/r, s , r . E = B.
q
14.3.5 . . i = 1 , 1 = 1/ 1 12 .
1 c
p
c, , 0e = /, = 1 1 2 . ,
2 c = c(1 )/(1 1 ),

q

, e = 0e / 1 22 = 1 (1
1 ).
. 1 .
. E1 = 1 B1 .
~ B].
~ = [
~
14.3.6. . E
~ B],
~
. E E = [
0
0
~
~
~
B . B B.
~B
~ ' [
~ 0 ].
E
. . , .

14.3.7 . .
, c . , +. . c
, c.
, .
/ = 2 ,
E 2 E.
364

. , E , : IB/c + 2 E = 0,
I = c , B , , . ,
B,
~ E].
~ = [
~
B
~ =
. ( ) B
0
0
~
~
~
[ E ], E .
~ E].
~ 0 E.
~ B
~ ' [
~
E
p
p
2
2
14.3.9. ) 1/
p 1 ; ) 1/ 1 .
2
14.3.10. 1/
p 1 .
p
14.3.11. Emax = Q/(R2 1 2 ), max = Q/(4R2 1 2 ), min = Q/(4R2 ).

14.3.12 . ,
p c.
= 1/ 1 2 .
, .
E , Ek . .
-. -
-.
r, c - r.
, ,
2 , 3
.
1 2
~ = q
~
r.
E
3
r
(1 2 sin2 )3/2
. ~ E].
~ = [
~
B

14.3.13 . c ,
c.
p
= 1/ 1 2 ,
. , c.
E 0 = 4/ B 0 = +4.
c . ,
14.3.8.:
E = 4 2 (1/ 2 ),

B = 4 2 (1 1/).
365

14.3.14 . c ,
p
B, = 1/ 1 2 B. ,
, , : E 0 = B/.
c , , 14.3.8.:
E = 2 (1 1/)B. , , U = Eh = 2 hB(1 1/).
p
14.3.15. (1 + )/(1 ) .
p
14.3.16 . (1 + /n)/ (1 2 ) .
14.3.17. (1 + )/(1 ) .
14.3.18 . (1 + sin )/(1 sin ) .
q
14.3.19. (1 + 1 )/ 1 12 ) .
p
14.3.20. 1/ 1 2 ; = j/c.

14.3.21 . .
14.3.22. E = 4[ j(t0 x0 /c)] = 4[ jt0 + l0 2 /c].
14.3.23 . .
. E = 4( jt).
, c,





x0
j
l0
E 0 = 4 j t0
= 4 t0 +
,
c

c
p
l , = 1/ 1 2 .
14.3.24. P = vM .
14.3.25 . P = vM .
3 , R = e2 /(2m v 2 ).
14.3.26. F = 2ev/R
e

14.3.28 . . F , ,
Fk = QE cos ,

F = QE sin (1 2 ),


ak = k cos ,

a = k sin ,

k = QE

p
1 /M,

Q, M , E , .
,
. .
14.4.

14.4.1. ) c
p
= 1/ 1 2 :
T = .
366

) 2me c, = 2me c/(eE), E .


T 1 me c/(eE), 1 c = 2c(1 + 2 )
. T = .
14.4.2 . , ,
q
2me v1
q
= 1 u2 /c2 ,
1 =
eE 1 v12 /c2
v1 = (v + u)/(1 + vu/c2 ).
E = 2me (v + u)/[e (1 u2 /c2 )

q
1 v 2 /c2 ].

p
14.4.3. E = me v/(e 1 v 2 /c2 ).
p
14.4.4. ) 1 1 u2 /c2 .
q
v 0 = v 2 + u2 v 2 u2 /c2 .


q
1
u
) p
1 + (1 1 v 2 /c2 ) .
v
1 u2 /c2

14.4.5. =

me v
e E

v 0 = (v + u)/(1 + vu/c2 ).
!
1
1
p
p
.
1 4v 2 /c2
1 v 2 /c2

me c2
.
eE p
0
14.4.7. p = p. 1/ 1 2 .
s


me c2 R 2
14.4.8. v = c/ 1 +
.
2
e z
s
14.4.6. x =

14.4.9 . 1/(1 2 ) .

sin2 +

cos2
.
(1 2 )2

c
.
14.4.10 . v = p
1 + (mc/2qE)2
p
2
14.4.11.
p ) c 1/ 1
0
. l = l 1 2 .
)
!
!
me
me
1
2
2
p
c m = p
me c = eEl, l =
1 .
eE
1 2
1 2
, c,
!
me c2
1
p
l1 =
1 ,
eE
1 2

. l = c ,
p
= me c/(eE 1 2 c.
l0 = l1 + l =

p
p
me
(1 1 2 ) = l 1 2 .
eE
367

!
q
me c2
1 + uv/c2
p
14.4.12 . E = p
1 u2 /c2 .
1 v 2 /c2
1 v 2 /c2
p
14.4.13. = (2 lEl/me c2 )me l/eE.
q
E
1
14.4.14. l =
= 1 . =
m0 E(2 + E/m0 c2 ) = 0,34 .
eE
eE
2
6
14.4.15. 2N = 1 = 2 10 me c2 . k = 2N (1/N ) ' 2000
.
me c2 + E 2mp c2 + E
me c2 + E 2mp c2 + E

tg p p  1, e '

p =
14.4.16. tg e =
2
2
2me c + E mp c + E
2me c2 + E mp c2 + E
0,075 .
s


me c2
v
v 2 cos2
me c2 2
14.4.17. v1 =
+
+

.
l
e
sin2
sin4
p
14.4.18. E = (mp c2 )2 + (eBR)2 mp c2 = 4,3 . Ee = 80,5 .
sin
E(E me c2 ) = 0,04 , N ' 4 %.
14.4.19. B =
el q
1
1
1
E 2 (mp c2 )2 = ; Re = .
14.4.20. Rp =
eB
3
3
eB
14.4.21. =
.
me c(1 + eU/me c2 )

14.4.22. B = me c2 /(eR N 2 1 = 0,28 .


p
14.4.23. T = me c2 /(eB 1 2 .
me c2 (1 + 1 )
p
14.4.24. T =
.
eB(1 + 12 ) 1 2
p
14.4.25. E = (me c2 )2 + (eBh)2 me c2 = 8,5 .
me vc
p
14.4.26. l =
.
eB 1 v 2 /c2
p
14.4.27. E = (me c2 )2 + (eBR)2 [1 + (h/2R)2 ] me c2 .
E
14.4.28. v = c .
B

14.4.29 . B =
1 k2 ,
k = E/B, , kc,
B E,
.

vmax = c

1 + k
,
1 + 1 k

vmin = c = c

1 k
,
1 1 k

1 c .
vmax = c[2k + (1 + k2 )]/(1 + k2 + 2k).
p
14.4.30 . ev = (me c2 )2 + (hH)2 me c2 .
14.4.31 . c E

1 c kc. k = E/B (. 14.4.12).
p
1 = 2 (1 k2 ) + k2 , vmax = c(1 + k)/(1 + 1 k).
14.5.
14.5.1. m = M/2.
14.5.2.m = W/c2 = 4,4 /.
p
p
14.5.3. m1 = m/2, m0 = m 1 2 /2, E = mc2 (1 1 2 )/2.
368

p
14.5.4. m = (k + 1)mp , v = c 1 1/k2 .
2
14.5.5. E1 = c (mp me ) = 938 , E2 = c2 (m0 me )/2 = 67 .
14.5.6. M1 = M + m, p = mc.
14.5.7 . v = cm/(M m).
p

2mt),

t < M/(2m).
14.5.8. v = cmt/(M

mt),

m = mt(M
p0
14.5.9. M = m1 + m2 , v = (m1 v1 )2 + (m2 v2 )2 + 2m1 m2 v1 v2 cos /(m1 + m2 ).
14.5.10. me = 0,51 , mp = 939 , m0 = 135 , m = 2820 .
14.5.11. EK = (E m0 c2 /2)/E = 152 .

2
m0
14.5.12 . EK < 2me c2
1 .
p 4me
14.5.13. EK = M c2 (M c2 )2 Ee (Ee + 2me c2 ) E2 .
14.5.15. v = c cos
.
2
14.5.16. E1 = c2 m0 (m0 + 4mp )/(2mp ), E2 = c2 m (m + 4mp )/(2mp E3 ) = 6mp c2 .
14.5.17. E = 2c2 (m2p m2e )/me , N = 2(mp /me 1) = 3,7 103 .
r

14.5.18. E = mp c2 1 + (1 m2e /m2p ) ctg 2 .


2
m
m
14.5.19. . v =
c, m = m. . u >
c.
M"
# M q

(m + me )2
14.5.20. Emax = E 1
1 + 1 (m0 c2 /E)2 = 4,4 ; Emin = 0.
2
m0


2m2e
1
14.5.21.
m
c2 ,
2
m
p
(m me )2 c2 /(2m ).
E + Ee2 + (me c2 )2
p
14.5.22. Emax = E
.
2E + Ee Ee2 (me c2 )2
m
14.5.23. m =
, m0e = me + m m .
m
1+ m
(1 cos )
e

24

369

25.11.2007 .

70 100/16
. . . 30,4
.-. . 32,4
100 .

-
630090 , . , 2

You might also like